SlideShare una empresa de Scribd logo
1 de 199
REUMATOLOGIA
ENARM
CASO CLINICO
• A 57-year-old man presents with a swollen, painful left knee. He fell on
the knee 3 days ago while hurrying up the stairs. On examination, there is
a swollen knee with palpable effusion and decreased range of motion.
Adiagnostic tap is preformed and 5 cc of transparent fluid is removed. The
WBC count is 1000/mL (20% polymorphonuclear neutrophils [PMNs]),
glucose is equal to plasma, viscosity is high, and lactate dehydrogenase
(LDH) is low.
(A) normal synovial fluid
(B) noninflammatory effusion
(C) inflammatory effusion
(D) septic arthritis
(E) hemorrhagic effusion
(B) This man has a noninflammatory effusion likely
from the trauma caused by the fall. In the
noninflammatory category, the fluid is transparent,
WBC 200–2000/mL (<25% PMNs), glucose
is normal, and LDH is low. Another cause
for a noninflammatory effusion is osteoarthritis.
(Kasper, p. 2032)
CASO CLINICO
• A77-year-old woman presents with a swollen, painful right knee.
She fell on the knee 3 days ago while hurrying up the stairs. On
examination, there is a swollen knee with palpable effusion and
decreased range of motion. A diagnostic tap is preformed and 5 cc
of opaque fluid is removed. The WBC count is 20,000/Ml (50%
PMNs), glucose is lower than plasma, viscosity is low, and the LDH is
high.
(A) normal synovial fluid
(B) noninflammatory effusion
(C) inflammatory effusion
(D) septic arthritis
(E) hemorrhagic effusion
(C) This woman has an inflammatory category
of effusion in view of the opaque color, high
WBC 2000–10,000 (>50% PMNs), low glucose,
and high LDH. Common causes for this include
crystal induced arthritis, SLE, and RA. In septic
arthritis, WBC is usually 50,000/mL or more
and often >100,000/mL with >75% PMNs.
Other important tests on synovial fluid include
Gram stain and culture when an inflammatory
effusion is suspected clinically.
ARTRITIS REUMATOIDE
• 1.- En que enfermedades suelen ser positivos los ANA +?
 R = LES, esclerodermia, Sjogren y enfermedad del tejido conectivo
• 2. El FR que anticuerpo lo compone?
 R = IgM dirigida contra el fragmento fc de IgG
• 3. En que enfermedades se presenta comúnmente la trombocitosis?
 R = Vasculitis, enfermedad de Still y AR.
• 4. En que enfermedades se presenta comúnmente la trombocitopenia?
 R = LES y Sx antifosfolipido
• 5. Que tratamiento se utiliza en el fenómeno de Raynaud?
 R = Nifedipino
• 6. Cuales manifestaciones clínicas y de lab sugieren AR?
 R = Artritis de MAS DE 3 ARTICULACIONES, RIGIDEZ MATUTINA, nódulo reumatoide y
FR positivo lo sugieren.
• 7. Cual es el HLA implicado en la AR?
 R = HLA DR4
• 8. Cuales son los criterios revisados en 1987 para la clasificación de la AR?
1) Rigidez matutina MAYOR A 30 MINUTOS
2) Artritis de 3 O MAS articulaciones
3) Artritis de las manos
4) Artritis SIMÉTRICA
5) NÓDULOS reumatoides
6) FR positivo en suero
7) Cambios radiológicos
• 9. Cual es el órgano blanco más afectado en la AR?
 R = La membrana SINOVIAL
• 10. Cuales son las manifestaciones extrarticulares más comunes de la AR?
 R = ANEMIA NORMO-NORMO por uso de AINES con STDB, TROMBOCITOSIS, nódulos
subcutáneos y SEQUEDAD DE MUCOSAS (Sx Sjogren secundario)
• 11. Cuales son las manifestaciones provocadas por la AR en las glándulas
exocrinas y mucosas?
 R = Principalmente en las PARÓTIDAS, LAGRIMALES, mucosa y serosa de la
VRS y CUELLO UTERINO produciendo DOLOR Y AUMENTO DEL VOLUMEN
• 12. Cuales son las manifestaciones oftalmológicas de la AR?
 R = EPIESCLERITIS y ESCLEROMALACIA (adelgazamiento).
• 13. Cuales son las manifestaciones cardiovasculares de la AR?
1) VALVULARES: MITRAL Y AORTICA.
2) VASCULARES: Vasculitis y ateroesclerosis.
• 14. Cuales son las manifestaciones renales de la AR?
 R = MICROALBUMINURIA o MICROHEMATURIA.
• 15. Que manifestación pulmonar da la AR?
 R = FIBROSIS pulmonar manifestada en RX COMO PANAL DE ABEJA.
• 16. En que consiste el Síndrome de FELTY?
 R = AR, ESPLENOMEGALIA y LEUCOPENIA a expensas de neutrófilos.
• 17. En que consiste el síndrome de CAPLAN?
 R = AR, NODULOSIS reumatoide pulmonar y NEUMOCONIOSIS.
• 18. Que anticuerpos en la AR tienen la mas alta especificidad diagnostica?
 R = ANTICUERPOS ANTIPEPTIDOS CITRULINADOS ANTI- CCP
• 19. Cual es el manejo de la AR?
1) Los FAME (fármacos moduladores de la enfermedad) deben utilizarse
cuando se hace el dx y antes de que aparezcan los cambios erosivos.
2) La CLOROQUINA y el METOTREXATE son INDUCTORES DE REMISIÓN en
AR.
• 20. Cual es el efecto secundario mas común de la CLOROQUINA, la cual es
un inductor de la remisión de AR?
 R = Musculopatia (miopatía).
ARTRITIS CRONICA JUVENIL
CASO CLINICO
• A 25-year-old woman develops painful swelling of both hands and
wrists. She is also very stiff in the morning. Physical examination
reveals erythema, swelling and joint line tenderness of the proximal
interphalangeal, MCP, and wrists joints. Her RF is positive, ANA is
negative, and x-rays of the hands show early erosive joint changes.
Which of the following medications is most likely to prevent
progression of disease?
(A) D-penicillamine
(B) antimalarial
(C) methotrexate
(D) NSAID or aspirin
(E) gold
(C) Methotrexate, 7.5–20 mg once weekly, is
the most commonly recommended disease
modifying drug, because its effect is more rapid
and patients are able to tolerate it for longer
periods of time. Maximum improvement with
methotrexate occurs after 6 months of therapy.
Toxicity includes GI upset, oral ulceration, and
liver function abnormalities. GI upset in particular
may be ameliorated by concurrent folic
acid administration. Pneumonitis has also been
reported.
CASO CLINICO
• A 29-year-old woman develops symptoms of painful
swelling, and stiffness of both hands. Physical examination
reveals involvement of the proximal interphalangeal joints
and MCP joints. Aclinical diagnosis of rheumatoid arthritis
is made. Which of the following is the most likely cause of
the inflammation in her joints?
(A) activated T cells
(B) antineutrophil antibodies
(C) microvascular injury
(D) interleukin-4 (IL-4)
(E) precipitated RF
(A) Numerous mediators of inflammation are
found in the synovium of patients with
rheumatoid arthritis (RA). The evidence favoring
activated T cells as the initiators of the
inflammation include the predominance of
CD4+ T cells in the synovium, the increase in
soluble interleukin-2 (IL-2) receptors (a product
of T-cell activation), and amelioration of symptoms by T-cell
removal.
CASO CLINICO
• A 63-year-old man is worried that he has RA because his RF
serology is positive in a low titer. He has pain in his left hand and
right knee, which usually bother him in the evening. He has not
noticed any inflammation or swelling and there is no history of
morning stiffness. On examination, there are no active joints. Which
of the following statements regarding the presence of RF is correct?
(A) is positive in 10–20% of people over age 65
(B) is positive in almost 100% of “classical” RA
(C) is seen only in RA
(D) is always abnormal
(E) is frequently present in osteoarthritis
(A) The presence of RF has little predictive
power in determining the diagnosis of RA.
However, it is useful in determining prognosis,
as high titers of RF are associated with more
severe and progressive disease, as well as with
extra-articular manifestations.
CASO CLINICO
• A 30-year-old woman develops painful swelling of her hands,
pleuritic chest pain, and shortness of breath on exertion. She is also
very stiff in the morning. Physical examination reveals involvement
of the proximal interphalangeal joints and MCP joints. Air entry is
decreased to the right lower lobe and the area is dull on percussion.
Her RF is elevated and a CXR reveals a pleural effusion. A diagnostic
tap is performed, and 500 cc of a straw-colored fluid is removed.
Which of the following biochemical patterns is consistent with a
pleural effusion due to her primary condition?
(A) exudate protein and LDH ratios with low glucose
(B) exudate protein and LDH ratios with high glucose
(C) transudate protein and LDH ratios with high glucose
(D) transudate protein and LDH ratios with low glucose
(E) normal pleural fluid
(A) Pleuritis is common at autopsy in patients
with RA but is not usually symptomatic.
Typically, the pleural fluid shows increased
pleural fluid: serum protein and LDH ratios
(exudate pattern), and low glucose and low
complement levels. Pleuropulmonary
manifestations are more common in men with
CASO CLINICO
• A 54-year-old woman with rheumatoid arthritis (RA) presents with
fatigue and low white count (white blood cells [WBC] 2500/mL) on
routine blood work. She has no active joint symptoms and her RA is
controlled on low dose methotrexate and NSAIDs. On examination,
she has chronic joint deformities of her hands and a palpable
spleen, which is a new finding. Which of the following is the most
likely diagnosis for her low white count?
(A) methotrexate therapy
(B) rheumatoid nodules disrupting bone marrow architecture
(C) Felty’s syndrome
(D) normal variation
(E) myelofibrosis
• (C) The triad of chronic RA, splenomegaly,
and neutropenia is called Felty’s syndrome. It
is associated with high titers of RF and
extraarticular disease. The increased
susceptibility to infections is secondary to
both decreased neutrophil number and
function. Felty’s syndrome is rare in African
Americans.
CASO CLINICO
• A 29-year-old woman develops painful swelling of both
hands. She is also very stiff in the morning. Physical
examination reveals involvement of the proximal
interphalangeal joints and metacarpophalangeal (MCP)
joints. Her RF is positive and ANA is negative. Which of the
following medications is most likely to improve her joint
pain symptoms?
(A) D-penicillamine
(B) an antimalarial
(C) methotrexate
(D) NSAID or aspirin
(E) gold
• (D) This patient has RA and aspirin or other nonsteroidal
agents are effective medications for relieving the signs and
symptoms of disease. They do little to modify the course of
the disease, however. The new generation of NSAIDs that are
more specific inhibitors of cyclooxygenase 2 cause less GI
toxicity. Glucocorticoids are very powerful at suppressing
signs and symptoms of disease and may alter disease
progression. Methotrexate is an important disease modifying
drug (DMRD) used to prevent joint destruction. Gold and
antimalarials were important DMRDs in the past before the
use of methotrexate and newer “biological” agents.
CASO CLINICO
• A 50-year-old man develops fatigue and painful swelling of
both hands. He is also very stiff in the morning and requires
longer time to get ready for work. Physical examination
reveals erythema, swelling, and tenderness on palpation of
the proximal interphalangeal joints and MCP joints. Plain x-
rays of the hand are taken. Which of the following x-ray
findings is characteristic of this condition?
(A) loss of articular cartilage and bone erosion
(B) normal
(C) osteolytic changes
(D) osteosclerotic changes
(E) osteolytic and osteosclerotic changes together
(A) This patient has features of rheumatoid
arthritis, and early in RA there may not be any
bony changes seen, except nonspecific findings
of soft tissue swelling and joint effusions. With
longer active inflammation of the joints, loss of
cartilage and bony erosions can be seen. The
value of x-rays is to determine the extent of
bone and cartilage damage.
CASO CLINICO
• A 40-year-old woman complains of being very “stiff” in the morning, in
addition to having sore hands and wrists. The symptoms have come on
very gradually and she can not recall exactly when they started. Her only
other complaint is that of being fatigued. Physical examination reveals
involvement of the proximal interphalangeal joints, MCP joints, and wrist
joints in a symmetrical fashion. She also has nodules over both elbows.
Which of the following is predictive of developing extraarticular features
of her condition?
(A) her knees are involved early
(B) there is a poor articular response to disease-suppressing medication
(C) humoral immunity is suppressed
(D) cellular immunity is suppressed
(E) she develops an antibody to her own immunoglobulin (RF)
(E) Extra-articular manifestations of RA generally
develop in patients with high titers of
autoantibody to the Fc component of IgG (also
known as rheumatoid factor (RF).
CASO CLINICO
• A 67-year-old man has a long history of symmetrical small
joint arthritis with deformities. He now develops shortness
of breath on exertion with a dry cough, but no sputum or
chest discomfort. His heart sounds have a loud P2, and the
lungs have fine bibasilar crackles. Which of the following is
the most likely diagnosis of the pulmonary condition
associated with his arthritis?
(A) pleuritis
(B) cavitating lesions
(C) intrapulmonary nodules
(D) interstitial fibrosis
(E) diffuse pneumonitis
(D) In RA, pleural involvement is very common
at autopsy but infrequently causes symptoms.
Interstitial lung disease (ILD) is the most
common manifestation of rheumatoid lung disease.
RA associated interstitial lung disease
(RA-ILD) is usually similar to idiopathic pulmonary
fibrosis (IPF) in terms of its clinical presentation,
pathology, disease spectrum, and
pathogenesis. Presentation is more common at
age 50–60 years, in men (M:F = 2–3:1), and in
association with seropositive and erosive joint
disease. If pleural fluid is present, glucose levels
are very low.
OSTEOARTROSIS
• 21. Cual es la causa de dolor e incapacidad mas
frecuente en ancianos?
R = La OSTEOARTROSIS
• 22. Los nódulos de Heberden (distales) y de Bouchard
(proximales) en que enfermedad se manifiestan?
R = Osteoartrosis
• 23. Cual es el órgano blanco de la OA?
R = CARTÍLAGO ARTICULAR con distribución asimétrica
• 24. Cual es el cuadro clínico y radiologico de la OA?
 R = Dolor articular, RIGIDEZ MATUTINA DE NO >30 MIN después de haber
realizado una actividad leve-moderada una noche antes, perdida de la
funcionabilidad, CRECIMIENTO DE OSTEOFITOS, micro fracturas, irritación de
terminales nerviosas.
• 25. Como diagnosticas OA?
1) HC, no se necesitan pruebas de laboratorio.
2) Radiográficamente: DISMINUCIÓN DEL ESPACIO ARTICULAR, presencia de
osteofitos o quistes subcondrales e incremento de la densidad ósea sub condral,
lo que se le llama esclerosis subcondral.
• 26. Cual es el tx de elección de la OA?
 R = ACIDO HIALURONICO, PARACETAMOL y demás AINES. El TRAMADOL se utiliza
en pacientes que no toleran AINES, inhibidores de la COX-2 o déficit renal grave.
• 27. Cual es el tx de la OA utilizado en pacientes intolerantes a AINES o con daño
renal?
 R = TRAMADOL, GLUCOSAMINA, GLUCOCORTICOIDES INTRARTICULARES en caso
de proceso agudo, HIALURONATO el cual restaura las propiedades elásticas y de
viscosidad del liquido sinovial, CIRUGÍA.
CASO CLINICO
• A 60-year-old man has pain in his left hand and right knee, which is
interfering with his work. The pain came on gradually, first in his hand 6
months ago and now in his knee. It is usually fine when he wakes up, but
gets worse as the day progresses. There is no history of any trauma, and
he is otherwise well. Taking over-the-counter NSAIDs usually relieves the
pain. On examination, there is bony soft tissue swelling of his second and
third DIP joints in the left hand and crepitus over the right knee with
flexion. There is no erythema or joint effusion. Which of the following
characteristics is a risk factor for this condition?
(A) being Chinese
(B) being African
(C) being male
(D) being overweight
(E) hyperthyroidism
CASO CLINICO
• A 67-year-old woman has pain in her left hand and right knee, which is
interfering with her activities. The pain came on gradually, first in her hand
6 months ago and now in her knee. It is usually fine when she wakes up,
but gets worse as the day progresses. There is no history of any trauma,
and she is otherwise well. Taking over-the-counter NSAIDs usually relieves
the pain. On examination, there is bony soft tissue swelling of his second
and third DIP joints in the left hand and crepitus over the right knee with
flexion. There is no erythema or joint effusion. Which of the following best
describes this disease condition?
(A) disease of the synovial membrane
(B) disease of the articular cartilage
(C) disease of the entire joint
(D) disease of the subchondral bone
(E) disease of the ligaments
CASO CLINICO
• Which of the following is the most common
location for osteoarthritis?
(A) hip
(B) base of thumb
(C) knee
(D) spine
(E) DIP joints of hand
(E) Heberden’s nodes, bony enlargement of the
DIP joints of the hand, are the most common
type of osteoarthritis. Although they can present
acutely with pain and inflammation, they
are frequently slow in developing and relatively
asymptomatic.
CASO CLINICO
• A 60-year-old man has pain in his left hand and right knee, which is
interfering with his work. The pain came on gradually, first in his hand 6
months ago and now in his knee. It is usually fine when he wakes up, but
gets worse as the day progresses. There is no history of any trauma, and
he is otherwise well. Taking over-the-counter NSAIDs usually relieves the
pain. On examination, there is bony soft tissue swelling of his second and
third DIP joints in the left hand and crepitus over the right knee with
flexion. There is no erythema or joint effusion. Which of the following
characteristics is a risk factor for this condition?
(A) being Chinese
(B) being African
(C) being male
(D) being overweight
(E) hyperthyroidism
(D) There are numerous diseases and risk factors
associated with the development of
osteoarthritis. People of Chinese and African
heritage have a lower risk than Caucasians,
while Native Americans have a higher risk.
Women are more affected than men, and obesity
is a significant risk factor. Hyperthyroidism
is not one of the many metabolic/endocrine
disorders associated with osteoarthritis.
CASO CLINICO
• A 57-year-old man has pain in his left hand and right knee, which is
interfering with his work. The pain came on gradually, first in his hand 6
months ago and now in his knee. It is usually fine when he wakes up, but
gets worse as the day progresses. There is no history of any trauma, and
he is otherwise well. Taking over-the-counter NSAIDs usually relieves the
pain. On examination, there is bony soft tissue swelling of his second and
third distal interphalangeal (DIP) joints in the left hand and crepitus over
the right knee with flexion. There is no erythema or joint effusion. Which
of the following is the initial change noticed in the pathogenesis of
osteoarthritis?
(A) abnormal chondrocyte function
(B) a defect in the extracellular matrix of cartilage
(C) inflammatory changes in subchondral bone
(D) ligament inflammation
(E) synovial inflammation
• (B) It is most likely that the primary change in
osteoarthritis occurs in the cartilage. It is
possible that there is a disruption of the
collagen network of the cartilage, specifically a
disruption of the “glue” holding together
adjacent fibers.
CASO CLINICO
• A 74-year-old woman has pain in her left hand and right knee, which
started months ago, and is now interfering with her activities. The pain
gets worse as the day progresses. There is no history of any trauma, and
she is otherwise well. Taking over-the-counter acetaminophen usually
relieves the pain. On examination, there is bony soft tissue swelling of her
second and third DIP joints in the left hand and crepitus over the right
knee with flexion. There is no erythema or joint effusion. Which of the
following is the most likely explanation for the joint pain of osteoarthritis?
(A) synovial inflammation is not the cause
(B) ligament inflammation is a common cause
(C) clinically visible (via plain x-ray) fractures are a common cause of pain
(D) osteophytes can cause pain
(E) muscles are not involved
• (D) Osteophytes can cause pain by stretching
periosteal nerve endings. Synovial inflammation
is frequently seen in osteoarthritis, but not
in ligament inflammation. Microfractures, but
not macrofractures, commonly cause pain.
Muscle spasm can be an important factor in
the joint pain.
Policondritis recidivante
• A 45-year-old man has had intermittent swelling and pain
in the superior part of his auricles for several years. Mild
arthritis usually accompanies these episodes. Last year he
also had redness, pain, and swelling over the bridge of his
nose. Which of the following is the most likely diagnosis?
(A) psoriatic arthritis
(B) Behçet’s syndrome
(C) Wegener’s granulomatosis
(D) relapsing polychondritis
(E) rheumatoid arthritis
(D) The disease is relapsing polychondritis and
is characterized by frequent remissions and
exacerbations of lesions and is rarely fatal.
Auricular chondritis and nasal chondritis are
the most common manifestations. It can also be
secondary to SLE, RA, Sjögren’s syndrome, and
vasculitis.
GOTA• 28. Cual es el cc de la gota?
1) Los primeros ataques suelen afectar una sola articulación siendo la mas frecuente LA
PRIMERA MTF (podagra), tobillos, RODILLAS (gonagra), tarsos, muñecas y dedos de las
manos, cursa con FIEBRE.
2) La cetoacidosis diabética, anemia hemolítica, síndrome de lisis tumoral y psoriasis cursan con
hiperuricemia.
• 29. Menciona algunos fármacos uricosuricos y cuando se trata la gota asintomática?
1) PROBENECID, benzobromarona, LOSARTAN y sulfinpirazona.
2) La gota asintomática se trata en casos de litiasis renal, acido úrico > 11, ataques de gota
reiterados y gota crónica.
• 30. Que birrefringencia te indica la identificación de cristales de urato sódico en el liquido
sinovial?
 R = Birrefringencia NEGATIVA intensa, tienen FORMA DE AGUJA.
• 31. Que característica macroscópica tiene el liquido sinovial en un px con ataque agudo de gota?
 R = Apariencia PURULENTA con predominio de PMN
• 32. Que medicamento esta contraindicado en el ataque agudo de gota?
 R = ALOPURINOL que es un inhibidor de la Xantina-oxidasa
• 33. Cual es el efecto adverso más importante de los uricosuricos?
 R = Pueden causar NEFROLITIASIS
• 34. Que causa la pseudogota?
 R = Cristales de PIROFOSFATO DE CALCIO
• 35. Que etiología tiene la Pseudogota?
 R = Hemocromatosis, hiperparatiroidismo, hipofosfatasia e
hipomagnesemia.
• 36. Cual es el cuadro clínico de la pseudogota?
 R = Ataque MONOARTICULAR u oligoarticular, PSEUDOPODAGRA, rodillas
(gonagra) y DEDOS DE MANOS (queiagra).
• 37. Cual es la birrefringencia de la pseudogota?
 R = Birrefringencia POSITIVA pero débil, tienen FORMA DE BASTONES O
ROMBOS.
• 38. Cual es el tratamiento de elección de la pseudogota?
 R = PARACETAMOL
• 39. Cual es el tratamiento profiláctico de la pseudogota?
 R = COLCHICINA .6 mg c/12
CASO CLINICO
• A 72-year-old man injures his right knee in a car accident,
and now it is swollen and extremely painful to bend the
knee. X-rays of the knee rule out a fracture, and joint fluid
aspiration reveals an opaque-colored fluid containing
rhomboid crystals with weak-positive birefringence. Which
of the following is the most appropriate next step in
management?
(A) oral prednisone
(B) intravenous antibiotics
(C) oral NSAIDs
(D) acetaminophen
(E) allopurinol
(C) In traumatic arthritis, swellings, ecchymoses, muscular spasms, and
tenderness tend to be present, but fractures must be excluded. This
man has calcium pyrophosphate (CPPD) crystalinduced
monoarthritis, so called “pseudogout.” It is most common in the
elderly and can be precipitated by minor trauma. The crystals have
a rhomboid shape, and the clinical presentation can mimic that of
gout. It can be associated with metabolic abnormalities such as
hyperparathyroidism or hemochromatosis. Treatment is with an
NSAID for 7–10 days. If there are multiple joints involved, then
steroids can be considered. An alternative to oral NSAIDs is intra-
articular steroids for single joint disease. Allopurinol is not effective
in CPPD. Synovial fluid is bloody, but the fluid is of normal viscosity,
so a “string” test is usually positive.
CASO CLINICO
• A63-year-old man develops pain and swelling in his
knee. It appears warm, red, and swollen with
decreased range of movement. A diagnostic aspiration
is performed. Which of the following will most likely
distinguish pseudogout from gout?
(A) positive birefringent crystals
(B) acute onset
(C) involvement of single joints
(D) involvement of large joints
(E) association with diabetes
• (A) Pseudogout (calcium pyrophosphate
crystals—CPPD) is distinguishable from gout
by positive birefringent crystals. CPPD are
short, blunt rhomboids, and urate crystals (seen
in gout) are needle-shaped with negative
birefringence.
CASO CLINICO
A 63-year-old man with an 8-year history of
recurrent severe arthritis in his large toes has an
elevated creatinine level. Which of the following
mechanisms is the most likely explanation for
his renal impairment?
(A) GN
(B) vascular injury
(C) uric acid kidney stones
(D) distal tubular atrophy
(E) renal parenchymal uric acid crystals
(E) The typical renal lesions in gout are urate
crystals in the medulla or pyramids, with surrounding
mononuclear and giant cell reaction.
The degree of renal impairment, however, does
not correlate with hyperuricemia, and the
decline in renal function correlates with aging,
hypertension, renal calculi, or unrelated
nephropathy. (Kasper, p. 1704)
ESPONDILITIS ANQUILOSANTE
• 40. Que espondiloartropatias se asocian al HLA-B27?
 R = ESPONDILITIS ALQUILOSANTE, ARTRITIS REACTIVA, ESPONDILOARTROSIS
PSORIASICA, uveítis anterior aguda.
• 41. Cual es el síntoma inicial de la espondilitis alquilosante (EA)?
 R = DOLOR en regiones SACROILIACAS Y LUMBARES. En la ex fis HAY DISMINUCIÓN DE LA
MOVILIDAD DE LA COLUMNA LUMBAR, dolor al presionar las regiones sacroiliacas y disminución de la
expansión del tórax.
• 42. Cual es la manifestación extrarticular mas frecuente de la EA?
 R = UVEITIS ANTERIOR, 2/3 de px tiene CAMBIOS EN LA MUCOSA Y SUBMUCOSA DEL COLON E ÍLEON
TERMINAL
• 43. Cuales son los hallazgos más comunes en imagen obtenidas en la espondilitis anquilosante?
 R = Erosiones y esclerosis sacroiliacas, IMAGEN EN COLUMNA DE BAMBU
• 44. Cual es el manejo de la EA?
 R = AINES, COX-2, Infliximab, Etanercept mas terapia física. Indometacina, INFLIXIMAB que disminuye el FNTa.
CASO CLINICO
• A 22-year-old man has symptoms of low back pain and
stiffness. After several months of mild symptoms, he notes
more severe stiffness at night and hip pain. On physical
examination, there is paravertebral muscle tenderness and
limited flexion of the lumbar spine. Figure shows an x-ray of
the lumbar spine. Which of the following is the most likely
diagnosis?
(A) Reiter syndrome
(B) Marfan syndrome
(C) ankylosing spondylitis (AS)
(D) RA
(E) pseudogout
CASO CLINICO
• A 27-year-old man has a history of low back pain and stiffness. After
several months of mild symptoms, he notes more severe stiffness at
night and hip pain. On physical examination, there is paravertebral
muscle tenderness and limited flexion of the lumbar spine. X-ray of
the lumbar spine shows sacroiliitis. In addition to recommending
physiotherapy and exercise, which of the following is the most
appropriate next step in management?
(A) NSAID therapy
(B) phenylbutazone
(C) azathioprine
(D) acetaminophen
(E) prednisone
• (A) All NSAIDs are probably equally effective
in the treatment of this man’s ankylosing
spondylitis. Options include indomethacin or
naproxen, but not phenylbutazone since it can
cause aplastic anemia. Exercise and maintaining
proper posture are very important.
CASO CLINICO
• A 27-year-old man has a history of low back pain and stiffness.
Recently, he has noticed more severe stiffness at night and hip pain.
The symptoms improve in the morning after doing some
“stretching” exercises. On physical examination, there is
paravertebral muscle and sacroiliac joint tenderness with limited
flexion of the lumbar spine. A2/6 diastolic murmur is also heard at
the left sternal border radiating to the apex. Which of the following
is the most likely diagnosis for the diastolic murmur?
(A) mitral stenosis
(B) tricuspid stenosis
(C) aortic insufficiency
(D) pulmonic insufficiency
(E) tetralogy of Fallot
(C) The frequency of aortic insufficiency has
been about 4% in ankylosing spondylitis (AS).
Other cardiac valve anomalies are not increased
in incidence. Rarely, congestive heart failure or
third degree heart block can occur as well.
(Kasper, p. 1994).
CASO CLINICO
• A23-year-old man notices new low back pain, stiffness, and left eye
discomfort. Sunlight also bothers his eyes. The back pain is worse at
night and described as a dull ache in the back and buttock area. On
physical examination, there is paravertebral muscle, iliac crest, and
ischial tuberosity tenderness with limited flexion of the lumbar
spine. His eye is inflamed and the pupil is constricted. Pelvic x-rays
show sacroiliitis. Which of the following is the most likely diagnosis
for his eye symptoms (it is the most common extra-articular
manifestation of this condition)?
(A) glaucoma
(B) acute anterior uveitis
(C) keratitis
(D) conjunctivitis
(E) episcleritis
(B) Acute anterior uveitis is the most common
extra-articular manifestation of ankylosing
spondylitis (AS). Pain, photophobia, and
increased lacrimation are the usual symptoms.
Attacks are unilateral and tend to recur, often
in the other eye. Cataracts and secondary glaucoma
are not uncommon sequelae. The iritis is
usually managed with local glucocorticoid
administration in association with a mydriatic
agent.
ENFERMEDAD DE REITER
• 45. Cuales son los datos más comunes de ARTRITIS REACTIVA o
enfermedad de Reiter?
 R = Ligado al HLA B27, oligoartritis, CONJUNTIVITIS, URETRITIS,
CERVICITIS, ULCERAS ORALES x un CUADRO INFECCIOSO PREVIO.
• 46. Que precede a una manifestación de ARTRITIS REACTIVA o
síndrome de Reiter?
 R = Alguna INFECCIÓN por SALMONELLA, SHIGELLA, YERSINIA,
CLAMYDIA TRACHOMATIS +++o
CAMPYLOBACTER que PRECEDE a la MANIFESTACIÓN ARTICULAR
3-4 semanas después.
• 47. Cual es el estudio de laboratorio o gabinete de elección del
síndrome de Reiter?
 R = CULTIVO CON ANTIBIOGRAMA, se puede encontrar también Ac
anti bacteriano en el suero o liquido sinovial o DNA bacteriano.
ARTRITIS SORIASICA
• 48. Que es la artritis soriasica?
 R = Ligado al HLA B27. Se define como una ARTRITIS usualmente
SERONEGATIVA que se ASOCIA A SORIASIS.
• 49. Cual es el cuadro clínico de la artritis soriasica?
 R = PRECEDE PSORIASIS A ARTRITIS, ARTRITIS ASIMÉTRICA con DEDOS EN
APARIENCIA DE SALCHICHA (DACTILITIS) de los dedos de manos y pies.
• 50. Como diagnosticas artritis soriasica?
 R = HC, y RX con evidencia de SACROILEITIS ASIMÉTRICA, alteraciones de la
conformación de hueso nuevo con IMAGEN DE PUNTA DE LÁPIZ EN UNA
COPA.
• 51. Cual es el manejo de la artritis soriasica?
 R = AINES E INFLIXIMAB. Tratamiento de soriasis.
CASO CLINICO
• A 19-year-old man has a chronic papulosquamous skin
disorder involving his knees and elbows. He now has dull
low back pain and morning stiffness that lasts a few hours.
On examination, forward flexion at the lumbar spine is
reduced but neck movements are normal. There is stress
tenderness of both sacroiliac joints. Which of the following
is the most likely diagnosis?
(A) RA
(B) ankylosing spondylitis (AS)
(C) psoriatic arthritis
(D) reactive arthritis
(E) PMR
(C) This patient has a sacroiliac form of arthritis
seen in patients with psoriasis. Patients with
psoriasis can develop five different patterns of
musculoskeletal symptoms. These include
arthritis of the DIP joints; asymmetric oligoarthritis;
symmetric polyarthritis similar to RA; spine
and sacroiliac type; and arthritis mutilans, a
highly destructive form. Most patients with
psoriatic arthritis also have nail involvement.
Only about a quarter actually develop a progressive,
destructive disease. Uric acid may be
elevated because of high tissue turnover but is
not part of the pathogenesis of joint disease.
ARTRITIS GONOCOCICA
• 52. Cual es el cuadro clínico de la artritis
gonococcica?
R = Poliartritis, tenosinovitis (60%) y monoartritis
purulenta en la rodilla 40%
• 53. Como diagnosticas y tratas la artritis
gonococcica?
R = Cultivo de liquido sinovial, CEFTRIAXONA iv o
LEVOFLOXACINO a los alérgicos a penicilina.
HLA B27
• Cuales patologías están ligadas al HLA B27?
1) Espondilitis anquilosante
2) Artritis reactiva o Síndrome de Reiter
3) Artritis soriasica
RESUMEN ESPONDILOARTROPATIAS
SERONEGATIVAS
CUCI Y CHRON
• 54. Cuales son los 2 tipos de artropatía en px con enfermedad de
Chron y CUCI?
 R = ESPONDILITIS ANQUILOSANTE y ARTRITIS PERIFÉRICA.
• 55. Cual es el cc de la enfermedad de CHRON?
 R = DOLOR ABDOMINAL, FIEBRE, diarrea, ULCERAS ORALES,
anorexia, hipersensibilidad de vientre al tacto, masa abdominal,
disminución de peso y SANGRE EN EVACUACIONES.
• 56. Cual es el cc de CUCI?
 R = Diarrea, HEMATOQUEZIA, DOLOR ABDOMINAL con
hipersensibilidad y LEUCOS AUMENTADOS EN LAS EVACUACIONES.
CASO CLINICO
• A 32-year-old woman has a long history of intermittent bloody
diarrhea and crampy abdominal pain. Recently, her bowel
symptoms have increased and she is having 4–6 bowel movements
a day with mucus and blood in the stool. She now has a low-grade
fever, loss of appetite, and new musculoskeletal symptoms. Which
of the following is the best description of this patient’s
accompanying arthritis?
(A) asymmetric migratory polyarthritis involving the large joints of the
arms and legs
(B) not usually associated with disease flares
(C) a progressive, crippling course
(D) symmetrical small joint involvement
(E) seropositive
(A) In inflammatory bowel disease, there are
two common types of arthritis involvement.
The first is an asymmetric, migratory polyarthritis
that affects the large joints of the
lower and upper extremities and is closely
related to the activity of bowel disease.
Spondylitis is also common (though not always
symptomatic) and is not always related to
activity of bowel disease.
ESPONDILOARTROPATIA
INDIFERENCIADA
• 57. Que caracteriza a las espondiloartropatias indiferenciadas?
 R = TODOS AQUELLOS CC QUE REÚNEN CRITERIOS DE
CLASIFICACIÓN de las espondiloartropatias PERO NOLOS
CRITERIOS ESPECÍFICOS o cc característicos de c/u de ellas.
• 58. Cuales son las manifestaciones clínicas mas comunes de las
espondiloartropatias indiferenciadas?
 R = 60-75% desarrolla datos de espondilitis alquilosante o artritis
soriasica.
• 59. Cuales son los medicamentos utilizados en las
espondiloartropatias indiferenciadas?
 R = AINES, COX-2, infliximab
FIBROMIALGIA
• 60. Como se define a la fibromialgia?
 R = Dolores difusos, presentes por > 3 MESES y que involucran los 4
CUADRANTES DEL CUERPO Y ESQUELETO AXIAL, NO HAY SUEÑO
REPARADOR y AMANECEN CANSADOS + la presencia a la PALPACIÓN
DIGITAL DE 11 PUNTOS DOLOROSOS DE LOS 18.
• 61. Que estudio auxiliar te ayuda a diagnosticar fibromialgia?
 R = No hay estudio que pueda ayudar
• 62. Cual es el tratamiento de la fibromialgia?
1) AMITRIPTILINA antes de dormir,
2) FLUOXETINA para el dolor, no hay tratamiento especifico
• 63. Cual es el pronóstico de la fibromialgia?
 R = DESFAVORABLE ya que el 25% desarrolla incapacidad importante.
SX ANTIFOSFOLIPIDO
• 65. Cuales son los criterios laboratoriales diagnósticos de SAF?
 R = Anticuerpos ANTICARDIOLIPINAS y anticoagulante LUPICO.
• 66. Cual es la etiología del SAF?
 R = Se desconoce. Los linfocitos T.
• 67. Cuales son los mecanismos por los cuales se presenta trombosis en el SAF?
 R = INHIBICIÓN de la actividad ANTICOAGULANTE, inhibición de FIBRINÓLISIS y respuesta
mediada por células aumentada
• 68. Cual es el cc característico del SAF?
 R = TROMBOSIS, livedo reticularis, ABORTOS FRECUENTES.
• 69. Que es el SAF catastrófico?
 R = Resulta de una forma acelerada que produce falla organica multiple
• 70. Cual es el manejo del SAF?
 R = PREDNISONA ELIMINA ANTICOAGULANTE. ASA 100 mg cada 24 hrs, estatinas,
HEPARINA EN EMBARAZADAS y WARFARINA DESPUÉS DEL EMBARAZO.
FIEBRE REUMATICA
• 71. En la fiebre reumática, dentro de las
manifestaciones cardiacas, que podemos encontrar?
R = NÓDULOS/cuerpos de ASCHOFF (que son una
confluencia de monocitos y macrófagos rodeados por
fibrosis) y CARDITIS (Infiltrado inflamatorio que abarca
las 3 capas del corazón). Se utilizan LOS CRITERIOS DE
JONES para su diagnostico.
• 72. Cual es el tratamiento de elección para la fiebre
reumática?
R = PENICILINA, ASA y HALOPERIDOL EN CASO DE
COREA.
LUPUS ERITEMATOSO SISTEMICO
• 73. Que HLA tienen relación con el LES?
1) HLA B8, HLA DR2 y DR3.
2) Los alelos de HLA están en el brazo corto del CROMOSOMA 6.
• 74. Que fármacos inducen la producción de anticuerpos antinucleares y LES?
 R = HIDRALAZINA, procainamida, clorpromazina, ISONIAZIDA y diversos anticonvulsivantes.
• 75. Cual es el cc del LES en cada aparato – sistema y con que padecimientos se relaciona?
 1. Se relaciona con: PTI, ANEMIA HEMOLITICA Y TIROIDITIS.
 2. Constitucionales: la FATIGA es lo mas común
 3. Mucocutaneas: 80%, LESIONES DISCOIDES observándose en cara, pabellones auriculares y el
AREA DE V DEL ESCOTE. Eritema malar o en ALAS DE MARIPOSA QUE ES FOTOSENSIBLE. EL
neonatal QUE SE PRESENTA EN LOS HIJOS DE PX CON LES POR EL PASO DE IgG
CARACTERIZÁNDOSE POR LESIONES CUTÁNEAS Y BLOQUEO CARDIACO CONGÉNITO EN
LOS PRIMEROS 6 MESES DE EDAD. ALOPECIA.
 4. Musculoesqueletico: ARTRALGIAS con artritis. Osteonecrosis en la cabeza femoral, humeral,
cóndilos femorales, carpo, tarso manifestándose clínicamente con dolor que mejora con el reposo.
 5. Renal: nefritis lupica siendo la mas común la TIPO IV ¨GLOMERULONEFRITIS PROLIFERATIVA
DIFUSA¨.
 6. SNC: Meningitis aséptica, EVC, síndromes desmielinizantes y cefaleas.
 7. Cardiopulmonar: En corazón con VEGETACIONES ENDOCARDICAS de LIBMAN-SACKS, en
pleura lo mas común es pleuritis con o sin derrame pleural con niveles altos de proteínas DHL,
complejos inmunes, ANA y células LE. LA VALVULA MAS AFECTADA ES LA MITRAL.
 8. Gastrointestinal: Se puede presentar con SEROSITIS, disfagia y dispepsia muy a menudo.
 9. Hematológico: PANCITOPENIA Y >TTP. Anemia por deficiencia de hierro MICROCITICA-
HIPOCROMICA.
• 76. Cual es el inmunosupresor de elección para la nefritis lupica y cual es el de elección en
caso de serositis.?
1) Nefritis lupica: CICLOFOSFAMIDA
2) Serositis: METOTREXATO
• 77. Cuantos criterios se revisaron en 1997 para dx de lupus?
 11 criterios entre los cuales están:
 1. Eritema malar
 2. Lupus discoide
 3. Foto sensibilidad
 4. Ulceras orales
 5. Artritis no erosiva
 6. Serositis
 7. Afección renal
 8. Afección neurológica
 9. Afección hematológica
 10. Afección inmunológica
 11. ANA +
 y se requieren de 4 criterios para el diagnostico.
• 78. Que medicamentos han mejorado la supervivencia en pacientes con nefropatía lupica?
 R = GLUCOCORTICOIDES + CICLOFOSFAMIDA
CASO CLINICO
• A22-year-old woman develops a red rash over her cheeks,
and pain and swelling in both knees as well as several small
joints in her hands. Medical evaluation reveals oral
ulceration and 3+ proteinuria. Which of the following is the
most sensitive test for the diagnosis of this condition?
(A) LE cells
(B) ANAs
(C) anti-Sm
(D) anti-Ro
(E) antiphospholipid
(B) ANAs are present in 98% of patients with
SLE. Repeatedly negative tests make the diagnosis
of SLE very unlikely. Unfortunately, the
test is not specific and may be positive in
normal people (especially in older individuals),
or secondary to infections, drugs, or other
autoimmune disorders.
CASO CLINICO
• A 25-year-old woman with SLE notices increasing fatigue and
shortness of breath on exertion. Her usual SLE symptoms of joint
discomfort, chest pain, and fevers are not present. On examination,
there are no active joints, but she is jaundiced. Laboratory studies
show hemoglobin 9 g/dL, WBC 5000/mL, platelets 150,000/mL,
reticulocyte count 4%, direct antiglobulin test: positive, total
bilirubin 4 g/dL, direct 0.5 mg/dL, aspartate amino transferase (AST)
20 U/L, alanine amino transferase (ALT) 15 U/L, LDH 300 U/L. Which
of the following is the most likely diagnosis?
(A) autoimmune hepatitis
(B) hemolytic anemia
(C) viral hepatitis
(D) acute blood loss
(E) Gilbert syndrome
(B) This patient with SLE has developed autoimmune
hemolytic anemia. Treatment consists of
high-dose oral steroids. The positive Coombs’
test (direct antiglobulin test), high indirect bilirubin,
high LDH, and high retic count are all supportive
of hemolytic anemia. Anormal AST and
ALT rule out hepatitis.
CASO CLINICO
• A30-year-old woman develops a rash over her cheeks,
nose, and ears. She also has pain and swelling in her wrists
as well as several small joints in her hands. Medical
evaluation reveals oral ulceration and 3+ proteinuria. Her
ANAis positive. Which of the following is the most likely
cardiac manifestation of her disease?
(A) pericarditis
(B) myocarditis
(C) aortic regurgitation
(D) nonbacterial endocarditis
(E) myocardial vasculitis with infarction
(A) This patient has SLE and pericarditis, sometimes
leading to tamponade, is the most common
manifestation of cardiac disease. Myocarditis
does occur and can cause arrhythmias, sudden
death, or heart failure. Libman-Sacks endocarditis
is associated with thrombotic events
or, less commonly, valvular regurgitation.
Myocardial infarction is more commonly a
result of atherosclerotic disease than vasculitis.
CASO CLINICO
• A 24-year-old is referred for assessment of a low white cell count.
She has no past medical history and is not on any medications. Her
only symptoms are of joint discomfort in her hands, and occasional
sharp chest pains that change with breathing. On examination,
there is inflammation of some MCP and DIP joints in both hands,
and the rest of the examination is normal. Her WBC is 3500/mL and
the lymphocytes are low (15%) and PMNs are normal. Which of the
following is the most likely diagnosis?
(A) periarteritis nodosa
(B) SLE
(C) scleroderma
(D) DM
(E) osteoarthritis
(B) Leukopenia occurs in almost two-thirds of
the SLE patients, and the differential count is
usually normal. Lymphocytes and platelets can
also be reduced.
CASO CLINICO
• A 26-year-old woman develops a red rash over her cheeks
and pain, and swelling in both wrists as well as several
small joints in her hands. The rash gets worse on sun
exposure and involves her cheeks, nose, ears, and chin.
Medical evaluation reveals oral ulceration and 3+
proteinuria. Which of the following is the most specific test
for diagnosis of this condition?
(A) lupus erythematosus (LE) cells
(B) antinuclear antibody (ANA)
(C) anti-Sm antibody
(D) anti-Ro antibody
(E) antiphospholipid antibody
• (C) Anti-Sm detects a protein complexed to six
species of small nuclear ribonucleic acid
(RNA). It is believed to be very specific for SLE.
However, only 30% of patients have a positive
test. In the case presented, there are enough
clinical criteria (four) to confirm the diagnosis
of SLE with 98% specificity and 97% sensitivity.
CASO CLINICO
• A young woman presents with a facial rash, arthralgias, and
fatigue. The rash on her face is erythematous and raised,
her heart and lungs are normal, and wrists are swollen and
tender on palpation. She has mild thrombocytopenia
(90,000/mL). Which of the following is the most
appropriate initial autoantibody test?
(A) anti-double-stranded (ds) deoxyribonucleic acid (DNA)
(B) anti-Sm
(C) anti-Ro or La
(D) ANA
(E) antiphospholipid antibodies (lupus anticoagulant)
• (D) ANA is the most important diagnostic
autoantibodies in patients being evaluated for
SLE. It is rare to have ANA negative SLE. The
other antibodies can occur in SLE but not as
sensitive for the diagnosis as ANA. AntidsDNA
and anti-SM are specific for SLE but
not sensitive.
CASO CLINICO
• A34-year-old woman develops a red rash over her cheeks,
frequent oral ulcers, and pain and swelling in both wrists as
well as in severalsmall joints in her hands. Medical
evaluation reveals a positive ANA, and 3+ proteinuria.
Which of the following organ involvement will cause the
most symptoms during the course of this disease?
(A) renal pathology
(B) cardiopulmonary pathology
(C) musculoskeletal pathology
(D) thrombotic events
(E) skin changes
(C) About 95% of patients will develop
musculoskeletal symptoms during the course of
SLE.
Arthralgias and myalgias predominate, but
arthritis, hand deformities, myopathy, and avascular
necrosis of bone also occur. About 85% of
patients will have hematologic disease and 80%
will have skin manifestations.
CASO CLINICO
• A27-year-old woman presents with a red rash over her
cheeks, and pain and swelling in both knees as well as
several small joints in her hands. She notes that the rash is
worse with sun exposure. Medical evaluation reveals oral
ulceration, positive ANA, and 3+ proteinuria. Which of the
following is the most likely mechanism for the renal
damage in this condition?
(A) vasculitis
(B) microemboli
(C) antibasement membrane antibodies
(D) deposition of circulating immune complexes
(E) primary tubular atrophy
(D) Renal disease is usually secondary to
deposition of circulating immune complex.
Although most patients with SLE have such
deposits, only half have clinical nephritis as
defined by proteinuria. Renal biopsy can
provide both prognostic and therapeutic
information.
ENFERMEDAD DE STILL
• 79. Que caracteriza clínicamente a la enfermedad de Still?
 R = VARIANTE DE LA ARTRITIS REUMATOIDE en la cual hay FIEBRE DE
MAS DE 40 y con precipitación posterior varios grados debajo, odinofagia y
poliadenopatias. LO PRECEDE FARINGITIS.
• 80. Cual es una manifestación característica de la enfermedad de still?
 R = EXANTEMA NO PRURIGINOSO, maculopapular de COLOR SALMÓN
en TÓRAX Y ABDOMEN.
• 81. Cual es el laboratorio distintivo de la enfermedad de Still?
 R = LEUCOCITOSIS > 40, VSG AUMENTADA, hipergamaglobulinemia IgG y anemia.
NO se detectan AUTOANTICUERPOS NI FR.
• 82. Cual es el manejo de la enfermedad de Still?
 R = AINES
CASO CLINICO
• An 18-year-old man has had fever for several weeks. The
fever occurs on an almost daily basis and is associated with
an evanescent salmon-colored truncal rash. He has diffuse
arthralgias, and an extensive investigation for infections
and malignancy is negative. Which of the following is
diagnostic of this condition?
(A) high-titer rheumatoid factor (RF)
(B) positive ANA
(C) response to steroid therapy
(D) response to nonsteroidal antiinflammatory drug (NSAID)
therapy
(E) lymph node biopsy
• (D) Still’s disease (juvenile RA) in an adult
may present as fever of unknown origin.
Unfortunately, RF is often negative, and a
response to NSAIDs along with exclusion of
other diseases confirms the diagnosis.
ESCLEROSIS SISTEMICA O
ESCLERODERMIA
• 83. Como se define la enfermedad llamada esclerosis sistémica o esclerodermia?
 R = Es una enfermedad que se caracteriza por la aparición de fibrosis difusa de la piel y órganos
internos
• 84. Que es el síndrome de CREST en esclerosis sistémica o esclerodermia?
 R = Calcinosis cutánea, fenómeno de Raynaud, trastorno de la motilidad Esofágica, eSclerodactilia
y Telangectasia
• 85. Cuales son las variantes de esclerosis sistémica o esclerodermia y sus principales afecciones?
1) LIMITADA 80%: en la que hay ENDURECIMIENTO DE LA PIEL, manos o cara.
2) DIFUSA: afecta tronco y extremidades, muñecas, tobillos, rodillas, RIÑÓN
(GLOMERULOESCLEROSIS DIFUSA) y corazón con CARDIOPATÍA RESTRICTIVA. La principal
CAUSA DE MUERTE ES PULMONAR POR HIPERTENSIÓN O FIBROSIS.
• 86. Cuales son las manifestaciones cutáneas de la esclerodermia?
 R = La piel se ve brillante , con PERDIDAS DE PLIEGUES Y ATROFIA DE FOLÍCULOS PILOSOS, se
siente ACARTONADA, la afección facial provoca una cara afilada.
• 87. Cuales son las manifestaciones vasculares de la esclerodermia?
 R = Fenómeno de Raynaud en dedos de manos, pies, nariz, oídos y lengua.
• 88. Cuales son las manifestaciones pulmonares de la esclerodermia?
 R = Enfermedad pulmonar INTERSTICIAL EN LA DIFUSA CON PATRÓN RESTRICTIVO. Hipertensión
pulmonar y cor pulmonale. En la TAC se puede ver si hay actividad (imagen en vidrio despulido) o
solo fibrosis (imagen en panal de abejas)
• 89. Cuales son las manifestaciones GI de la esclerodermia?
 R = La mayoría tiene AFECCIÓN ESOFÁGICA CARACTERIZADA POR ERGE, pirosis y dificultad para el
paso de alimentos sólidos siendo de preferencia los líquidos.
• 90. Cual es la manifestación renal de la esclerodermia?
 R = Crisis renal esclerodermica (HAS MALIGNA) que conlleva a una IR rápidamente progresiva.
• 91. Cuales son los laboratorios de esclerosis sistémica o esclerodermia?
 R = ANA +, depende del órgano afectado, AC VS ESCLERODERMIA SCL-70. En la BIOPSIA aparece
EXCESO DE DEPOSITO DE COLÁGENO EN PIEL, TCS Y VASOS SANGUÍNEOS que puede aparecer de
manera lineal (esclerosis lineal) o en parches hipocromicos. DIFUSA: ANA y TOPOISOMERASA I, SX
CREST: AC ANTICENTROMERO.
• 92. Cual es el manejo de la esclerosis sistémica o esclerodermia?
 R = D- PENICILAMINA. Para el FENÓMENO DE RAYNAUD se da NIFEDIPINO o losartan.
Iloprost que es análogo de la prostaciclina para ulceras digitales. CICLOFOSFAMIDA, corticoides,
SINDENAFIL para ENFERMEDAD INTERSTICIAL PULMONAR. PIEL (morfea del tronco hipocromica)
luz UV, CORTICOIDES TÓPICOS, si se generaliza se da metotrexate o CORTICOIDES SISTÉMICOS.
CALCINOSIS diltiazem o NIFEDIPINO. DAÑO RENAL IECAS. TELANGIECTASIAS ESCLEROSIS. GI
PROCINETICOS Y ERITROMICINA por que ESTIMULA LA MOTILINA, NO CISAPRIDA (QUE PUEDE
PRODUCIR SX QT LARGO, TAQUICARDIA VENTRICULAR Y DESENLACES FATALES). ARTRALGIAS
acetaminofen, PERICARDITIS AINES, corticoides o CICLOFOSFAMIDA.
CASO CLINICO
• A39-year-old woman complains of developing painful pale fingers
on cold exposure for the past 5 years. Recently, she has noticed
swollen fingers and tight skin, which limit flexion and extension. She
also has new abdominal symptoms that are bothersome. On
examination, the skin on the fingers is smooth and shiny with
associated edema. The rest of the examination is normal. Which
part of the gastrointestinal (GI) tract is most frequently involved in
this condition?
(A) esophagus
(B) stomach
(C) duodenum
(D) ileum
(E) colon
• (A) This patient has scleroderma and
esophageal symptoms are present in more than
50% of patients. They are due to the reduced
tone of the gastroesophageal sphincter and
dilation of the distal esophagus. Gastric and
small intestinal motility problems can also
occur. Vascular ectasia in the GI tract can result
in bleeding.
CASO CLINICO
• A22-year-old woman develops color change in her fingers
with cold exposure. The fingers turn white, then blue, and
finally red. Which of the following statements regarding this
condition is incorrect?
(A) may lead to gangrene of the fingers
(B) may precede the onset of scleroderma
(C) symptoms can be brought on by vibration or stress
(D) pallor (white color) associated with coldness and
numbness while rubor (red color) associated with pain and
tingling
(E) affects the sexes equally
(E) Raynaud’s phenomenon may lead to gangrene
of the fingers. It can be primary
(Raynaud’s disease) or secondary to other diseases,
especially scleroderma, in which it can be
the presenting symptom. In women, the primary
form is common (over 50%), and the phenomenon
is generally much more frequent in
women. Digital infarction is much more
common in relationship to scleroderma than it
is in primary Raynaud’s disease.
ENFERMEDAD MIXTA DEL TEJIDO
CONECTIVO
MIOSITIS Y DERMATOMIOSITIS
• 93. Que tipo de padecimientos son la miositis y dermatomiositis?
 R = Etiologia desconocida y se caracterizan por DEBILIDAD MUSCULAR PROXIMAL
GRADUAL Y PROGRESIVA.
• 94. Que células predominan en la biopsia con DERMATOMIOSITIS?
 R = Infiltrado de CÉLULAS B Y T CD4
• 95. Que células predominan en la biopsia con POLIMIOSITIS?
 R = Predominan las células T CD8 A NIVEL ENDOMICIAL.
• 96. Cuales son los datos clínicos de la miositis y dermatomiositis?
1) Dificultad para la deglución por afección de musculo estriado, EXANTEMA COLOR
ROJO OBSCURO CARACTERÍSTICO CON DISTRIBUCIÓN MALAR PARECIDO A LES
QUE PUEDE ESTAR EN CARA, CUELLO Y ESPALDA.
2) EDEMA PERIORBITARIO AMORATADO (heliotropo) SOBRE LOS PARPADOS.
3) Fiebre, DISMINUCIÓN DE LA FUERZA PROXIMAL y V del cuello.
• 97. El SIGNO DE GOTTRON es patognomónico de miositis y
dermatomiositis, que lo caracteriza?
 R = ERITEMA PERIUNGUEAL, PLACAS ERITEMATO-ESCAMOSAS SOBRE
DORSO DE LAS ARTICULACIONES INTERFALANGICAS Y
METACARPOFALANGICAS.
• 98. Según los criterios de Bohan y Peter como diferencias la
dermatomiositis de la miositis-polimiositis?
 R = 1) DERMATOMIOSITIS: 3 de 4 criterios MAS ERITEMA. 2) MIOSITIS: 4
criterios SIN ERITEMA.
• 99. Cuales son los datos de laboratorio de miositis y dermatomiositis?
 R = DHL Y CPK ELEVADA, AC ANTI RO-LA, AC ANTI JO y BIOPSIA
MUSCULAR CON INFILTRADO LINFOCITARIO
• 100. Cual es el manejo de la polimiositis y dermatomiositis?
 R = PREDNISONA, la cual puede o no combinarse con inmunosupresores.
CASO CLINICO
• A 64-year-old woman presents with fatigue, and musculoskeletal
symptoms. She reports of no headache, jaw discomfort, or visual
disturbance. Her physical examination is normal, and laboratory
testing reveals an elevated ESR of 75 mm/h. She is started on
prednisone 10 mg/day and notices a dramatic improvement in her
symptoms after 1 week. Which of the following are the most typical
symptoms of this disorder?
(A) heliotrope rash
(B) proximal muscle weakness
(C) painful peripheral neuropathies
(D) stiffness and pain of proximal muscles
(E) hematuria
(D) This patient has PMR. It is characterized by
stiffness, aching, and pain in proximal muscle
groups in the neck, shoulders, back, hips, and
thighs. It is considerably more common than
temporal arteritis. Both diseases are almost
exclusively seen in the over 50 age group.
CASO CLINICO
• A 64-year-old man is having difficulty getting out of a chair.
He has lost 15 lb and feels tired all the time. On
examination, there is a blue purple rash on his eyelids and
knuckles, and muscle strength in his proximal muscles is
rated 4 out of 5. His creatinine kinase (CK) level is elevated
and he is started on prednisone. Which of the following is
the most important in monitoring response to therapy?
(A) testing of muscle strength
(B) sedimentation rates
(C) urine transaminase enzymes
(D) EMG
(E) alkaline phosphatase
(A) The course of muscle necrosis in dermatomyositis
can be best followed by repeated CK
determinations. Repeated muscle biopsies are
rarely required. However, the goal of therapy is
to increase muscle strength and function, so following
muscle strength is the key clinical assessment
of response to therapy.
CASO CLINICO
• A 63-year-old man presents with weakness and hemoptysis, but no
fever, cough, or sputum. He has a 60-pack-per-year history of
smoking. The chest x-ray (CXR) reveals a lung mass with mediastinal
widening. On examination, there is a blue purple discoloration of
the upper eyelids and erythema on his knuckles. He has proximal
muscle weakness rated 4+/5, normal reflexes, and sensation. Which
of the following is the most likely diagnosis for his muscle
weakness?
(A) SLE
(B) scleroderma
(C) dermatomyositis (DM)
(D) polyarteritis
(E) Weber-Christian disease
(C) This man has dermatomyositis, a paraneoplastic
phenomenon of many cancers. The most
common tumors associated with DM have been
bronchogenic carcinomas, ovarian cancers, breast
cancers, and melanoma but many others have
occurred. The malignancy may antedate or postdate
the myositis. Older age makes malignancy
more likely. The extent of the workup for malignancy,
if DM is the presentation, depends on
clinical circumstances, but history and physical
examination, not x-rays, are the cornerstones of
evaluation.
SX SJOGREN
• 101. Cual es el cc del síndrome de Sjogren?
 R = XEROSTOMIA, XEROFTALMIA Y ATROFIA PAPILAR LINGUAL. Los
síntomas aumentan con la aplicación de ANTICOLINERGICOS,
ANTIDEPRESIVOS Y DIURÉTICOS.
• 102. Cuales son los datos de laboratorio de laboratorio sugerentes de Sx
de Sjogren?
 R = ANA 95% y ANTI RO-LA, SSA Y SSB. Prueba de SCHRIMER MENOR A 5
MM cuando lo normal es mayor a 10 después de 5 min.
• 103. Cual es el manejo del Sx de Sjogren?
1) SALIVA Y LAGRIMAS ARTIFICIALES.
2) PILOCARPINA.
CASO CLINICO
• A 32-year-old woman develops symptoms secondary to a
dry mouth and dry eyes. She has enlarged salivary glands.
Studies forautoantibodies to Ro (SS-A) are positive.
Asalivary gland biopsy reveals lymphocytic infiltration.
Which of the following is the most likely diagnosis?
(A) sarcoidosis
(B) primary Sjögren’s syndrome
(C) human immunodeficiency virus (HIV) infection
(D) lymphoma
(E) amyloidosis
• (B) The sicca syndrome is a recognized feature of Sjögren’s
syndrome. It can be primary or secondary to other
autoimmune disorders such as RA, SLE, scleroderma, or
vasculitis. Primary Sjögren’s syndrome is most common in
middle-aged women; sicca symptoms can also occur as a
complication of HIV infection or in sarcoidosis. Sjögren’s
syndrome is more likely to have positive serology, while the
serology in sarcoid or HIV is negative. Both HIV and Sjögren
can have lymphocytic infiltration, but in HIV it is
predominantly by CD8+ lymphocytes, whereas in Sjögren’s
syndrome, the infiltration is by CD4+ lymphocytes. In
sarcoidosis, biopsy reveals granulomas.
RABDOMIOLISIS
• 104. En que consiste la rabdomiolisis?
R = Consiste en la NECROSIS de musculo esquelético,
relacionado comúnmente a traumatismo.
• 105. Por que se produce IRA en rabdomiolisis?
R = Daño tubular por filtración de MIOGLOBINA
• 106. Cual es el manejo de la rabdomiolisis?
R = AUMENTO DE LIQUIDOS de 4-6 litros y vigilar
función renal.
POLIARTERITIS NODOSA
• 107. Cuales son los signos y síntomas de poliarteritis nodosa?
 R = DOLOR EN EXTREMIDAD POR ARTRALGIA, MIALGIA QUE AFECTA
PANTORRILLAS, puede afectar riñón causando HAS, IRA O HEMORRAGIA
POR MICROANEURISMAS. MONONEURITIS MÚLTIPLE (PIE CAÍDO),
corazón con IAM. Livides reticular, nódulos subcutáneos, y ulceras en piel. Son
necesarios 3 de 10 criterios para su diagnostico.
• 108. Cuales son las pruebas de laboratorio en la poliarteritis nodosa y con que
otro virus se relaciona habitualmente?
 R = DESCARTAR VHB QUE SE ASOCIA EN 20-30%. ANCA (-) Y ANA (-) .Anemia,
leucocitosis y trombocitosis. Se confirma diagnostico con toma de BIOPSIA DE LA
LESIÓN.
• 109. Que datos se obtienen al tomar la biopsia en la poliarteritis nodosa?
 R = BIOPSIA DE ARTERIA de mediano o pequeño calibre con PRESENCIA DE
INFILTRADO GRANULOCITICO, PMN Y CÉLULAS MONONUCLEARES EN LA PARED DEL
VASO.
• 110. Cual es el manejo de la poliarteritis nodosa?
 R = ESTEROIDES, pero SI SE ASOCIA CON VHB SE AGREGAN RETROVIRALES.
CASO CLINICO
• A 39-year-old man has had several weeks of fever, abdominal pain,
weight loss, and lack of energy. Three days prior to assessment, he
developed a left foot drop. Physical examination confirms left
peroneal nerve damage and a bilateral sensory peripheral
neuropathy in both legs. Laboratory evaluation reveals ESR of 105
mm/h, WBC of 14,000/mL, and a negative serologic test for ANCA.
The eosinophil count is normal. Which of the following is a
reasonable method of establishing a diagnosis?
(A) testicular biopsy
(B) skin biopsy
(C) spiral computerized tomography (CT) of chest
(D) further serologic testing
(E) abdominal angiography
(E) This patient likely has PAN. ANCA and
other serology are usually negative in PAN;
positive serology suggests another diagnosis.
The optimal diagnostic strategy is the biopsy of an
affected organ. However angiography to
look for aneurysms of small- and medium sized
arteries generally has higher yield than
blind biopsy of unaffected organs. The lungs
are not a characteristic site of involvement.
CASO CLINICO
• A 45-year-old man has had several weeks of fever, abdominal pain, weight
loss, and lack of energy. Three days prior to assessment, he developed a
left foot drop. His blood pressure is 160/90 mm Hg, pulse 80/min, and
physical examination confirms left peroneal nerve damage and a bilateral
sensory peripheral neuropathy in both legs. There are no skin rashes.
Laboratory evaluation reveals ESR of 105 mm/h, WBC of 14,000/mL, and
negative serologic tests for ANCA and ANA. Eosinophil count is normal.
Which of the following chronic viral infections is sometimes associated
with this?
(A) high cytomegalovirus (CMV) titers
(B) herpesvirus material in circulating immune complexes
(C) hepatitis B material in circulating immune complexes
(D) epidemiologic relationship to Coxsackie B virus
(E) triggering of symptoms following viral gastroenteritis secondary to
rotavirus infection
(C) About 20–30% of patients with PAN have
hepatitis B antigenemia. Circulating immune
complexes containing hepatitis B antigen and
immunoglobulin have been detected, and
immunofluorescence of blood vessel walls
have also demonstrated hepatitis B antigen.
Antiviral therapy has been used in these cases.
CASO CLINICO
• A 42-year-old woman has had several weeks of fever, abdominal pain, weight loss,
and lack of energy. Three days prior to assessment, she developed a left foot drop
and rash on her legs. Her blood pressure is 160/90 mm Hg, pulse 80/min, and
physical examination confirms left peroneal nerve damage and a bilateral sensory
peripheral neuropathy in both legs. The rash looks like livedo reticularis.
Laboratory evaluation reveals ESR of 105 mm/h, WBC of 14,000/mL, and negative
serologic tests for ANCA and ANA. Eosinophil count is normal, and urinalysis is
negative for casts, protein, and red cells. Biopsy of the skin rash shows
inflammation of the small blood vessels. Which of the following is the most
appropriate next step in management?
(A) plasmapheresis
(B) steroid therapy alone
(C) combination therapy with steroids and cyclophosphamide
(D) cyclophosphamide therapy alone
(E) combination therapy with steroids and methotrexate
(C) Current treatment for polyarteritis nodosa
(PAN) mimics that of Wegener’s granulomatosis
in the initial treatment with combination
steroid and cyclophosphamide therapy. This
will result in up to a 90% long-term remission
rate even after discontinuation of therapy. In
cases associated with hepatitis B infection,
plasmapheresis is sometimes used as initial
therapy.
CASO CLINICO
• A 55-year-old woman has had several weeks of fever, abdominal pain,
weight loss, and lack of energy. Three days prior to the assessment, she
developed a left foot drop. Her blood pressure is 160/90 mm Hg, pulse
80/min, and physical examination confirms left peroneal nerve damage
and a bilateral sensory peripheral neuropathy in both legs. There are no
skin rashes. Laboratory evaluation reveals ESR of 105 mm/h, WBC of
14,000/mL, and negative serologic tests for antineutrophil cytoplasmic
antibody (ANCA) and ANA. The eosinophil count is normal, and urinalysis
is negative for casts, protein, and red cells. Which of the following is the
most likely mechanism for renal injury in this condition?
(A) nephrotic syndrome
(B) diffuse glomerulonephritis
(C) granuloma formation
(D) necrotizing vasculitis of vessels
(E) exclusively small vessel involvement
(D) This patient has polyarteritis nodosa (PAN)
and in classic PAN, unlike microscopic
polyangiitis, both small and medium vessels
are involved. The renal lesions are ischemic
secondary to fibrinoid necrosis of the vessels. In
microscopic polyangiitis, a diffuse glomerulonephritis
is frequently present. The most
common organ systems involved are the kidneys,
musculoskeletal system, and peripheral
nervous system.
POLIMIALGIA REUMATICA/ ARTERITIS
DE CELULAS GIGANTES
• 111. Cual es la diferencia entre polimialgia reumática y arteritis de células
gigantes?
 R = LA POLIMIALGIA REUMÁTICA SOLA NO PRODUCE CEGUERA y responde a la
terapéutico con prednisona y la ARTERITIS DE CÉLULAS GIGANTES OCASIONA
CEGUERA y necesita mayores dosis de prednisona.
• 112. Cuales son los datos clínicos de polimialgia reumática?
 R = DOLOR Y RIGIDEZ DE HOMBRO y región de la cintura acompañado de fiebre y
disminución de peso.
• 113. Cuales son los datos clínicos de arteritis de células gigantes?
 R = Cefalea, SENSIBILIDAD DE CUERO CABELLUDO, síntomas visuales,
CLAUDICACIÓN MANDIBULAR.
• 114. Cuales son los datos de la biopsia y USG en ACG de la arteria temporal?
 R = INFILTRADO en túnica ½ y adventicia con linfocitos y células plasmáticas. USG
CON SIGNO DEL HALO con sensibilidad 62% y especificidad 82%
• 115. Que dato se encuentra en la biopsia de la arteria temporal
en ACG?
 R = Fragmentación de la lamina elástica e infiltrado por linfocitos y
macrófagos, células gigantes en la lamina elástica interna.
• 116. Cual es el manejo de la polimialgia reumática?
 R = PREDNISONA POR 1ª
• 117. Cual es el manejo de la ACG?
 R = PREDNISONA Y PREVENCIÓN DE CEGUERA
PERMANENTE
CASO CLINICO
• A 74-year-old man presents with a history of increasing frequency
of headaches, fatigue, and weight loss for 3 months. He has had
migraine headaches in the past, but these are differentfrom them.
He is also experiencing back, shoulder, and hip discomfort, which is
worse in the morning. His head and neck examination is normal.
Range of motion in the shoulders and hips is reduced because of
discomfort but there is no active inflammation. Which of the
following signs or symptoms is most helpful in the diagnosis?
(A) throat pain on swallowing
(B) pain in the jaw when chewing
(C) malaise
(D) fatigue
(E) sweating
(B) Although malaise, fatigue, and sweating
are common in temporal arteritis, they are too
nonspecific to help in making the diagnosis.
Claudication of the jaw and tongue, while not
very sensitive for temporal arteritis, are more
specific than the constitutional symptoms.
Odynophagia is not a characteristic of this
disease.
CASO CLINICO
• A 69-year-old man comes to the office complaining of fatigue, and weight loss for
3 months. He also reports having frequent headaches, which are new for him.
There are no other constitutional symptoms of fever, chills or night sweats. He
does have chronic lower back pain but lately he has noticed pain in his shoulder,
and hip as well. The muscle and joint symptoms are worse in the morning, and the
stiffness lasts for 1 hour. His jaw also hurts when he is chewing food. Head and
neck examination is normal, there is no lymphadenopathy and fundoscopy is
normal. Range of motion in theshoulders and hips is reduced because of
discomfort but there is no active inflammation. Which of the following is the most
feared complication in patients with this condition?
(A) blindness
(B) cortical stroke
(C) limb claudication
(D) renal infarction
(E) aortic aneurysm
(A) Although all these complications have been
reported in giant cell arteritis, the only one with
a significant likelihood is blindness secondary
to ischemic optic neuropathy. Thus, if the disease
is suspected, urgent diagnosis and treatment
is required.
CASO CLINICO
• A 67-year-old man complains of frequent headaches that are new for him.
They are usually not very severe, and relieved with acetaminophen. He
also has some back, shoulder, and hip discomfort, which is worse in
themorning as well he feels quite fatigued and does not have his usual
energy level. On examination, his neck is supple to flexion, fundi and
thyroid examination are normal. Range of motion in the shoulders and
hips is reduced because of discomfort but there is no active inflammation.
There are no focal deficits on screening neurologic examination. Which of
the following is the most appropriate initial diagnostic test?
(A) immunoelectrophoresis
(B) c-ANCA levels
(C) ESR
(D) creatine phosphokinase (CPK)
(E) hemoglobin and red cell indices
(C) Almost all patients with temporal arteritis
will have an elevated ESR. Although a high ESR
cannot make the diagnosis, a normal ESR
helps in excluding the diagnosis. C-ANCA is a
diagnostic tool for Wegener’s granulomatosis.
Elevated CPK is not seen in temporal arteritis,
even with associated PMR. Normochromic, or
slightly hypochromic, anemia often seen in
temporal arteritis is too nonspecific to be of
much diagnostic help.
CASO CLINICO
• A 57-year-old woman is complaining of frequent headaches and
scalp tenderness. She also has arthralgias, fatigue, and discomfort
in her jaw when she chews. On examination, her head and neck is
normal, but the right temporal artery is tender on palpation. Her
erythrocyte sedimentation rate (ESR) is 50 mm/h and hemoglobin
10.5 g/dL. Atemporal artery biopsy is obtained for diagnostic
confirmation. Which of the following is the most appropriate next
step in management?
(A) intravenous high-dose steroids
(B) acetylsalicylic acid
(C) indomethacin
(D) low-dose (prednisone 40 mg/day) steroids by mouth
(E) topical steroid creams
• (D) The response of pain, stiffness, and
headaches to 40–60 mg of prednisone is
dramatic in giant cell arteritis. The duration of
treatment is not known but most patients
require treatment for more than 2 years. ESR
is used to monitor response to therapy.
Patients need treatment and evaluation for
the complications of long-term steroid use
such as osteoporosis and diabetes.
CASO CLINICO
• A 75-year-old woman has abrupt onset of soreness,
and severe stiffness of the shoulders and upper thighs
with low grade fever. Physical examination is entirely
normal, but ESR is over 100 mm/h. Which of the
following is the most likely diagnosis?
(A) dermatomyositis (DM)
(B) osteoarthritis
(C) polymyalgia rheumatica (PMR)
(D) midline granuloma
(E) sarcoidosis
• (C) This patient has PMR. Proximal arm and hip
muscle/joint discomfort is the hallmark of this
disorder. Difficulty in getting out of bed or rising
from a chair may suggest polymyositis, but the
muscles are normal when muscle strength is
assessed. In general, PMR causes painful muscles,
not weak muscles. However, pain may lead
to profound disuse atrophy and apparent muscle
weakness. In these cases, normal CK and nonspecific
muscle biopsy still allow accurate differentiation
from polymyositis.
CASO CLINICO
• A71-year-old woman comes to the office with a history of headaches,
fatigue, and weight loss for 3 months. The headaches are new for her, and
usually not very severe. Her jaw also hurts when she is chewing food. Two
days prior, she had briefly lost partial vision in her left eye. There were no
other neurologic symptoms at the time. On examination, her neck is
supple to flexion, fundi and neurologic examinations are normal. She is
started on prednisone 60 mg/day and a biopsy is performed to confirm
the diagnosis. Which of the following is the most likely change seen on the
biopsy to confirm the diagnosis?
(A) immune complex deposition
(B) arteritis with giant cells
(C) lymphocytic infiltration
(D) type II muscle fiber atrophy
(E) polyphasic potentials on electromyography (EMG)
(B) Temporal artery biopsy is required for
definitive diagnosis of giant cell arteritis,
because of the relatively nonspecific nature of
the presenting symptoms, signs, and routine
laboratory tests. The arteritis can be segmental,
however, and great care must be taken in the
pathologic assessment.
GRANULOMATOSIS DE WEGENER
• 118. Cuales son las vasculitis asociadas a ANCA?
 R = Granulomatosis de WEGENER, SX DE CHURG-STRAUSS y POLIANGEITIS
MICROSCÓPICA.
• 119. En que consiste la granulomatosis de Wegener?
 R = VASCULITIS de arterias pequeñas, LESIONES GRANULOMATOSAS NECROSANTES de VÍAS
AÉREAS SUPERIORES E INFERIORES Y GLOMERULONEFRITIS. Se sospecha en personas con
AFECCIONES RESPIRATORIAS RECURRENTES.
• 120. Cuales son los datos clínicos de la granulomatosis de Wegener?
 R = RINITIS crónica o SINUSITIS CRÓNICA. PULMONAR: 96% con ESPUTO
SANGUINOLENTO, dolor torácico y disnea. SÍNTOMAS DE VÍAS AÉREAS SUPERIORES E
INFERIORES CON REBELDÍA AL TRATAMIENTO 90%. Afección RENAL: con
GLOMERULOESCLEROSIS FOCAL Y SEGMENTARIA que puede evolucionar a IR rápidamente
progresiva.
• 121. Cuales son los datos de gabinete de la granulomatosis de Wegener?
 R = ANCAc. ANCAp .Biopsia renal con GLOMERULONEFRITIS FOCAL Y SEGMENTARIA
NECROSANTE. TAC de tórax con infiltrados, nódulos, granulomas y calcificaciones.
Histologicamente vasculitis, inflamación granulomatosa, necrosis.
• 122. Cual es el manejo de granulomatosis de Wegener?
 R = CICLOFOSFAMIDA, prednisona, metotrexate.
CASO CLINICO
A 28-year-old woman presents with a recent
episode of coughing up some blood, frequent
nosebleeds, and now decreased urine output.
A nasal mucosa ulcer was seen on inspection.
Her urinalysis is positive for protein and red
cells consistent with a GN. The CXR shows
two cavitary lesions and her serology is positive
for antineutrophil cytoplasmic antibodies
(ANCA). Which of the following is the most
likely diagnosis?
(A) Wegener’s granulomatosis
(B) bacterial endocarditis
(C) Goodpasture’s syndrome
(D) lupus erythematosus
(E) poststreptococcal disease
(A) Numerous diseases are associated with
renal and pulmonary manifestations, including
lupus, Goodpasture’s syndrome, and
Wegener’s granulomatosis. Wegener’s is typically
associated with antineutrophil cytoplasmic
antibodies. (Kasper, p. 2005)
SX DE CHURG STRAUSS
• 123. Como se define el síndrome de Churg-Strauss?
 R = Como una VASCULITIS necrosante sistémica y tiene una gran ASOCIACIÓN
CON ASMA 98-100%
• 124. Cual es el cc del síndrome de Churg-Strauss?
 FASE I: ASMA o RINITIS alérgica con meses o años de duración.
 FASE II: INFILTRACIÓN TISULAR EOSINOFILICA con cuenta arriba de 1000 por ml.
 FASE III: Vasculitis sistémica que afecta corazón, pulmones, piel, nervios periféricos
• 125. Cuales son los datos de gabinete y biopsia en el síndrome de Churg-Strauss?
 R = ANCAp Y EOSINOFILIA. Biopsia con GRANULOMA EOSINOFILO
extravascular con zona de necrosis central rodeada de infiltrado celular mixto.
• 126. Cual es el manejo del síndrome de Churg-Strauss?
 R = El tratamiento para el síndrome de Churg-Strauss incluye GLUCOCORTICOIDES
como la prednisolona y otras drogas inmunosupresoras como la azatioprina y
ciclofosfamida. La enfermedad es crónica y de por vida.
POLIANGEITIS MICROSCOPICA
• 127. Cuales son los datos clínicos de la poliangeitis microscópica?
 R = RIÑÓN con GLOMERULONEFRITIS de rápida progresión y PULMÓN con
HEMORRAGIA difusa.
• 128. Que se observa en la biopsia renal en un paciente con poliangeitis microscópica?
 R = Demuestra GLOMERULONEFRITIS NECROSANTE FOCAL Y SEGMENTARIA
pauciinmune, con PROLIFERACIÓN CAPILAR EN FORMA DE MEDIAS LUNAS.
• 129. Que datos radiológicos y sintomáticos tiene un paciente a nivel pulmonar en un
paciente con poliangeitis microscópica?
 R = La HEMORRAGIA PULMONAR se documenta al observar un infiltrado o sombras
alveolares en un paciente con HEMOPTISIS, disnea (hipoxemia) y anemia en ausencia de
edema pulmonar o infección.
• 130. Cual es el diagnostico diferencial en un paciente con poliangeitis microscópica?
 R = SX DE GOODPASTURE que también presenta un síndrome pulmón-riñón pero NO
PRESENTA VASCULITIS NI AFECCIÓN MULTISISTEMICA.
• 131 Cual es el manejo de la poliangeitis microscópica?
 R = GLUCOCORTICOIDES Y CICLOFOSFAMIDA. Sin tratamiento la mortalidad a 5 años es del
85%.
PURPURA DE HENOCH SCHONLEIN
• 132. En que consiste la purpura de Henoch Schonlein?
 R = Vasculitis sistémica de pequeño calibre mas común en niños
• 133. Cuales son los rasgos típicos de la purpura de Henoch Schonlein?
1) ANTECEDENTE DE ENFERMEDAD VIRAL.
2) TRIADA CLÁSICA DE PURPURA, DOLOR ABDOMINAL TIPO
CÓLICO Y ARTRITIS.
3) Purpura palpable NO TROMBOPENICA en región glútea y miembros
pelvicos, dolor abdominal, artritis, hematuria.
4) Lesiones en la piel de forma clásica en extremidades inferiores pero
pueden manifestarse en cualquier zona.
5) GI: Dolor tipo cólico, nausea y vomito.
6) RENAL: Glomerulonefritis o síndrome nefrótico.
• 134. Cuales son lo hallazgos patológicos en la biopsia y gabinete de la
purpura de Henoch Schonlein?
 R = Depósitos de IgA EN BIOPSIA y análisis sanguíneo SIN TROMBOPENIA
ni alteración de la coagulación.
SÍNDROME DE BECHET
• 135. Que caracteriza al síndrome de Bechet?
 1) ULCERA ORAL, ENCIA Y LABIOS: Ulcera dolorosa redonda (aftosa), borde
eritematoso y cubierta por pseudomembrana amarillento.
 2) ULCERAS GENITALES: Pene, escroto o vulva similares a las orales.
 3) OFTALMICAS: Hipopion, epiescleritis, vasculitis retineana e UVEÍTIS con
dolor, ojo rojo y visión borrosa.
• 136. Que pruebas diagnostica existe para el diagnostico de síndrome de
Bechet?
 R = PATERGIA + que consiste en puncionar con una aguja estéril el
antebrazo del paciente y si aparece un nódulo eritematoso aséptico o una
pústula mayor a 2 mm es positivo.
• 137. Cual es el manejo del síndrome de Bechet?
1) GLUCOCORTICOIDES.
2) TALIDOMIDApara ULCERAS ORAL Y VAGINAL
ARTERITIS DE TAKAYASU
• 138. Que tipo de alteración es la arteritis de Takayasu?
 R = Es una vasculitis que afecta a la aorta y sus ramas así como a las arterias pulmonares
produciendo síntomas isquémicos principalmente.
• 139. Cuales son los datos clínicos de la arteritis de Takayasu?
1) SISTEMICOS: FIEBRE, pérdida de peso, astenia, artralgias.
2) Manifestaciones isquémicas como frialdad de una o mas extremidades, cefalea, amaurosis,
diplopía y la PRESIÓN EN AMBOS BRAZOS DIFIERE 10 MM/HG.
• 140. Cuales son los datos de laboratorio y gabinete principales en la arteritis de Takayasu?
1) LABORATORIO: VSG AUMENTADA, trombocitosis, ANEMIA NORMO-NORMO.
2) ANGIOGRAFIA: estrechamientos con adelgazamientos progresivos, oclusiones y
aneurismas.
• 141. Cual es el manejo de la arteritis de Takayasu?
1) PREDNISONA
2) ANGIOPLASTIA
CASO CLINICO
• A 24-year-old woman notices pain in her left arm, made worse with
use. She also has fatigue, fever, night sweats, and arthralgias. On
examination, there are no palpable lymph nodes, and the joints and
muscle strength are normal. The left radial pulse is absent, and
there is a bruit over the left subclavian and common carotid
arteries. Preliminary laboratory investigations reveal an elevated
ESR and mild anemia. Which of the following vascular findings is
most likely to be found in her?
(A) high pressure in the legs and low pressure in the arms
(B) low pressure in the legs and high pressure in the arms
(C) high-pitched diastolic murmur
(D) a relentless course to death
(E) hypotension
(A) High pressure in the legs and low pressure
in the arms characterize Takayasu’s syndrome.
Clinical manifestations include easy fatigability
of the arms and atrophy of the soft tissues of the
face. The course is variable, and spontaneous
remissions can occur. The disease predominantly
affects young women.
ENFERMEDAD DE KAWASAKY
• 142. Cual es el cc de la enfermedad de Kawasaky?
R = Enfermedad FEBRIL generalmente EN <5ª y se
caracteriza por eritema en mucosa oral, EROSIONES
EN LOS LABIOS, LENGUA EN FRESA, vasculitis
coronaria.
• 143. Cual es el manejo de la enfermedad de
Kawasaky?
R = GAMAGLOBULINA IV + ASA por 14 días.
OSTEOMIELITIS TB O MAL DE POTT
• 145. En un paciente con ANEMIA DE CÉLULAS FALCIFORMES, que agente causal es
mas común de provocar osteomielitis?
 R = Salmonella
• 146. Cual es el tratamiento de elección para todas las lesiones óseas causadas por
coccidioides inmitis?
 R = ITRACONAZOL
• 147. Cual es el cc de la osteomielitis tuberculosa?
 R = DOLOR DORSAL Y LUMBAR con dificultad para sentarse, ponerse de pie o
flexionar el cuerpo, con dolor a la palpación y contracción muscular protectora, se
aprecia malestar general crónico y manifestaciones pulmonares de tb.
• 148. Cuales son los hallazgos radiológicos de la osteomielitis tuberculosa o mal de
Pott?
 R = LESIONES OSTEOLITICAS EN LA COLUMNA, ASÍ COMO PERDIDA DE LA ALTURA
DEL ESPACIO INTERVERTEBRAL.
• 149. Cual es la complicación mas grave de la osteomielitis tb en la
columna vertebral (espondilitis tuberculosa o enfermedad de
Pott)?
 R = Paraplejia
• 150. Cual es el régimen de tx en la osteomielitis tuberculosa?
 R = I, R Y P durante 6 MESES
• 151. Cual es el cc de la artritis tuberculosa?
• R = La artritis periférica se manifiesta como MONOARTRITIS EN MAS
DEL 90% de los casos, siendo las ARTICULACIONES GRANDES COMO
LA RODILLA O LA CADERA LAS MAS AFECTADAS. El dolor o limitación
funcional son síntomas tardíos.
• 152. Cual es la triada clásica de Phemister radiológico en la
osteomelitis tuberculosa o mal de Pott?
• R = OSTEOPENIA PERIARTICULAR, EROSIONES ARTICULARES
PERIFÉRICAS Y DISMINUCIÓN GRADUAL DEL ESPACIO ARTICULAR. El
diagnostico definitivo se realiza mediante la biopsia de tejido sinovial,
con cultivos y tinciones demostrando el bacilo.
OSTEOSARCOMA
• 153. Cuales son los tumores benignos de los huesos y cual es su tratamiento de elección?
 R = OSTEOMAS OSTEOIDES y el tratamiento de elección es la FOTOCOAGULACIÓN CON LASER
• 154. Que es una atropatia neurogena o articulación de Charcot?
 R = Consiste en la DESTRUCCIÓN ARTICULAR resultante de la PERDIDA o disminución en la
PROPIOSEPCION DEL DOLOR Y TEMPERATURA. LA INYECCIÓN INTRARTICULAR DE
HIDROCORTISONA PUEDE OCASIONARLO
• 155. Que lugar ocupa en frecuencia el osteosarcoma?
 R = 2do lugar después del mieloma y es maligno, ataca principalmente a niños, adolescentes y
adultos jóvenes CRECIENDO GENERALMENTE EN ZONAS DE DESARROLLO EPIFISIARIO.
• 156. Cual es el cc del osteosarcoma?
 R = Es una neoplasia que crece con rapidez y altamente destructiva metastatiza al pulmón
rápidamente. El síntoma principal es el DOLOR con afección articular, AUMENTO DE LA
TEMPERATURA LOCAL.
• 157. Cuales son los hallazgos de gabinete en el osteosarcoma?
 R = La FA esta aumentada. La RM es de elección. En la radiografía simple presenta una IMAGEN EN
¨SOL NACIENTE¨.
• 158. Cual es el manejo del osteosarcoma?
 R = QUIMIOTERAPIA neoadyuvante mas RESECCIÓN con supervivencia de un 90%
TUMOR DE EWING
• 159. Donde se localiza frecuentemente el tumor de Ewing?
1) El sarcoma de Ewing es una neoplasia maligna de crecimiento rápido, que
NACE DE LAS CÉLULAS PRIMITIVAS DE LA MEDULA ÓSEA, normalmente en
la cavidad medular de HUESOS LARGOS.
2) Inicia creciendo en el interior de la cavidad medular, perforando el hueso
cortical de la diáfisis y elevando el periostio que da el signo
RADIOGRÁFICO EN CAPAS DE CEBOLLA.
• 160. Cual es el cc del tumor de Ewing?
 R = DOLOR, SENSIBILIDAD LOCAL, manifestaciones sistémicas. DEBIDO AL
CENTRO NECRÓTICO HAY FEBRÍCULA, LEUCOCITOSIS y elevación de la VSG.
• 161. Cual es el pronóstico del tumor de Ewing?
1) MALO CON UNA MORTALIDAD DEL 95% pese al tratamiento debido a
METÁSTASIS PULMONARES.
2) El tratamiento temprano reduce la mortalidad a 50% y se hace con quimio
y radioterapia.
OSTEOMIELITIS AGUDA
• 162. Cual es el microorganismo mas frecuente de la osteomielitis aguda?
1) STAPHYLOCOCCUS AUREUS, la puerta de entrada suele ser la piel a través de erosiones,
abrasiones, pústulas o quemaduras.
2) En LACTANTES los agentes mas frecuentes son los ESTREPTOCOCOS (NEUMOCOCO) O
HAEMOPHILUS INLUENZAE.
• 163. Cual es el cc característico de la osteomielitis aguda?
 R = Existe el ANTECEDENTE DE TRAUMA O INFECCIÓN recurrente de VÍAS RESPIRATORIAS O
PIEL. El primer síntoma es el dolor, acompañado de anorexia y FIEBRE. La TUMEFACCIÓN de
partes blandas es un signo tardío.
• 164. Que estudios de gabinete son útiles para el diagnostico de osteomielitis aguda?
 R = La radiografía demuestran cambios solo en etapas tardías, pero el CAMBIO MAS
PRECOZ DE ESTA ES UNA REACCIÓN PERIOSTICA. LA RESONANCIA Y LA GAMAGRAFIA
SON MAS ESPECIFICAS DE MANERA TEMPRANA.
• 165. Cual es el estándar de oro para el dx de osteomielitis aguda?
 R = LA BIOPSIA DE HUESO, a la que se le realiza CULTIVO CON ANTIBIOGRAMA.
• 166. Cual es el manejo de la osteomielitis aguda?
 R = Manejo empírico con PENICILINA G Y CLINDAMICINA DE 4-6 SEMANAS y
ESCARIFICACIÓN para aumentar el flujo sanguíneo y promover la formación de vasos nuevos
en el hueso.
OSTEOMIELITIS CRONICA
• 167. Cual es el cc de la osteomielitis crónica?
 R = EL paciente PERSISTE CON UNA LESIÓN RESIDUAL DOLOROSA
después de la resolución de la fase aguda, acompañada de
tumefacción, dolor a la palpación e impotencia.
• 168. Cuales son los hallazgos de gabinete en la osteomielitis
crónica?
 R = Se encuentra un SECUESTRO CLARO EN EL HUESO QUE SE
OBSERVAN COMO CAVIDADES RADIOLUCIDAS CON TEJIDO DE
GRANULACIÓN EN SU INTERIOR, con aumento de volumen de
tejidos blandos y posteriormente OSTEOPENIA y LISIS ÓSEA POR
DESTRUCCIÓN TRABECULAR.
• 169. Cual es el manejo de la ostemielitis crónica?
 R = RETIRAR TODO EL HUESO NECRÓTICO, y antibióticos partiendo
de los resultados de la biopsia de hueso.
ARTRITIS SEPTICA PIOGENA
• 171. Cuales son los factores de riesgo para la artritis séptica piógena (gonocócica o no)?
 R = Abuso de drogas IV, jóvenes sexualmente activos, enfermedades concomitante como DM, AR o lupus,
antecedentes de cirugía o prótesis.
• 172. Cual es el cc de la artritis séptica piógena gonocócica o no?
 R = Es una entidad de inicio agudo y progresivo caracterizado por dolor e inflamación local (salvo en el primer
año de vida donde es similar a la sepsis). El dolor es moderado a intenso con limitación funcional, aumento de
la temperatura local y rubor es monoarticular, cuando es poliarticular es en paciente inmunocomprometido
siendo la rodilla la mas afectada.
• 173. Cuales son los hallazgos radiológicos en la artritis séptica piógena?
 R = Edema de tejidos blandos y derrame articular. EN AFECCIÓN POR ANAEROBIOS SE VE GAS EN TEJIDOS
BLANDOS. El gamagrama óseo con tecnecio (valora flujo) o con galio (valora actividad inflamatoria) pueden
utilizarse de manera temprana.
• 174. Cual es el manejo de la artritis séptica piógena?
1) Consiste en la administración de antibióticos y drenaje de la articulación afectada. En caso de ARTRITIS
PIÓGENA debe administrarse de 3 A 4 SEMANAS y en caso de ARTRITIS GONOCÓCICA DE 7 A 14 DÍAS.
2) En menores de 2M: DICLOXACILINA mas CEFOTAXIMA
3) En LACTANTES: DICLOXACILINA mas CEFOTAXIMA
4) En niños MAYORES DE 3ª: DICLOXACILINA
5) Artritis séptica GONOCÓCICA: 1g por via IM o IV cada 24 hrs.
6) Artritis séptica en DROGADICTOS por via IV: DICLOXACILINA mas GENTAMICINA
7) Artritis séptica en INMUNOCOMPROMETIDOS: CEFTAZIDIMA mas AMIKACINA
8) Artritis séptica POSTOPERATORIA: VANCOMICINA mas AMIKACINA
9) Artitis séptica en FRACTURA ABIERTA: AMOXICILINA con ACIDO CLAVULANICA
RESUMEN DE LAS ENFERMEDADES
METABOLICAS OSEAS
ENFERMEDAD DE PAGET
• Figure shows an x-ray of a 40-year-old
Caucasian man with symptoms of sinusitis and
an incidental finding in the skull. Which of the
following is the most likely diagnosis?
(A) normal variant
(B) osteomyelitis
(C) Paget’s disease
(D) hemangioma
(E) metastatic disease
• (C) This is a rarefied area involving the frontal and
parietal bones, and it is an early stage of Paget’s
disease in which calvarial thickening and foci or
radiopacity are not present within the radiolucent
area. At this stage of the disease, a cross-section
through the margin of the lesion reveals a compact
inner and outer table in the normal portion, whereas
the diploe widens and extends to the outer and inner
surfaces of the calvarium without a change in the
calvarial thickness in the lesion.
• El tratamiento incluye AINES, bifosfonatos y
calcitonina.
CPK
• Las siguientes patologías cursan con aumento
de la CPK
1) HIPOTIROIDISMO
2) RAZA NEGRA
3) POLIMIOSITIS Y
4) DERMATOMIOSITIS
Consejos
1. Leer diariamente y en bloques
2. Adiós Partys un tiempo
3. Has ejercicio y come bien durante el estudio
4. Toma algún curso bueno si tienes la posibilidad
5. Ten Fe.
BIBLIOGRAFIA
• EXARMED
• PAPADAKIS
• CTO
• HARRISON
• AMIR
• USMLE STEPS

Más contenido relacionado

La actualidad más candente

Clase posgrado clinica 2013
Clase posgrado clinica 2013Clase posgrado clinica 2013
Clase posgrado clinica 2013
cursobianualMI
 
Sindrome Nefritico
Sindrome NefriticoSindrome Nefritico
Sindrome Nefritico
Luis Rios
 
Complicaciones crónicas de la diabetes
Complicaciones crónicas de la diabetesComplicaciones crónicas de la diabetes
Complicaciones crónicas de la diabetes
CFUK 22
 
Glomerulopatías
GlomerulopatíasGlomerulopatías
Glomerulopatías
Cynthia1502
 
Sindrome antifosfolipido
Sindrome antifosfolipidoSindrome antifosfolipido
Sindrome antifosfolipido
Olga Doria
 

La actualidad más candente (20)

HCM - Reumatología - LES
HCM - Reumatología - LESHCM - Reumatología - LES
HCM - Reumatología - LES
 
Sindrome de sjogren
Sindrome de  sjogrenSindrome de  sjogren
Sindrome de sjogren
 
NEFROPATIA IgA
NEFROPATIA IgANEFROPATIA IgA
NEFROPATIA IgA
 
NEFROLOGIA CLINICA: Glomerulonefritis membranosa
NEFROLOGIA CLINICA: Glomerulonefritis membranosaNEFROLOGIA CLINICA: Glomerulonefritis membranosa
NEFROLOGIA CLINICA: Glomerulonefritis membranosa
 
Glomerulopatías primarias Lizbeth García Esquivel
Glomerulopatías primarias Lizbeth García EsquivelGlomerulopatías primarias Lizbeth García Esquivel
Glomerulopatías primarias Lizbeth García Esquivel
 
Clase posgrado clinica 2013
Clase posgrado clinica 2013Clase posgrado clinica 2013
Clase posgrado clinica 2013
 
(2013-01-24) Sindrome de Sjögren ppt
(2013-01-24) Sindrome de Sjögren ppt(2013-01-24) Sindrome de Sjögren ppt
(2013-01-24) Sindrome de Sjögren ppt
 
Sindrome Nefritico
Sindrome NefriticoSindrome Nefritico
Sindrome Nefritico
 
SINDROME NEFROTICO CORTICORRESISTENTE TARDIO - PEDIATRIA
SINDROME NEFROTICO CORTICORRESISTENTE TARDIO - PEDIATRIASINDROME NEFROTICO CORTICORRESISTENTE TARDIO - PEDIATRIA
SINDROME NEFROTICO CORTICORRESISTENTE TARDIO - PEDIATRIA
 
Glomerulonefritis
GlomerulonefritisGlomerulonefritis
Glomerulonefritis
 
Síndrome nefrítico
Síndrome nefríticoSíndrome nefrítico
Síndrome nefrítico
 
Complicaciones crónicas de la diabetes
Complicaciones crónicas de la diabetesComplicaciones crónicas de la diabetes
Complicaciones crónicas de la diabetes
 
Guia Practica Clinica Lupus Eritematoso
Guia Practica Clinica Lupus EritematosoGuia Practica Clinica Lupus Eritematoso
Guia Practica Clinica Lupus Eritematoso
 
Glomerulopatías
GlomerulopatíasGlomerulopatías
Glomerulopatías
 
Anemia por enfermedad crónica - hemolítica
Anemia por enfermedad crónica - hemolíticaAnemia por enfermedad crónica - hemolítica
Anemia por enfermedad crónica - hemolítica
 
El Laboratorio En ReumatologíA
El Laboratorio En ReumatologíAEl Laboratorio En ReumatologíA
El Laboratorio En ReumatologíA
 
Sindrome antifosfolipido
Sindrome antifosfolipidoSindrome antifosfolipido
Sindrome antifosfolipido
 
NEFRITIS LÚPICA
NEFRITIS LÚPICANEFRITIS LÚPICA
NEFRITIS LÚPICA
 
Sindrome nefritico
Sindrome nefriticoSindrome nefritico
Sindrome nefritico
 
Glomerulonefritis Rápidamente Progresiva
Glomerulonefritis Rápidamente ProgresivaGlomerulonefritis Rápidamente Progresiva
Glomerulonefritis Rápidamente Progresiva
 

Destacado

2015 enfermedad de still del adulto
2015 enfermedad de still del adulto2015 enfermedad de still del adulto
2015 enfermedad de still del adulto
tangart88
 
(2012-12-11) Uso de los corticoides en la practica clinica (doc)
(2012-12-11) Uso de los corticoides en la practica clinica (doc)(2012-12-11) Uso de los corticoides en la practica clinica (doc)
(2012-12-11) Uso de los corticoides en la practica clinica (doc)
UDMAFyC SECTOR ZARAGOZA II
 
Corticoides y artritis reumatoidea
Corticoides y artritis reumatoideaCorticoides y artritis reumatoidea
Corticoides y artritis reumatoidea
Edgar Pazmino
 

Destacado (20)

5.NEFROLOGIA
5.NEFROLOGIA5.NEFROLOGIA
5.NEFROLOGIA
 
4.GASTROENTEROLOGIA Y CIRUGIA
4.GASTROENTEROLOGIA Y CIRUGIA4.GASTROENTEROLOGIA Y CIRUGIA
4.GASTROENTEROLOGIA Y CIRUGIA
 
2.CARDIOLOGIA
2.CARDIOLOGIA2.CARDIOLOGIA
2.CARDIOLOGIA
 
10.NEUMOLOGIA
10.NEUMOLOGIA10.NEUMOLOGIA
10.NEUMOLOGIA
 
7.DERMATOLOGIA
7.DERMATOLOGIA7.DERMATOLOGIA
7.DERMATOLOGIA
 
1.HEMATOLOGIA
1.HEMATOLOGIA1.HEMATOLOGIA
1.HEMATOLOGIA
 
PEDIATRIA ENARM
PEDIATRIA ENARMPEDIATRIA ENARM
PEDIATRIA ENARM
 
8.NEUROLOGIA
8.NEUROLOGIA8.NEUROLOGIA
8.NEUROLOGIA
 
6.INFECTOLOGIA
6.INFECTOLOGIA6.INFECTOLOGIA
6.INFECTOLOGIA
 
ENARM COMPENDIO
ENARM  COMPENDIOENARM  COMPENDIO
ENARM COMPENDIO
 
ENARM Obstetricia ginecologia
ENARM Obstetricia ginecologiaENARM Obstetricia ginecologia
ENARM Obstetricia ginecologia
 
Hipertension en el embarazo ACOG 2013
Hipertension en el embarazo ACOG 2013Hipertension en el embarazo ACOG 2013
Hipertension en el embarazo ACOG 2013
 
9.ENDOCRINOLOGIA
9.ENDOCRINOLOGIA9.ENDOCRINOLOGIA
9.ENDOCRINOLOGIA
 
Glucocorticoides
GlucocorticoidesGlucocorticoides
Glucocorticoides
 
Corticoides
CorticoidesCorticoides
Corticoides
 
Terapéutica en Reumatología
Terapéutica en ReumatologíaTerapéutica en Reumatología
Terapéutica en Reumatología
 
2015 enfermedad de still del adulto
2015 enfermedad de still del adulto2015 enfermedad de still del adulto
2015 enfermedad de still del adulto
 
(2012-12-11) Uso de los corticoides en la practica clinica (doc)
(2012-12-11) Uso de los corticoides en la practica clinica (doc)(2012-12-11) Uso de los corticoides en la practica clinica (doc)
(2012-12-11) Uso de los corticoides en la practica clinica (doc)
 
Corticoides y artritis reumatoidea
Corticoides y artritis reumatoideaCorticoides y artritis reumatoidea
Corticoides y artritis reumatoidea
 
Enfermedad de still
Enfermedad de stillEnfermedad de still
Enfermedad de still
 

Similar a 3.REUMATOLOGIA

Pediatric vasculitides
Pediatric vasculitidesPediatric vasculitides
Pediatric vasculitides
dattasrisaila
 
4. Lecture 3 - Classification of anemias.ppt
4. Lecture 3 - Classification of anemias.ppt4. Lecture 3 - Classification of anemias.ppt
4. Lecture 3 - Classification of anemias.ppt
KelfalaHassanDawoh
 

Similar a 3.REUMATOLOGIA (20)

Lupus overview for journalist
Lupus overview for journalistLupus overview for journalist
Lupus overview for journalist
 
ra-2013-final.pptx
ra-2013-final.pptxra-2013-final.pptx
ra-2013-final.pptx
 
強皮症腎クリーゼ
強皮症腎クリーゼ強皮症腎クリーゼ
強皮症腎クリーゼ
 
Artritis
ArtritisArtritis
Artritis
 
Peripheral vascular disease
Peripheral vascular diseasePeripheral vascular disease
Peripheral vascular disease
 
1 diff-diag- Dr. Ahmed El yasaqy
1 diff-diag- Dr. Ahmed El yasaqy1 diff-diag- Dr. Ahmed El yasaqy
1 diff-diag- Dr. Ahmed El yasaqy
 
Systemic lupus erythematosus
Systemic lupus erythematosusSystemic lupus erythematosus
Systemic lupus erythematosus
 
Multiple Myeloma (Case presentation)
Multiple Myeloma (Case presentation) Multiple Myeloma (Case presentation)
Multiple Myeloma (Case presentation)
 
Approach to and recent advances in the management of rheumatoid arthritis
Approach to and recent advances in the management of rheumatoid arthritisApproach to and recent advances in the management of rheumatoid arthritis
Approach to and recent advances in the management of rheumatoid arthritis
 
Approach to and recent advances in management of rheumatoid arthritis
Approach to and recent advances in management of rheumatoid arthritisApproach to and recent advances in management of rheumatoid arthritis
Approach to and recent advances in management of rheumatoid arthritis
 
Pediatric vasculitides
Pediatric vasculitidesPediatric vasculitides
Pediatric vasculitides
 
Orbital inflammatory disease
Orbital inflammatory diseaseOrbital inflammatory disease
Orbital inflammatory disease
 
extra articular manifestation of rheumatoid arthritis.pptx
extra articular manifestation of rheumatoid  arthritis.pptxextra articular manifestation of rheumatoid  arthritis.pptx
extra articular manifestation of rheumatoid arthritis.pptx
 
Rheumatoid arthritis
Rheumatoid arthritisRheumatoid arthritis
Rheumatoid arthritis
 
Deep venous thrombosis seminar
Deep venous thrombosis seminarDeep venous thrombosis seminar
Deep venous thrombosis seminar
 
Rhematoid arthritis by dr shyam sunder sharma
Rhematoid arthritis by dr shyam sunder sharmaRhematoid arthritis by dr shyam sunder sharma
Rhematoid arthritis by dr shyam sunder sharma
 
jama_siegel_2024_rv_240007_1712240383.96073.pptx
jama_siegel_2024_rv_240007_1712240383.96073.pptxjama_siegel_2024_rv_240007_1712240383.96073.pptx
jama_siegel_2024_rv_240007_1712240383.96073.pptx
 
Junior Medillectuals- Prelims
Junior Medillectuals- PrelimsJunior Medillectuals- Prelims
Junior Medillectuals- Prelims
 
Rheumatoid arthritis
Rheumatoid arthritisRheumatoid arthritis
Rheumatoid arthritis
 
4. Lecture 3 - Classification of anemias.ppt
4. Lecture 3 - Classification of anemias.ppt4. Lecture 3 - Classification of anemias.ppt
4. Lecture 3 - Classification of anemias.ppt
 

Último

Jalna Call Girls 👙 6297143586 👙 Genuine WhatsApp Number for Real Meet
Jalna Call Girls 👙 6297143586 👙 Genuine WhatsApp Number for Real MeetJalna Call Girls 👙 6297143586 👙 Genuine WhatsApp Number for Real Meet
Jalna Call Girls 👙 6297143586 👙 Genuine WhatsApp Number for Real Meet
Call Girls Service
 
ooty Call Girls 👙 6297143586 👙 Genuine WhatsApp Number for Real Meet
ooty Call Girls 👙 6297143586 👙 Genuine WhatsApp Number for Real Meetooty Call Girls 👙 6297143586 👙 Genuine WhatsApp Number for Real Meet
ooty Call Girls 👙 6297143586 👙 Genuine WhatsApp Number for Real Meet
Call Girls Service
 
Call Girls in Udaipur Girija Udaipur Call Girl ✔ VQRWTO ❤️ 100% offer with...
Call Girls in Udaipur  Girija  Udaipur Call Girl  ✔ VQRWTO ❤️ 100% offer with...Call Girls in Udaipur  Girija  Udaipur Call Girl  ✔ VQRWTO ❤️ 100% offer with...
Call Girls in Udaipur Girija Udaipur Call Girl ✔ VQRWTO ❤️ 100% offer with...
mahaiklolahd
 
palanpur Call Girls 👙 6297143586 👙 Genuine WhatsApp Number for Real Meet
palanpur Call Girls 👙 6297143586 👙 Genuine WhatsApp Number for Real Meetpalanpur Call Girls 👙 6297143586 👙 Genuine WhatsApp Number for Real Meet
palanpur Call Girls 👙 6297143586 👙 Genuine WhatsApp Number for Real Meet
Call Girls Service
 
Bhagalpur Call Girls 👙 6297143586 👙 Genuine WhatsApp Number for Real Meet
Bhagalpur Call Girls 👙 6297143586 👙 Genuine WhatsApp Number for Real MeetBhagalpur Call Girls 👙 6297143586 👙 Genuine WhatsApp Number for Real Meet
Bhagalpur Call Girls 👙 6297143586 👙 Genuine WhatsApp Number for Real Meet
Call Girls Service
 
Mangalore Call Girls 👙 6297143586 👙 Genuine WhatsApp Number for Real Meet
Mangalore Call Girls 👙 6297143586 👙 Genuine WhatsApp Number for Real MeetMangalore Call Girls 👙 6297143586 👙 Genuine WhatsApp Number for Real Meet
Mangalore Call Girls 👙 6297143586 👙 Genuine WhatsApp Number for Real Meet
Call Girls Service
 
neemuch Call Girls 👙 6297143586 👙 Genuine WhatsApp Number for Real Meet
neemuch Call Girls 👙 6297143586 👙 Genuine WhatsApp Number for Real Meetneemuch Call Girls 👙 6297143586 👙 Genuine WhatsApp Number for Real Meet
neemuch Call Girls 👙 6297143586 👙 Genuine WhatsApp Number for Real Meet
Call Girls Service
 
Patna Call Girls 👙 6297143586 👙 Genuine WhatsApp Number for Real Meet
Patna Call Girls 👙 6297143586 👙 Genuine WhatsApp Number for Real MeetPatna Call Girls 👙 6297143586 👙 Genuine WhatsApp Number for Real Meet
Patna Call Girls 👙 6297143586 👙 Genuine WhatsApp Number for Real Meet
Call Girls Service
 
Nanded Call Girls 👙 6297143586 👙 Genuine WhatsApp Number for Real Meet
Nanded Call Girls 👙 6297143586 👙 Genuine WhatsApp Number for Real MeetNanded Call Girls 👙 6297143586 👙 Genuine WhatsApp Number for Real Meet
Nanded Call Girls 👙 6297143586 👙 Genuine WhatsApp Number for Real Meet
Call Girls Service
 
Best Lahore Escorts 😮‍💨03250114445 || VIP escorts in Lahore
Best Lahore Escorts 😮‍💨03250114445 || VIP escorts in LahoreBest Lahore Escorts 😮‍💨03250114445 || VIP escorts in Lahore
Best Lahore Escorts 😮‍💨03250114445 || VIP escorts in Lahore
Deny Daniel
 
Top 20 Famous Indian Female Pornstars Name List 2024
Top 20 Famous Indian Female Pornstars Name List 2024Top 20 Famous Indian Female Pornstars Name List 2024
Top 20 Famous Indian Female Pornstars Name List 2024
Sheetaleventcompany
 
Sambalpur Call Girls 👙 6297143586 👙 Genuine WhatsApp Number for Real Meet
Sambalpur Call Girls 👙 6297143586 👙 Genuine WhatsApp Number for Real MeetSambalpur Call Girls 👙 6297143586 👙 Genuine WhatsApp Number for Real Meet
Sambalpur Call Girls 👙 6297143586 👙 Genuine WhatsApp Number for Real Meet
Call Girls Service
 
Thoothukudi Call Girls 👙 6297143586 👙 Genuine WhatsApp Number for Real Meet
Thoothukudi Call Girls 👙 6297143586 👙 Genuine WhatsApp Number for Real MeetThoothukudi Call Girls 👙 6297143586 👙 Genuine WhatsApp Number for Real Meet
Thoothukudi Call Girls 👙 6297143586 👙 Genuine WhatsApp Number for Real Meet
Call Girls Service
 
Call Girls Service Anantapur 📲 6297143586 Book Now VIP Call Girls in Anantapur
Call Girls Service Anantapur 📲 6297143586 Book Now VIP Call Girls in AnantapurCall Girls Service Anantapur 📲 6297143586 Book Now VIP Call Girls in Anantapur
Call Girls Service Anantapur 📲 6297143586 Book Now VIP Call Girls in Anantapur
gragmanisha42
 
Bihar Sharif Call Girls 👙 6297143586 👙 Genuine WhatsApp Number for Real Meet
Bihar Sharif Call Girls 👙 6297143586 👙 Genuine WhatsApp Number for Real MeetBihar Sharif Call Girls 👙 6297143586 👙 Genuine WhatsApp Number for Real Meet
Bihar Sharif Call Girls 👙 6297143586 👙 Genuine WhatsApp Number for Real Meet
Call Girls Service
 

Último (20)

Jalna Call Girls 👙 6297143586 👙 Genuine WhatsApp Number for Real Meet
Jalna Call Girls 👙 6297143586 👙 Genuine WhatsApp Number for Real MeetJalna Call Girls 👙 6297143586 👙 Genuine WhatsApp Number for Real Meet
Jalna Call Girls 👙 6297143586 👙 Genuine WhatsApp Number for Real Meet
 
Kochi call girls Mallu escort girls available 7877702510
Kochi call girls Mallu escort girls available 7877702510Kochi call girls Mallu escort girls available 7877702510
Kochi call girls Mallu escort girls available 7877702510
 
❤️Call girls in Jalandhar ☎️9876848877☎️ Call Girl service in Jalandhar☎️ Jal...
❤️Call girls in Jalandhar ☎️9876848877☎️ Call Girl service in Jalandhar☎️ Jal...❤️Call girls in Jalandhar ☎️9876848877☎️ Call Girl service in Jalandhar☎️ Jal...
❤️Call girls in Jalandhar ☎️9876848877☎️ Call Girl service in Jalandhar☎️ Jal...
 
Dehradun Call Girls 8854095900 Call Girl in Dehradun Uttrakhand
Dehradun Call Girls 8854095900 Call Girl in Dehradun  UttrakhandDehradun Call Girls 8854095900 Call Girl in Dehradun  Uttrakhand
Dehradun Call Girls 8854095900 Call Girl in Dehradun Uttrakhand
 
Escorts Service Ahmedabad🌹6367187148 🌹 No Need For Advance Payments
Escorts Service Ahmedabad🌹6367187148 🌹 No Need For Advance PaymentsEscorts Service Ahmedabad🌹6367187148 🌹 No Need For Advance Payments
Escorts Service Ahmedabad🌹6367187148 🌹 No Need For Advance Payments
 
ooty Call Girls 👙 6297143586 👙 Genuine WhatsApp Number for Real Meet
ooty Call Girls 👙 6297143586 👙 Genuine WhatsApp Number for Real Meetooty Call Girls 👙 6297143586 👙 Genuine WhatsApp Number for Real Meet
ooty Call Girls 👙 6297143586 👙 Genuine WhatsApp Number for Real Meet
 
Call Girls in Udaipur Girija Udaipur Call Girl ✔ VQRWTO ❤️ 100% offer with...
Call Girls in Udaipur  Girija  Udaipur Call Girl  ✔ VQRWTO ❤️ 100% offer with...Call Girls in Udaipur  Girija  Udaipur Call Girl  ✔ VQRWTO ❤️ 100% offer with...
Call Girls in Udaipur Girija Udaipur Call Girl ✔ VQRWTO ❤️ 100% offer with...
 
palanpur Call Girls 👙 6297143586 👙 Genuine WhatsApp Number for Real Meet
palanpur Call Girls 👙 6297143586 👙 Genuine WhatsApp Number for Real Meetpalanpur Call Girls 👙 6297143586 👙 Genuine WhatsApp Number for Real Meet
palanpur Call Girls 👙 6297143586 👙 Genuine WhatsApp Number for Real Meet
 
Kolkata Call Girls Miss Inaaya ❤️ at @30% discount Everyday Call girl
Kolkata Call Girls Miss Inaaya ❤️ at @30% discount Everyday Call girlKolkata Call Girls Miss Inaaya ❤️ at @30% discount Everyday Call girl
Kolkata Call Girls Miss Inaaya ❤️ at @30% discount Everyday Call girl
 
Bhagalpur Call Girls 👙 6297143586 👙 Genuine WhatsApp Number for Real Meet
Bhagalpur Call Girls 👙 6297143586 👙 Genuine WhatsApp Number for Real MeetBhagalpur Call Girls 👙 6297143586 👙 Genuine WhatsApp Number for Real Meet
Bhagalpur Call Girls 👙 6297143586 👙 Genuine WhatsApp Number for Real Meet
 
Mangalore Call Girls 👙 6297143586 👙 Genuine WhatsApp Number for Real Meet
Mangalore Call Girls 👙 6297143586 👙 Genuine WhatsApp Number for Real MeetMangalore Call Girls 👙 6297143586 👙 Genuine WhatsApp Number for Real Meet
Mangalore Call Girls 👙 6297143586 👙 Genuine WhatsApp Number for Real Meet
 
neemuch Call Girls 👙 6297143586 👙 Genuine WhatsApp Number for Real Meet
neemuch Call Girls 👙 6297143586 👙 Genuine WhatsApp Number for Real Meetneemuch Call Girls 👙 6297143586 👙 Genuine WhatsApp Number for Real Meet
neemuch Call Girls 👙 6297143586 👙 Genuine WhatsApp Number for Real Meet
 
Patna Call Girls 👙 6297143586 👙 Genuine WhatsApp Number for Real Meet
Patna Call Girls 👙 6297143586 👙 Genuine WhatsApp Number for Real MeetPatna Call Girls 👙 6297143586 👙 Genuine WhatsApp Number for Real Meet
Patna Call Girls 👙 6297143586 👙 Genuine WhatsApp Number for Real Meet
 
Nanded Call Girls 👙 6297143586 👙 Genuine WhatsApp Number for Real Meet
Nanded Call Girls 👙 6297143586 👙 Genuine WhatsApp Number for Real MeetNanded Call Girls 👙 6297143586 👙 Genuine WhatsApp Number for Real Meet
Nanded Call Girls 👙 6297143586 👙 Genuine WhatsApp Number for Real Meet
 
Best Lahore Escorts 😮‍💨03250114445 || VIP escorts in Lahore
Best Lahore Escorts 😮‍💨03250114445 || VIP escorts in LahoreBest Lahore Escorts 😮‍💨03250114445 || VIP escorts in Lahore
Best Lahore Escorts 😮‍💨03250114445 || VIP escorts in Lahore
 
Top 20 Famous Indian Female Pornstars Name List 2024
Top 20 Famous Indian Female Pornstars Name List 2024Top 20 Famous Indian Female Pornstars Name List 2024
Top 20 Famous Indian Female Pornstars Name List 2024
 
Sambalpur Call Girls 👙 6297143586 👙 Genuine WhatsApp Number for Real Meet
Sambalpur Call Girls 👙 6297143586 👙 Genuine WhatsApp Number for Real MeetSambalpur Call Girls 👙 6297143586 👙 Genuine WhatsApp Number for Real Meet
Sambalpur Call Girls 👙 6297143586 👙 Genuine WhatsApp Number for Real Meet
 
Thoothukudi Call Girls 👙 6297143586 👙 Genuine WhatsApp Number for Real Meet
Thoothukudi Call Girls 👙 6297143586 👙 Genuine WhatsApp Number for Real MeetThoothukudi Call Girls 👙 6297143586 👙 Genuine WhatsApp Number for Real Meet
Thoothukudi Call Girls 👙 6297143586 👙 Genuine WhatsApp Number for Real Meet
 
Call Girls Service Anantapur 📲 6297143586 Book Now VIP Call Girls in Anantapur
Call Girls Service Anantapur 📲 6297143586 Book Now VIP Call Girls in AnantapurCall Girls Service Anantapur 📲 6297143586 Book Now VIP Call Girls in Anantapur
Call Girls Service Anantapur 📲 6297143586 Book Now VIP Call Girls in Anantapur
 
Bihar Sharif Call Girls 👙 6297143586 👙 Genuine WhatsApp Number for Real Meet
Bihar Sharif Call Girls 👙 6297143586 👙 Genuine WhatsApp Number for Real MeetBihar Sharif Call Girls 👙 6297143586 👙 Genuine WhatsApp Number for Real Meet
Bihar Sharif Call Girls 👙 6297143586 👙 Genuine WhatsApp Number for Real Meet
 

3.REUMATOLOGIA

  • 2.
  • 3.
  • 4.
  • 5. CASO CLINICO • A 57-year-old man presents with a swollen, painful left knee. He fell on the knee 3 days ago while hurrying up the stairs. On examination, there is a swollen knee with palpable effusion and decreased range of motion. Adiagnostic tap is preformed and 5 cc of transparent fluid is removed. The WBC count is 1000/mL (20% polymorphonuclear neutrophils [PMNs]), glucose is equal to plasma, viscosity is high, and lactate dehydrogenase (LDH) is low. (A) normal synovial fluid (B) noninflammatory effusion (C) inflammatory effusion (D) septic arthritis (E) hemorrhagic effusion
  • 6. (B) This man has a noninflammatory effusion likely from the trauma caused by the fall. In the noninflammatory category, the fluid is transparent, WBC 200–2000/mL (<25% PMNs), glucose is normal, and LDH is low. Another cause for a noninflammatory effusion is osteoarthritis. (Kasper, p. 2032)
  • 7. CASO CLINICO • A77-year-old woman presents with a swollen, painful right knee. She fell on the knee 3 days ago while hurrying up the stairs. On examination, there is a swollen knee with palpable effusion and decreased range of motion. A diagnostic tap is preformed and 5 cc of opaque fluid is removed. The WBC count is 20,000/Ml (50% PMNs), glucose is lower than plasma, viscosity is low, and the LDH is high. (A) normal synovial fluid (B) noninflammatory effusion (C) inflammatory effusion (D) septic arthritis (E) hemorrhagic effusion
  • 8. (C) This woman has an inflammatory category of effusion in view of the opaque color, high WBC 2000–10,000 (>50% PMNs), low glucose, and high LDH. Common causes for this include crystal induced arthritis, SLE, and RA. In septic arthritis, WBC is usually 50,000/mL or more and often >100,000/mL with >75% PMNs. Other important tests on synovial fluid include Gram stain and culture when an inflammatory effusion is suspected clinically.
  • 9. ARTRITIS REUMATOIDE • 1.- En que enfermedades suelen ser positivos los ANA +?  R = LES, esclerodermia, Sjogren y enfermedad del tejido conectivo • 2. El FR que anticuerpo lo compone?  R = IgM dirigida contra el fragmento fc de IgG • 3. En que enfermedades se presenta comúnmente la trombocitosis?  R = Vasculitis, enfermedad de Still y AR. • 4. En que enfermedades se presenta comúnmente la trombocitopenia?  R = LES y Sx antifosfolipido • 5. Que tratamiento se utiliza en el fenómeno de Raynaud?  R = Nifedipino
  • 10. • 6. Cuales manifestaciones clínicas y de lab sugieren AR?  R = Artritis de MAS DE 3 ARTICULACIONES, RIGIDEZ MATUTINA, nódulo reumatoide y FR positivo lo sugieren. • 7. Cual es el HLA implicado en la AR?  R = HLA DR4 • 8. Cuales son los criterios revisados en 1987 para la clasificación de la AR? 1) Rigidez matutina MAYOR A 30 MINUTOS 2) Artritis de 3 O MAS articulaciones 3) Artritis de las manos 4) Artritis SIMÉTRICA 5) NÓDULOS reumatoides 6) FR positivo en suero 7) Cambios radiológicos • 9. Cual es el órgano blanco más afectado en la AR?  R = La membrana SINOVIAL • 10. Cuales son las manifestaciones extrarticulares más comunes de la AR?  R = ANEMIA NORMO-NORMO por uso de AINES con STDB, TROMBOCITOSIS, nódulos subcutáneos y SEQUEDAD DE MUCOSAS (Sx Sjogren secundario)
  • 11. • 11. Cuales son las manifestaciones provocadas por la AR en las glándulas exocrinas y mucosas?  R = Principalmente en las PARÓTIDAS, LAGRIMALES, mucosa y serosa de la VRS y CUELLO UTERINO produciendo DOLOR Y AUMENTO DEL VOLUMEN • 12. Cuales son las manifestaciones oftalmológicas de la AR?  R = EPIESCLERITIS y ESCLEROMALACIA (adelgazamiento). • 13. Cuales son las manifestaciones cardiovasculares de la AR? 1) VALVULARES: MITRAL Y AORTICA. 2) VASCULARES: Vasculitis y ateroesclerosis. • 14. Cuales son las manifestaciones renales de la AR?  R = MICROALBUMINURIA o MICROHEMATURIA. • 15. Que manifestación pulmonar da la AR?  R = FIBROSIS pulmonar manifestada en RX COMO PANAL DE ABEJA.
  • 12. • 16. En que consiste el Síndrome de FELTY?  R = AR, ESPLENOMEGALIA y LEUCOPENIA a expensas de neutrófilos. • 17. En que consiste el síndrome de CAPLAN?  R = AR, NODULOSIS reumatoide pulmonar y NEUMOCONIOSIS. • 18. Que anticuerpos en la AR tienen la mas alta especificidad diagnostica?  R = ANTICUERPOS ANTIPEPTIDOS CITRULINADOS ANTI- CCP • 19. Cual es el manejo de la AR? 1) Los FAME (fármacos moduladores de la enfermedad) deben utilizarse cuando se hace el dx y antes de que aparezcan los cambios erosivos. 2) La CLOROQUINA y el METOTREXATE son INDUCTORES DE REMISIÓN en AR. • 20. Cual es el efecto secundario mas común de la CLOROQUINA, la cual es un inductor de la remisión de AR?  R = Musculopatia (miopatía).
  • 14.
  • 15.
  • 16.
  • 17.
  • 18. CASO CLINICO • A 25-year-old woman develops painful swelling of both hands and wrists. She is also very stiff in the morning. Physical examination reveals erythema, swelling and joint line tenderness of the proximal interphalangeal, MCP, and wrists joints. Her RF is positive, ANA is negative, and x-rays of the hands show early erosive joint changes. Which of the following medications is most likely to prevent progression of disease? (A) D-penicillamine (B) antimalarial (C) methotrexate (D) NSAID or aspirin (E) gold
  • 19. (C) Methotrexate, 7.5–20 mg once weekly, is the most commonly recommended disease modifying drug, because its effect is more rapid and patients are able to tolerate it for longer periods of time. Maximum improvement with methotrexate occurs after 6 months of therapy. Toxicity includes GI upset, oral ulceration, and liver function abnormalities. GI upset in particular may be ameliorated by concurrent folic acid administration. Pneumonitis has also been reported.
  • 20. CASO CLINICO • A 29-year-old woman develops symptoms of painful swelling, and stiffness of both hands. Physical examination reveals involvement of the proximal interphalangeal joints and MCP joints. Aclinical diagnosis of rheumatoid arthritis is made. Which of the following is the most likely cause of the inflammation in her joints? (A) activated T cells (B) antineutrophil antibodies (C) microvascular injury (D) interleukin-4 (IL-4) (E) precipitated RF
  • 21. (A) Numerous mediators of inflammation are found in the synovium of patients with rheumatoid arthritis (RA). The evidence favoring activated T cells as the initiators of the inflammation include the predominance of CD4+ T cells in the synovium, the increase in soluble interleukin-2 (IL-2) receptors (a product of T-cell activation), and amelioration of symptoms by T-cell removal.
  • 22. CASO CLINICO • A 63-year-old man is worried that he has RA because his RF serology is positive in a low titer. He has pain in his left hand and right knee, which usually bother him in the evening. He has not noticed any inflammation or swelling and there is no history of morning stiffness. On examination, there are no active joints. Which of the following statements regarding the presence of RF is correct? (A) is positive in 10–20% of people over age 65 (B) is positive in almost 100% of “classical” RA (C) is seen only in RA (D) is always abnormal (E) is frequently present in osteoarthritis
  • 23. (A) The presence of RF has little predictive power in determining the diagnosis of RA. However, it is useful in determining prognosis, as high titers of RF are associated with more severe and progressive disease, as well as with extra-articular manifestations.
  • 24. CASO CLINICO • A 30-year-old woman develops painful swelling of her hands, pleuritic chest pain, and shortness of breath on exertion. She is also very stiff in the morning. Physical examination reveals involvement of the proximal interphalangeal joints and MCP joints. Air entry is decreased to the right lower lobe and the area is dull on percussion. Her RF is elevated and a CXR reveals a pleural effusion. A diagnostic tap is performed, and 500 cc of a straw-colored fluid is removed. Which of the following biochemical patterns is consistent with a pleural effusion due to her primary condition? (A) exudate protein and LDH ratios with low glucose (B) exudate protein and LDH ratios with high glucose (C) transudate protein and LDH ratios with high glucose (D) transudate protein and LDH ratios with low glucose (E) normal pleural fluid
  • 25. (A) Pleuritis is common at autopsy in patients with RA but is not usually symptomatic. Typically, the pleural fluid shows increased pleural fluid: serum protein and LDH ratios (exudate pattern), and low glucose and low complement levels. Pleuropulmonary manifestations are more common in men with
  • 26. CASO CLINICO • A 54-year-old woman with rheumatoid arthritis (RA) presents with fatigue and low white count (white blood cells [WBC] 2500/mL) on routine blood work. She has no active joint symptoms and her RA is controlled on low dose methotrexate and NSAIDs. On examination, she has chronic joint deformities of her hands and a palpable spleen, which is a new finding. Which of the following is the most likely diagnosis for her low white count? (A) methotrexate therapy (B) rheumatoid nodules disrupting bone marrow architecture (C) Felty’s syndrome (D) normal variation (E) myelofibrosis
  • 27. • (C) The triad of chronic RA, splenomegaly, and neutropenia is called Felty’s syndrome. It is associated with high titers of RF and extraarticular disease. The increased susceptibility to infections is secondary to both decreased neutrophil number and function. Felty’s syndrome is rare in African Americans.
  • 28. CASO CLINICO • A 29-year-old woman develops painful swelling of both hands. She is also very stiff in the morning. Physical examination reveals involvement of the proximal interphalangeal joints and metacarpophalangeal (MCP) joints. Her RF is positive and ANA is negative. Which of the following medications is most likely to improve her joint pain symptoms? (A) D-penicillamine (B) an antimalarial (C) methotrexate (D) NSAID or aspirin (E) gold
  • 29. • (D) This patient has RA and aspirin or other nonsteroidal agents are effective medications for relieving the signs and symptoms of disease. They do little to modify the course of the disease, however. The new generation of NSAIDs that are more specific inhibitors of cyclooxygenase 2 cause less GI toxicity. Glucocorticoids are very powerful at suppressing signs and symptoms of disease and may alter disease progression. Methotrexate is an important disease modifying drug (DMRD) used to prevent joint destruction. Gold and antimalarials were important DMRDs in the past before the use of methotrexate and newer “biological” agents.
  • 30. CASO CLINICO • A 50-year-old man develops fatigue and painful swelling of both hands. He is also very stiff in the morning and requires longer time to get ready for work. Physical examination reveals erythema, swelling, and tenderness on palpation of the proximal interphalangeal joints and MCP joints. Plain x- rays of the hand are taken. Which of the following x-ray findings is characteristic of this condition? (A) loss of articular cartilage and bone erosion (B) normal (C) osteolytic changes (D) osteosclerotic changes (E) osteolytic and osteosclerotic changes together
  • 31. (A) This patient has features of rheumatoid arthritis, and early in RA there may not be any bony changes seen, except nonspecific findings of soft tissue swelling and joint effusions. With longer active inflammation of the joints, loss of cartilage and bony erosions can be seen. The value of x-rays is to determine the extent of bone and cartilage damage.
  • 32. CASO CLINICO • A 40-year-old woman complains of being very “stiff” in the morning, in addition to having sore hands and wrists. The symptoms have come on very gradually and she can not recall exactly when they started. Her only other complaint is that of being fatigued. Physical examination reveals involvement of the proximal interphalangeal joints, MCP joints, and wrist joints in a symmetrical fashion. She also has nodules over both elbows. Which of the following is predictive of developing extraarticular features of her condition? (A) her knees are involved early (B) there is a poor articular response to disease-suppressing medication (C) humoral immunity is suppressed (D) cellular immunity is suppressed (E) she develops an antibody to her own immunoglobulin (RF)
  • 33. (E) Extra-articular manifestations of RA generally develop in patients with high titers of autoantibody to the Fc component of IgG (also known as rheumatoid factor (RF).
  • 34. CASO CLINICO • A 67-year-old man has a long history of symmetrical small joint arthritis with deformities. He now develops shortness of breath on exertion with a dry cough, but no sputum or chest discomfort. His heart sounds have a loud P2, and the lungs have fine bibasilar crackles. Which of the following is the most likely diagnosis of the pulmonary condition associated with his arthritis? (A) pleuritis (B) cavitating lesions (C) intrapulmonary nodules (D) interstitial fibrosis (E) diffuse pneumonitis
  • 35. (D) In RA, pleural involvement is very common at autopsy but infrequently causes symptoms. Interstitial lung disease (ILD) is the most common manifestation of rheumatoid lung disease. RA associated interstitial lung disease (RA-ILD) is usually similar to idiopathic pulmonary fibrosis (IPF) in terms of its clinical presentation, pathology, disease spectrum, and pathogenesis. Presentation is more common at age 50–60 years, in men (M:F = 2–3:1), and in association with seropositive and erosive joint disease. If pleural fluid is present, glucose levels are very low.
  • 36. OSTEOARTROSIS • 21. Cual es la causa de dolor e incapacidad mas frecuente en ancianos? R = La OSTEOARTROSIS • 22. Los nódulos de Heberden (distales) y de Bouchard (proximales) en que enfermedad se manifiestan? R = Osteoartrosis • 23. Cual es el órgano blanco de la OA? R = CARTÍLAGO ARTICULAR con distribución asimétrica
  • 37. • 24. Cual es el cuadro clínico y radiologico de la OA?  R = Dolor articular, RIGIDEZ MATUTINA DE NO >30 MIN después de haber realizado una actividad leve-moderada una noche antes, perdida de la funcionabilidad, CRECIMIENTO DE OSTEOFITOS, micro fracturas, irritación de terminales nerviosas. • 25. Como diagnosticas OA? 1) HC, no se necesitan pruebas de laboratorio. 2) Radiográficamente: DISMINUCIÓN DEL ESPACIO ARTICULAR, presencia de osteofitos o quistes subcondrales e incremento de la densidad ósea sub condral, lo que se le llama esclerosis subcondral. • 26. Cual es el tx de elección de la OA?  R = ACIDO HIALURONICO, PARACETAMOL y demás AINES. El TRAMADOL se utiliza en pacientes que no toleran AINES, inhibidores de la COX-2 o déficit renal grave. • 27. Cual es el tx de la OA utilizado en pacientes intolerantes a AINES o con daño renal?  R = TRAMADOL, GLUCOSAMINA, GLUCOCORTICOIDES INTRARTICULARES en caso de proceso agudo, HIALURONATO el cual restaura las propiedades elásticas y de viscosidad del liquido sinovial, CIRUGÍA.
  • 38. CASO CLINICO • A 60-year-old man has pain in his left hand and right knee, which is interfering with his work. The pain came on gradually, first in his hand 6 months ago and now in his knee. It is usually fine when he wakes up, but gets worse as the day progresses. There is no history of any trauma, and he is otherwise well. Taking over-the-counter NSAIDs usually relieves the pain. On examination, there is bony soft tissue swelling of his second and third DIP joints in the left hand and crepitus over the right knee with flexion. There is no erythema or joint effusion. Which of the following characteristics is a risk factor for this condition? (A) being Chinese (B) being African (C) being male (D) being overweight (E) hyperthyroidism
  • 39. CASO CLINICO • A 67-year-old woman has pain in her left hand and right knee, which is interfering with her activities. The pain came on gradually, first in her hand 6 months ago and now in her knee. It is usually fine when she wakes up, but gets worse as the day progresses. There is no history of any trauma, and she is otherwise well. Taking over-the-counter NSAIDs usually relieves the pain. On examination, there is bony soft tissue swelling of his second and third DIP joints in the left hand and crepitus over the right knee with flexion. There is no erythema or joint effusion. Which of the following best describes this disease condition? (A) disease of the synovial membrane (B) disease of the articular cartilage (C) disease of the entire joint (D) disease of the subchondral bone (E) disease of the ligaments
  • 40.
  • 41. CASO CLINICO • Which of the following is the most common location for osteoarthritis? (A) hip (B) base of thumb (C) knee (D) spine (E) DIP joints of hand
  • 42. (E) Heberden’s nodes, bony enlargement of the DIP joints of the hand, are the most common type of osteoarthritis. Although they can present acutely with pain and inflammation, they are frequently slow in developing and relatively asymptomatic.
  • 43. CASO CLINICO • A 60-year-old man has pain in his left hand and right knee, which is interfering with his work. The pain came on gradually, first in his hand 6 months ago and now in his knee. It is usually fine when he wakes up, but gets worse as the day progresses. There is no history of any trauma, and he is otherwise well. Taking over-the-counter NSAIDs usually relieves the pain. On examination, there is bony soft tissue swelling of his second and third DIP joints in the left hand and crepitus over the right knee with flexion. There is no erythema or joint effusion. Which of the following characteristics is a risk factor for this condition? (A) being Chinese (B) being African (C) being male (D) being overweight (E) hyperthyroidism
  • 44. (D) There are numerous diseases and risk factors associated with the development of osteoarthritis. People of Chinese and African heritage have a lower risk than Caucasians, while Native Americans have a higher risk. Women are more affected than men, and obesity is a significant risk factor. Hyperthyroidism is not one of the many metabolic/endocrine disorders associated with osteoarthritis.
  • 45. CASO CLINICO • A 57-year-old man has pain in his left hand and right knee, which is interfering with his work. The pain came on gradually, first in his hand 6 months ago and now in his knee. It is usually fine when he wakes up, but gets worse as the day progresses. There is no history of any trauma, and he is otherwise well. Taking over-the-counter NSAIDs usually relieves the pain. On examination, there is bony soft tissue swelling of his second and third distal interphalangeal (DIP) joints in the left hand and crepitus over the right knee with flexion. There is no erythema or joint effusion. Which of the following is the initial change noticed in the pathogenesis of osteoarthritis? (A) abnormal chondrocyte function (B) a defect in the extracellular matrix of cartilage (C) inflammatory changes in subchondral bone (D) ligament inflammation (E) synovial inflammation
  • 46. • (B) It is most likely that the primary change in osteoarthritis occurs in the cartilage. It is possible that there is a disruption of the collagen network of the cartilage, specifically a disruption of the “glue” holding together adjacent fibers.
  • 47. CASO CLINICO • A 74-year-old woman has pain in her left hand and right knee, which started months ago, and is now interfering with her activities. The pain gets worse as the day progresses. There is no history of any trauma, and she is otherwise well. Taking over-the-counter acetaminophen usually relieves the pain. On examination, there is bony soft tissue swelling of her second and third DIP joints in the left hand and crepitus over the right knee with flexion. There is no erythema or joint effusion. Which of the following is the most likely explanation for the joint pain of osteoarthritis? (A) synovial inflammation is not the cause (B) ligament inflammation is a common cause (C) clinically visible (via plain x-ray) fractures are a common cause of pain (D) osteophytes can cause pain (E) muscles are not involved
  • 48. • (D) Osteophytes can cause pain by stretching periosteal nerve endings. Synovial inflammation is frequently seen in osteoarthritis, but not in ligament inflammation. Microfractures, but not macrofractures, commonly cause pain. Muscle spasm can be an important factor in the joint pain.
  • 49. Policondritis recidivante • A 45-year-old man has had intermittent swelling and pain in the superior part of his auricles for several years. Mild arthritis usually accompanies these episodes. Last year he also had redness, pain, and swelling over the bridge of his nose. Which of the following is the most likely diagnosis? (A) psoriatic arthritis (B) Behçet’s syndrome (C) Wegener’s granulomatosis (D) relapsing polychondritis (E) rheumatoid arthritis
  • 50. (D) The disease is relapsing polychondritis and is characterized by frequent remissions and exacerbations of lesions and is rarely fatal. Auricular chondritis and nasal chondritis are the most common manifestations. It can also be secondary to SLE, RA, Sjögren’s syndrome, and vasculitis.
  • 51. GOTA• 28. Cual es el cc de la gota? 1) Los primeros ataques suelen afectar una sola articulación siendo la mas frecuente LA PRIMERA MTF (podagra), tobillos, RODILLAS (gonagra), tarsos, muñecas y dedos de las manos, cursa con FIEBRE. 2) La cetoacidosis diabética, anemia hemolítica, síndrome de lisis tumoral y psoriasis cursan con hiperuricemia. • 29. Menciona algunos fármacos uricosuricos y cuando se trata la gota asintomática? 1) PROBENECID, benzobromarona, LOSARTAN y sulfinpirazona. 2) La gota asintomática se trata en casos de litiasis renal, acido úrico > 11, ataques de gota reiterados y gota crónica. • 30. Que birrefringencia te indica la identificación de cristales de urato sódico en el liquido sinovial?  R = Birrefringencia NEGATIVA intensa, tienen FORMA DE AGUJA. • 31. Que característica macroscópica tiene el liquido sinovial en un px con ataque agudo de gota?  R = Apariencia PURULENTA con predominio de PMN • 32. Que medicamento esta contraindicado en el ataque agudo de gota?  R = ALOPURINOL que es un inhibidor de la Xantina-oxidasa • 33. Cual es el efecto adverso más importante de los uricosuricos?  R = Pueden causar NEFROLITIASIS
  • 52.
  • 53. • 34. Que causa la pseudogota?  R = Cristales de PIROFOSFATO DE CALCIO • 35. Que etiología tiene la Pseudogota?  R = Hemocromatosis, hiperparatiroidismo, hipofosfatasia e hipomagnesemia. • 36. Cual es el cuadro clínico de la pseudogota?  R = Ataque MONOARTICULAR u oligoarticular, PSEUDOPODAGRA, rodillas (gonagra) y DEDOS DE MANOS (queiagra). • 37. Cual es la birrefringencia de la pseudogota?  R = Birrefringencia POSITIVA pero débil, tienen FORMA DE BASTONES O ROMBOS. • 38. Cual es el tratamiento de elección de la pseudogota?  R = PARACETAMOL • 39. Cual es el tratamiento profiláctico de la pseudogota?  R = COLCHICINA .6 mg c/12
  • 54.
  • 55.
  • 56. CASO CLINICO • A 72-year-old man injures his right knee in a car accident, and now it is swollen and extremely painful to bend the knee. X-rays of the knee rule out a fracture, and joint fluid aspiration reveals an opaque-colored fluid containing rhomboid crystals with weak-positive birefringence. Which of the following is the most appropriate next step in management? (A) oral prednisone (B) intravenous antibiotics (C) oral NSAIDs (D) acetaminophen (E) allopurinol
  • 57. (C) In traumatic arthritis, swellings, ecchymoses, muscular spasms, and tenderness tend to be present, but fractures must be excluded. This man has calcium pyrophosphate (CPPD) crystalinduced monoarthritis, so called “pseudogout.” It is most common in the elderly and can be precipitated by minor trauma. The crystals have a rhomboid shape, and the clinical presentation can mimic that of gout. It can be associated with metabolic abnormalities such as hyperparathyroidism or hemochromatosis. Treatment is with an NSAID for 7–10 days. If there are multiple joints involved, then steroids can be considered. An alternative to oral NSAIDs is intra- articular steroids for single joint disease. Allopurinol is not effective in CPPD. Synovial fluid is bloody, but the fluid is of normal viscosity, so a “string” test is usually positive.
  • 58. CASO CLINICO • A63-year-old man develops pain and swelling in his knee. It appears warm, red, and swollen with decreased range of movement. A diagnostic aspiration is performed. Which of the following will most likely distinguish pseudogout from gout? (A) positive birefringent crystals (B) acute onset (C) involvement of single joints (D) involvement of large joints (E) association with diabetes
  • 59. • (A) Pseudogout (calcium pyrophosphate crystals—CPPD) is distinguishable from gout by positive birefringent crystals. CPPD are short, blunt rhomboids, and urate crystals (seen in gout) are needle-shaped with negative birefringence.
  • 60. CASO CLINICO A 63-year-old man with an 8-year history of recurrent severe arthritis in his large toes has an elevated creatinine level. Which of the following mechanisms is the most likely explanation for his renal impairment? (A) GN (B) vascular injury (C) uric acid kidney stones (D) distal tubular atrophy (E) renal parenchymal uric acid crystals
  • 61. (E) The typical renal lesions in gout are urate crystals in the medulla or pyramids, with surrounding mononuclear and giant cell reaction. The degree of renal impairment, however, does not correlate with hyperuricemia, and the decline in renal function correlates with aging, hypertension, renal calculi, or unrelated nephropathy. (Kasper, p. 1704)
  • 62. ESPONDILITIS ANQUILOSANTE • 40. Que espondiloartropatias se asocian al HLA-B27?  R = ESPONDILITIS ALQUILOSANTE, ARTRITIS REACTIVA, ESPONDILOARTROSIS PSORIASICA, uveítis anterior aguda. • 41. Cual es el síntoma inicial de la espondilitis alquilosante (EA)?  R = DOLOR en regiones SACROILIACAS Y LUMBARES. En la ex fis HAY DISMINUCIÓN DE LA MOVILIDAD DE LA COLUMNA LUMBAR, dolor al presionar las regiones sacroiliacas y disminución de la expansión del tórax. • 42. Cual es la manifestación extrarticular mas frecuente de la EA?  R = UVEITIS ANTERIOR, 2/3 de px tiene CAMBIOS EN LA MUCOSA Y SUBMUCOSA DEL COLON E ÍLEON TERMINAL • 43. Cuales son los hallazgos más comunes en imagen obtenidas en la espondilitis anquilosante?  R = Erosiones y esclerosis sacroiliacas, IMAGEN EN COLUMNA DE BAMBU • 44. Cual es el manejo de la EA?  R = AINES, COX-2, Infliximab, Etanercept mas terapia física. Indometacina, INFLIXIMAB que disminuye el FNTa.
  • 63.
  • 64.
  • 65.
  • 66. CASO CLINICO • A 22-year-old man has symptoms of low back pain and stiffness. After several months of mild symptoms, he notes more severe stiffness at night and hip pain. On physical examination, there is paravertebral muscle tenderness and limited flexion of the lumbar spine. Figure shows an x-ray of the lumbar spine. Which of the following is the most likely diagnosis? (A) Reiter syndrome (B) Marfan syndrome (C) ankylosing spondylitis (AS) (D) RA (E) pseudogout
  • 67. CASO CLINICO • A 27-year-old man has a history of low back pain and stiffness. After several months of mild symptoms, he notes more severe stiffness at night and hip pain. On physical examination, there is paravertebral muscle tenderness and limited flexion of the lumbar spine. X-ray of the lumbar spine shows sacroiliitis. In addition to recommending physiotherapy and exercise, which of the following is the most appropriate next step in management? (A) NSAID therapy (B) phenylbutazone (C) azathioprine (D) acetaminophen (E) prednisone
  • 68. • (A) All NSAIDs are probably equally effective in the treatment of this man’s ankylosing spondylitis. Options include indomethacin or naproxen, but not phenylbutazone since it can cause aplastic anemia. Exercise and maintaining proper posture are very important.
  • 69. CASO CLINICO • A 27-year-old man has a history of low back pain and stiffness. Recently, he has noticed more severe stiffness at night and hip pain. The symptoms improve in the morning after doing some “stretching” exercises. On physical examination, there is paravertebral muscle and sacroiliac joint tenderness with limited flexion of the lumbar spine. A2/6 diastolic murmur is also heard at the left sternal border radiating to the apex. Which of the following is the most likely diagnosis for the diastolic murmur? (A) mitral stenosis (B) tricuspid stenosis (C) aortic insufficiency (D) pulmonic insufficiency (E) tetralogy of Fallot
  • 70. (C) The frequency of aortic insufficiency has been about 4% in ankylosing spondylitis (AS). Other cardiac valve anomalies are not increased in incidence. Rarely, congestive heart failure or third degree heart block can occur as well. (Kasper, p. 1994).
  • 71. CASO CLINICO • A23-year-old man notices new low back pain, stiffness, and left eye discomfort. Sunlight also bothers his eyes. The back pain is worse at night and described as a dull ache in the back and buttock area. On physical examination, there is paravertebral muscle, iliac crest, and ischial tuberosity tenderness with limited flexion of the lumbar spine. His eye is inflamed and the pupil is constricted. Pelvic x-rays show sacroiliitis. Which of the following is the most likely diagnosis for his eye symptoms (it is the most common extra-articular manifestation of this condition)? (A) glaucoma (B) acute anterior uveitis (C) keratitis (D) conjunctivitis (E) episcleritis
  • 72. (B) Acute anterior uveitis is the most common extra-articular manifestation of ankylosing spondylitis (AS). Pain, photophobia, and increased lacrimation are the usual symptoms. Attacks are unilateral and tend to recur, often in the other eye. Cataracts and secondary glaucoma are not uncommon sequelae. The iritis is usually managed with local glucocorticoid administration in association with a mydriatic agent.
  • 73. ENFERMEDAD DE REITER • 45. Cuales son los datos más comunes de ARTRITIS REACTIVA o enfermedad de Reiter?  R = Ligado al HLA B27, oligoartritis, CONJUNTIVITIS, URETRITIS, CERVICITIS, ULCERAS ORALES x un CUADRO INFECCIOSO PREVIO. • 46. Que precede a una manifestación de ARTRITIS REACTIVA o síndrome de Reiter?  R = Alguna INFECCIÓN por SALMONELLA, SHIGELLA, YERSINIA, CLAMYDIA TRACHOMATIS +++o CAMPYLOBACTER que PRECEDE a la MANIFESTACIÓN ARTICULAR 3-4 semanas después. • 47. Cual es el estudio de laboratorio o gabinete de elección del síndrome de Reiter?  R = CULTIVO CON ANTIBIOGRAMA, se puede encontrar también Ac anti bacteriano en el suero o liquido sinovial o DNA bacteriano.
  • 74.
  • 75. ARTRITIS SORIASICA • 48. Que es la artritis soriasica?  R = Ligado al HLA B27. Se define como una ARTRITIS usualmente SERONEGATIVA que se ASOCIA A SORIASIS. • 49. Cual es el cuadro clínico de la artritis soriasica?  R = PRECEDE PSORIASIS A ARTRITIS, ARTRITIS ASIMÉTRICA con DEDOS EN APARIENCIA DE SALCHICHA (DACTILITIS) de los dedos de manos y pies. • 50. Como diagnosticas artritis soriasica?  R = HC, y RX con evidencia de SACROILEITIS ASIMÉTRICA, alteraciones de la conformación de hueso nuevo con IMAGEN DE PUNTA DE LÁPIZ EN UNA COPA. • 51. Cual es el manejo de la artritis soriasica?  R = AINES E INFLIXIMAB. Tratamiento de soriasis.
  • 76.
  • 77. CASO CLINICO • A 19-year-old man has a chronic papulosquamous skin disorder involving his knees and elbows. He now has dull low back pain and morning stiffness that lasts a few hours. On examination, forward flexion at the lumbar spine is reduced but neck movements are normal. There is stress tenderness of both sacroiliac joints. Which of the following is the most likely diagnosis? (A) RA (B) ankylosing spondylitis (AS) (C) psoriatic arthritis (D) reactive arthritis (E) PMR
  • 78. (C) This patient has a sacroiliac form of arthritis seen in patients with psoriasis. Patients with psoriasis can develop five different patterns of musculoskeletal symptoms. These include arthritis of the DIP joints; asymmetric oligoarthritis; symmetric polyarthritis similar to RA; spine and sacroiliac type; and arthritis mutilans, a highly destructive form. Most patients with psoriatic arthritis also have nail involvement. Only about a quarter actually develop a progressive, destructive disease. Uric acid may be elevated because of high tissue turnover but is not part of the pathogenesis of joint disease.
  • 79. ARTRITIS GONOCOCICA • 52. Cual es el cuadro clínico de la artritis gonococcica? R = Poliartritis, tenosinovitis (60%) y monoartritis purulenta en la rodilla 40% • 53. Como diagnosticas y tratas la artritis gonococcica? R = Cultivo de liquido sinovial, CEFTRIAXONA iv o LEVOFLOXACINO a los alérgicos a penicilina.
  • 80. HLA B27 • Cuales patologías están ligadas al HLA B27? 1) Espondilitis anquilosante 2) Artritis reactiva o Síndrome de Reiter 3) Artritis soriasica
  • 81.
  • 83. CUCI Y CHRON • 54. Cuales son los 2 tipos de artropatía en px con enfermedad de Chron y CUCI?  R = ESPONDILITIS ANQUILOSANTE y ARTRITIS PERIFÉRICA. • 55. Cual es el cc de la enfermedad de CHRON?  R = DOLOR ABDOMINAL, FIEBRE, diarrea, ULCERAS ORALES, anorexia, hipersensibilidad de vientre al tacto, masa abdominal, disminución de peso y SANGRE EN EVACUACIONES. • 56. Cual es el cc de CUCI?  R = Diarrea, HEMATOQUEZIA, DOLOR ABDOMINAL con hipersensibilidad y LEUCOS AUMENTADOS EN LAS EVACUACIONES.
  • 84. CASO CLINICO • A 32-year-old woman has a long history of intermittent bloody diarrhea and crampy abdominal pain. Recently, her bowel symptoms have increased and she is having 4–6 bowel movements a day with mucus and blood in the stool. She now has a low-grade fever, loss of appetite, and new musculoskeletal symptoms. Which of the following is the best description of this patient’s accompanying arthritis? (A) asymmetric migratory polyarthritis involving the large joints of the arms and legs (B) not usually associated with disease flares (C) a progressive, crippling course (D) symmetrical small joint involvement (E) seropositive
  • 85. (A) In inflammatory bowel disease, there are two common types of arthritis involvement. The first is an asymmetric, migratory polyarthritis that affects the large joints of the lower and upper extremities and is closely related to the activity of bowel disease. Spondylitis is also common (though not always symptomatic) and is not always related to activity of bowel disease.
  • 86. ESPONDILOARTROPATIA INDIFERENCIADA • 57. Que caracteriza a las espondiloartropatias indiferenciadas?  R = TODOS AQUELLOS CC QUE REÚNEN CRITERIOS DE CLASIFICACIÓN de las espondiloartropatias PERO NOLOS CRITERIOS ESPECÍFICOS o cc característicos de c/u de ellas. • 58. Cuales son las manifestaciones clínicas mas comunes de las espondiloartropatias indiferenciadas?  R = 60-75% desarrolla datos de espondilitis alquilosante o artritis soriasica. • 59. Cuales son los medicamentos utilizados en las espondiloartropatias indiferenciadas?  R = AINES, COX-2, infliximab
  • 87.
  • 88. FIBROMIALGIA • 60. Como se define a la fibromialgia?  R = Dolores difusos, presentes por > 3 MESES y que involucran los 4 CUADRANTES DEL CUERPO Y ESQUELETO AXIAL, NO HAY SUEÑO REPARADOR y AMANECEN CANSADOS + la presencia a la PALPACIÓN DIGITAL DE 11 PUNTOS DOLOROSOS DE LOS 18. • 61. Que estudio auxiliar te ayuda a diagnosticar fibromialgia?  R = No hay estudio que pueda ayudar • 62. Cual es el tratamiento de la fibromialgia? 1) AMITRIPTILINA antes de dormir, 2) FLUOXETINA para el dolor, no hay tratamiento especifico • 63. Cual es el pronóstico de la fibromialgia?  R = DESFAVORABLE ya que el 25% desarrolla incapacidad importante.
  • 89. SX ANTIFOSFOLIPIDO • 65. Cuales son los criterios laboratoriales diagnósticos de SAF?  R = Anticuerpos ANTICARDIOLIPINAS y anticoagulante LUPICO. • 66. Cual es la etiología del SAF?  R = Se desconoce. Los linfocitos T. • 67. Cuales son los mecanismos por los cuales se presenta trombosis en el SAF?  R = INHIBICIÓN de la actividad ANTICOAGULANTE, inhibición de FIBRINÓLISIS y respuesta mediada por células aumentada • 68. Cual es el cc característico del SAF?  R = TROMBOSIS, livedo reticularis, ABORTOS FRECUENTES. • 69. Que es el SAF catastrófico?  R = Resulta de una forma acelerada que produce falla organica multiple • 70. Cual es el manejo del SAF?  R = PREDNISONA ELIMINA ANTICOAGULANTE. ASA 100 mg cada 24 hrs, estatinas, HEPARINA EN EMBARAZADAS y WARFARINA DESPUÉS DEL EMBARAZO.
  • 90.
  • 91.
  • 92. FIEBRE REUMATICA • 71. En la fiebre reumática, dentro de las manifestaciones cardiacas, que podemos encontrar? R = NÓDULOS/cuerpos de ASCHOFF (que son una confluencia de monocitos y macrófagos rodeados por fibrosis) y CARDITIS (Infiltrado inflamatorio que abarca las 3 capas del corazón). Se utilizan LOS CRITERIOS DE JONES para su diagnostico. • 72. Cual es el tratamiento de elección para la fiebre reumática? R = PENICILINA, ASA y HALOPERIDOL EN CASO DE COREA.
  • 93. LUPUS ERITEMATOSO SISTEMICO • 73. Que HLA tienen relación con el LES? 1) HLA B8, HLA DR2 y DR3. 2) Los alelos de HLA están en el brazo corto del CROMOSOMA 6. • 74. Que fármacos inducen la producción de anticuerpos antinucleares y LES?  R = HIDRALAZINA, procainamida, clorpromazina, ISONIAZIDA y diversos anticonvulsivantes. • 75. Cual es el cc del LES en cada aparato – sistema y con que padecimientos se relaciona?  1. Se relaciona con: PTI, ANEMIA HEMOLITICA Y TIROIDITIS.  2. Constitucionales: la FATIGA es lo mas común  3. Mucocutaneas: 80%, LESIONES DISCOIDES observándose en cara, pabellones auriculares y el AREA DE V DEL ESCOTE. Eritema malar o en ALAS DE MARIPOSA QUE ES FOTOSENSIBLE. EL neonatal QUE SE PRESENTA EN LOS HIJOS DE PX CON LES POR EL PASO DE IgG CARACTERIZÁNDOSE POR LESIONES CUTÁNEAS Y BLOQUEO CARDIACO CONGÉNITO EN LOS PRIMEROS 6 MESES DE EDAD. ALOPECIA.  4. Musculoesqueletico: ARTRALGIAS con artritis. Osteonecrosis en la cabeza femoral, humeral, cóndilos femorales, carpo, tarso manifestándose clínicamente con dolor que mejora con el reposo.  5. Renal: nefritis lupica siendo la mas común la TIPO IV ¨GLOMERULONEFRITIS PROLIFERATIVA DIFUSA¨.  6. SNC: Meningitis aséptica, EVC, síndromes desmielinizantes y cefaleas.  7. Cardiopulmonar: En corazón con VEGETACIONES ENDOCARDICAS de LIBMAN-SACKS, en pleura lo mas común es pleuritis con o sin derrame pleural con niveles altos de proteínas DHL, complejos inmunes, ANA y células LE. LA VALVULA MAS AFECTADA ES LA MITRAL.  8. Gastrointestinal: Se puede presentar con SEROSITIS, disfagia y dispepsia muy a menudo.  9. Hematológico: PANCITOPENIA Y >TTP. Anemia por deficiencia de hierro MICROCITICA- HIPOCROMICA.
  • 94. • 76. Cual es el inmunosupresor de elección para la nefritis lupica y cual es el de elección en caso de serositis.? 1) Nefritis lupica: CICLOFOSFAMIDA 2) Serositis: METOTREXATO • 77. Cuantos criterios se revisaron en 1997 para dx de lupus?  11 criterios entre los cuales están:  1. Eritema malar  2. Lupus discoide  3. Foto sensibilidad  4. Ulceras orales  5. Artritis no erosiva  6. Serositis  7. Afección renal  8. Afección neurológica  9. Afección hematológica  10. Afección inmunológica  11. ANA +  y se requieren de 4 criterios para el diagnostico. • 78. Que medicamentos han mejorado la supervivencia en pacientes con nefropatía lupica?  R = GLUCOCORTICOIDES + CICLOFOSFAMIDA
  • 95.
  • 96.
  • 97.
  • 98.
  • 99. CASO CLINICO • A22-year-old woman develops a red rash over her cheeks, and pain and swelling in both knees as well as several small joints in her hands. Medical evaluation reveals oral ulceration and 3+ proteinuria. Which of the following is the most sensitive test for the diagnosis of this condition? (A) LE cells (B) ANAs (C) anti-Sm (D) anti-Ro (E) antiphospholipid
  • 100. (B) ANAs are present in 98% of patients with SLE. Repeatedly negative tests make the diagnosis of SLE very unlikely. Unfortunately, the test is not specific and may be positive in normal people (especially in older individuals), or secondary to infections, drugs, or other autoimmune disorders.
  • 101. CASO CLINICO • A 25-year-old woman with SLE notices increasing fatigue and shortness of breath on exertion. Her usual SLE symptoms of joint discomfort, chest pain, and fevers are not present. On examination, there are no active joints, but she is jaundiced. Laboratory studies show hemoglobin 9 g/dL, WBC 5000/mL, platelets 150,000/mL, reticulocyte count 4%, direct antiglobulin test: positive, total bilirubin 4 g/dL, direct 0.5 mg/dL, aspartate amino transferase (AST) 20 U/L, alanine amino transferase (ALT) 15 U/L, LDH 300 U/L. Which of the following is the most likely diagnosis? (A) autoimmune hepatitis (B) hemolytic anemia (C) viral hepatitis (D) acute blood loss (E) Gilbert syndrome
  • 102. (B) This patient with SLE has developed autoimmune hemolytic anemia. Treatment consists of high-dose oral steroids. The positive Coombs’ test (direct antiglobulin test), high indirect bilirubin, high LDH, and high retic count are all supportive of hemolytic anemia. Anormal AST and ALT rule out hepatitis.
  • 103. CASO CLINICO • A30-year-old woman develops a rash over her cheeks, nose, and ears. She also has pain and swelling in her wrists as well as several small joints in her hands. Medical evaluation reveals oral ulceration and 3+ proteinuria. Her ANAis positive. Which of the following is the most likely cardiac manifestation of her disease? (A) pericarditis (B) myocarditis (C) aortic regurgitation (D) nonbacterial endocarditis (E) myocardial vasculitis with infarction
  • 104. (A) This patient has SLE and pericarditis, sometimes leading to tamponade, is the most common manifestation of cardiac disease. Myocarditis does occur and can cause arrhythmias, sudden death, or heart failure. Libman-Sacks endocarditis is associated with thrombotic events or, less commonly, valvular regurgitation. Myocardial infarction is more commonly a result of atherosclerotic disease than vasculitis.
  • 105. CASO CLINICO • A 24-year-old is referred for assessment of a low white cell count. She has no past medical history and is not on any medications. Her only symptoms are of joint discomfort in her hands, and occasional sharp chest pains that change with breathing. On examination, there is inflammation of some MCP and DIP joints in both hands, and the rest of the examination is normal. Her WBC is 3500/mL and the lymphocytes are low (15%) and PMNs are normal. Which of the following is the most likely diagnosis? (A) periarteritis nodosa (B) SLE (C) scleroderma (D) DM (E) osteoarthritis
  • 106. (B) Leukopenia occurs in almost two-thirds of the SLE patients, and the differential count is usually normal. Lymphocytes and platelets can also be reduced.
  • 107. CASO CLINICO • A 26-year-old woman develops a red rash over her cheeks and pain, and swelling in both wrists as well as several small joints in her hands. The rash gets worse on sun exposure and involves her cheeks, nose, ears, and chin. Medical evaluation reveals oral ulceration and 3+ proteinuria. Which of the following is the most specific test for diagnosis of this condition? (A) lupus erythematosus (LE) cells (B) antinuclear antibody (ANA) (C) anti-Sm antibody (D) anti-Ro antibody (E) antiphospholipid antibody
  • 108. • (C) Anti-Sm detects a protein complexed to six species of small nuclear ribonucleic acid (RNA). It is believed to be very specific for SLE. However, only 30% of patients have a positive test. In the case presented, there are enough clinical criteria (four) to confirm the diagnosis of SLE with 98% specificity and 97% sensitivity.
  • 109. CASO CLINICO • A young woman presents with a facial rash, arthralgias, and fatigue. The rash on her face is erythematous and raised, her heart and lungs are normal, and wrists are swollen and tender on palpation. She has mild thrombocytopenia (90,000/mL). Which of the following is the most appropriate initial autoantibody test? (A) anti-double-stranded (ds) deoxyribonucleic acid (DNA) (B) anti-Sm (C) anti-Ro or La (D) ANA (E) antiphospholipid antibodies (lupus anticoagulant)
  • 110. • (D) ANA is the most important diagnostic autoantibodies in patients being evaluated for SLE. It is rare to have ANA negative SLE. The other antibodies can occur in SLE but not as sensitive for the diagnosis as ANA. AntidsDNA and anti-SM are specific for SLE but not sensitive.
  • 111. CASO CLINICO • A34-year-old woman develops a red rash over her cheeks, frequent oral ulcers, and pain and swelling in both wrists as well as in severalsmall joints in her hands. Medical evaluation reveals a positive ANA, and 3+ proteinuria. Which of the following organ involvement will cause the most symptoms during the course of this disease? (A) renal pathology (B) cardiopulmonary pathology (C) musculoskeletal pathology (D) thrombotic events (E) skin changes
  • 112. (C) About 95% of patients will develop musculoskeletal symptoms during the course of SLE. Arthralgias and myalgias predominate, but arthritis, hand deformities, myopathy, and avascular necrosis of bone also occur. About 85% of patients will have hematologic disease and 80% will have skin manifestations.
  • 113. CASO CLINICO • A27-year-old woman presents with a red rash over her cheeks, and pain and swelling in both knees as well as several small joints in her hands. She notes that the rash is worse with sun exposure. Medical evaluation reveals oral ulceration, positive ANA, and 3+ proteinuria. Which of the following is the most likely mechanism for the renal damage in this condition? (A) vasculitis (B) microemboli (C) antibasement membrane antibodies (D) deposition of circulating immune complexes (E) primary tubular atrophy
  • 114. (D) Renal disease is usually secondary to deposition of circulating immune complex. Although most patients with SLE have such deposits, only half have clinical nephritis as defined by proteinuria. Renal biopsy can provide both prognostic and therapeutic information.
  • 115. ENFERMEDAD DE STILL • 79. Que caracteriza clínicamente a la enfermedad de Still?  R = VARIANTE DE LA ARTRITIS REUMATOIDE en la cual hay FIEBRE DE MAS DE 40 y con precipitación posterior varios grados debajo, odinofagia y poliadenopatias. LO PRECEDE FARINGITIS. • 80. Cual es una manifestación característica de la enfermedad de still?  R = EXANTEMA NO PRURIGINOSO, maculopapular de COLOR SALMÓN en TÓRAX Y ABDOMEN. • 81. Cual es el laboratorio distintivo de la enfermedad de Still?  R = LEUCOCITOSIS > 40, VSG AUMENTADA, hipergamaglobulinemia IgG y anemia. NO se detectan AUTOANTICUERPOS NI FR. • 82. Cual es el manejo de la enfermedad de Still?  R = AINES
  • 116. CASO CLINICO • An 18-year-old man has had fever for several weeks. The fever occurs on an almost daily basis and is associated with an evanescent salmon-colored truncal rash. He has diffuse arthralgias, and an extensive investigation for infections and malignancy is negative. Which of the following is diagnostic of this condition? (A) high-titer rheumatoid factor (RF) (B) positive ANA (C) response to steroid therapy (D) response to nonsteroidal antiinflammatory drug (NSAID) therapy (E) lymph node biopsy
  • 117. • (D) Still’s disease (juvenile RA) in an adult may present as fever of unknown origin. Unfortunately, RF is often negative, and a response to NSAIDs along with exclusion of other diseases confirms the diagnosis.
  • 118. ESCLEROSIS SISTEMICA O ESCLERODERMIA • 83. Como se define la enfermedad llamada esclerosis sistémica o esclerodermia?  R = Es una enfermedad que se caracteriza por la aparición de fibrosis difusa de la piel y órganos internos • 84. Que es el síndrome de CREST en esclerosis sistémica o esclerodermia?  R = Calcinosis cutánea, fenómeno de Raynaud, trastorno de la motilidad Esofágica, eSclerodactilia y Telangectasia • 85. Cuales son las variantes de esclerosis sistémica o esclerodermia y sus principales afecciones? 1) LIMITADA 80%: en la que hay ENDURECIMIENTO DE LA PIEL, manos o cara. 2) DIFUSA: afecta tronco y extremidades, muñecas, tobillos, rodillas, RIÑÓN (GLOMERULOESCLEROSIS DIFUSA) y corazón con CARDIOPATÍA RESTRICTIVA. La principal CAUSA DE MUERTE ES PULMONAR POR HIPERTENSIÓN O FIBROSIS. • 86. Cuales son las manifestaciones cutáneas de la esclerodermia?  R = La piel se ve brillante , con PERDIDAS DE PLIEGUES Y ATROFIA DE FOLÍCULOS PILOSOS, se siente ACARTONADA, la afección facial provoca una cara afilada. • 87. Cuales son las manifestaciones vasculares de la esclerodermia?  R = Fenómeno de Raynaud en dedos de manos, pies, nariz, oídos y lengua.
  • 119. • 88. Cuales son las manifestaciones pulmonares de la esclerodermia?  R = Enfermedad pulmonar INTERSTICIAL EN LA DIFUSA CON PATRÓN RESTRICTIVO. Hipertensión pulmonar y cor pulmonale. En la TAC se puede ver si hay actividad (imagen en vidrio despulido) o solo fibrosis (imagen en panal de abejas) • 89. Cuales son las manifestaciones GI de la esclerodermia?  R = La mayoría tiene AFECCIÓN ESOFÁGICA CARACTERIZADA POR ERGE, pirosis y dificultad para el paso de alimentos sólidos siendo de preferencia los líquidos. • 90. Cual es la manifestación renal de la esclerodermia?  R = Crisis renal esclerodermica (HAS MALIGNA) que conlleva a una IR rápidamente progresiva. • 91. Cuales son los laboratorios de esclerosis sistémica o esclerodermia?  R = ANA +, depende del órgano afectado, AC VS ESCLERODERMIA SCL-70. En la BIOPSIA aparece EXCESO DE DEPOSITO DE COLÁGENO EN PIEL, TCS Y VASOS SANGUÍNEOS que puede aparecer de manera lineal (esclerosis lineal) o en parches hipocromicos. DIFUSA: ANA y TOPOISOMERASA I, SX CREST: AC ANTICENTROMERO. • 92. Cual es el manejo de la esclerosis sistémica o esclerodermia?  R = D- PENICILAMINA. Para el FENÓMENO DE RAYNAUD se da NIFEDIPINO o losartan. Iloprost que es análogo de la prostaciclina para ulceras digitales. CICLOFOSFAMIDA, corticoides, SINDENAFIL para ENFERMEDAD INTERSTICIAL PULMONAR. PIEL (morfea del tronco hipocromica) luz UV, CORTICOIDES TÓPICOS, si se generaliza se da metotrexate o CORTICOIDES SISTÉMICOS. CALCINOSIS diltiazem o NIFEDIPINO. DAÑO RENAL IECAS. TELANGIECTASIAS ESCLEROSIS. GI PROCINETICOS Y ERITROMICINA por que ESTIMULA LA MOTILINA, NO CISAPRIDA (QUE PUEDE PRODUCIR SX QT LARGO, TAQUICARDIA VENTRICULAR Y DESENLACES FATALES). ARTRALGIAS acetaminofen, PERICARDITIS AINES, corticoides o CICLOFOSFAMIDA.
  • 120.
  • 121.
  • 122.
  • 123. CASO CLINICO • A39-year-old woman complains of developing painful pale fingers on cold exposure for the past 5 years. Recently, she has noticed swollen fingers and tight skin, which limit flexion and extension. She also has new abdominal symptoms that are bothersome. On examination, the skin on the fingers is smooth and shiny with associated edema. The rest of the examination is normal. Which part of the gastrointestinal (GI) tract is most frequently involved in this condition? (A) esophagus (B) stomach (C) duodenum (D) ileum (E) colon
  • 124. • (A) This patient has scleroderma and esophageal symptoms are present in more than 50% of patients. They are due to the reduced tone of the gastroesophageal sphincter and dilation of the distal esophagus. Gastric and small intestinal motility problems can also occur. Vascular ectasia in the GI tract can result in bleeding.
  • 125. CASO CLINICO • A22-year-old woman develops color change in her fingers with cold exposure. The fingers turn white, then blue, and finally red. Which of the following statements regarding this condition is incorrect? (A) may lead to gangrene of the fingers (B) may precede the onset of scleroderma (C) symptoms can be brought on by vibration or stress (D) pallor (white color) associated with coldness and numbness while rubor (red color) associated with pain and tingling (E) affects the sexes equally
  • 126. (E) Raynaud’s phenomenon may lead to gangrene of the fingers. It can be primary (Raynaud’s disease) or secondary to other diseases, especially scleroderma, in which it can be the presenting symptom. In women, the primary form is common (over 50%), and the phenomenon is generally much more frequent in women. Digital infarction is much more common in relationship to scleroderma than it is in primary Raynaud’s disease.
  • 127. ENFERMEDAD MIXTA DEL TEJIDO CONECTIVO
  • 128. MIOSITIS Y DERMATOMIOSITIS • 93. Que tipo de padecimientos son la miositis y dermatomiositis?  R = Etiologia desconocida y se caracterizan por DEBILIDAD MUSCULAR PROXIMAL GRADUAL Y PROGRESIVA. • 94. Que células predominan en la biopsia con DERMATOMIOSITIS?  R = Infiltrado de CÉLULAS B Y T CD4 • 95. Que células predominan en la biopsia con POLIMIOSITIS?  R = Predominan las células T CD8 A NIVEL ENDOMICIAL. • 96. Cuales son los datos clínicos de la miositis y dermatomiositis? 1) Dificultad para la deglución por afección de musculo estriado, EXANTEMA COLOR ROJO OBSCURO CARACTERÍSTICO CON DISTRIBUCIÓN MALAR PARECIDO A LES QUE PUEDE ESTAR EN CARA, CUELLO Y ESPALDA. 2) EDEMA PERIORBITARIO AMORATADO (heliotropo) SOBRE LOS PARPADOS. 3) Fiebre, DISMINUCIÓN DE LA FUERZA PROXIMAL y V del cuello.
  • 129. • 97. El SIGNO DE GOTTRON es patognomónico de miositis y dermatomiositis, que lo caracteriza?  R = ERITEMA PERIUNGUEAL, PLACAS ERITEMATO-ESCAMOSAS SOBRE DORSO DE LAS ARTICULACIONES INTERFALANGICAS Y METACARPOFALANGICAS. • 98. Según los criterios de Bohan y Peter como diferencias la dermatomiositis de la miositis-polimiositis?  R = 1) DERMATOMIOSITIS: 3 de 4 criterios MAS ERITEMA. 2) MIOSITIS: 4 criterios SIN ERITEMA. • 99. Cuales son los datos de laboratorio de miositis y dermatomiositis?  R = DHL Y CPK ELEVADA, AC ANTI RO-LA, AC ANTI JO y BIOPSIA MUSCULAR CON INFILTRADO LINFOCITARIO • 100. Cual es el manejo de la polimiositis y dermatomiositis?  R = PREDNISONA, la cual puede o no combinarse con inmunosupresores.
  • 130.
  • 131. CASO CLINICO • A 64-year-old woman presents with fatigue, and musculoskeletal symptoms. She reports of no headache, jaw discomfort, or visual disturbance. Her physical examination is normal, and laboratory testing reveals an elevated ESR of 75 mm/h. She is started on prednisone 10 mg/day and notices a dramatic improvement in her symptoms after 1 week. Which of the following are the most typical symptoms of this disorder? (A) heliotrope rash (B) proximal muscle weakness (C) painful peripheral neuropathies (D) stiffness and pain of proximal muscles (E) hematuria
  • 132. (D) This patient has PMR. It is characterized by stiffness, aching, and pain in proximal muscle groups in the neck, shoulders, back, hips, and thighs. It is considerably more common than temporal arteritis. Both diseases are almost exclusively seen in the over 50 age group.
  • 133. CASO CLINICO • A 64-year-old man is having difficulty getting out of a chair. He has lost 15 lb and feels tired all the time. On examination, there is a blue purple rash on his eyelids and knuckles, and muscle strength in his proximal muscles is rated 4 out of 5. His creatinine kinase (CK) level is elevated and he is started on prednisone. Which of the following is the most important in monitoring response to therapy? (A) testing of muscle strength (B) sedimentation rates (C) urine transaminase enzymes (D) EMG (E) alkaline phosphatase
  • 134. (A) The course of muscle necrosis in dermatomyositis can be best followed by repeated CK determinations. Repeated muscle biopsies are rarely required. However, the goal of therapy is to increase muscle strength and function, so following muscle strength is the key clinical assessment of response to therapy.
  • 135. CASO CLINICO • A 63-year-old man presents with weakness and hemoptysis, but no fever, cough, or sputum. He has a 60-pack-per-year history of smoking. The chest x-ray (CXR) reveals a lung mass with mediastinal widening. On examination, there is a blue purple discoloration of the upper eyelids and erythema on his knuckles. He has proximal muscle weakness rated 4+/5, normal reflexes, and sensation. Which of the following is the most likely diagnosis for his muscle weakness? (A) SLE (B) scleroderma (C) dermatomyositis (DM) (D) polyarteritis (E) Weber-Christian disease
  • 136. (C) This man has dermatomyositis, a paraneoplastic phenomenon of many cancers. The most common tumors associated with DM have been bronchogenic carcinomas, ovarian cancers, breast cancers, and melanoma but many others have occurred. The malignancy may antedate or postdate the myositis. Older age makes malignancy more likely. The extent of the workup for malignancy, if DM is the presentation, depends on clinical circumstances, but history and physical examination, not x-rays, are the cornerstones of evaluation.
  • 137. SX SJOGREN • 101. Cual es el cc del síndrome de Sjogren?  R = XEROSTOMIA, XEROFTALMIA Y ATROFIA PAPILAR LINGUAL. Los síntomas aumentan con la aplicación de ANTICOLINERGICOS, ANTIDEPRESIVOS Y DIURÉTICOS. • 102. Cuales son los datos de laboratorio de laboratorio sugerentes de Sx de Sjogren?  R = ANA 95% y ANTI RO-LA, SSA Y SSB. Prueba de SCHRIMER MENOR A 5 MM cuando lo normal es mayor a 10 después de 5 min. • 103. Cual es el manejo del Sx de Sjogren? 1) SALIVA Y LAGRIMAS ARTIFICIALES. 2) PILOCARPINA.
  • 138.
  • 139.
  • 140. CASO CLINICO • A 32-year-old woman develops symptoms secondary to a dry mouth and dry eyes. She has enlarged salivary glands. Studies forautoantibodies to Ro (SS-A) are positive. Asalivary gland biopsy reveals lymphocytic infiltration. Which of the following is the most likely diagnosis? (A) sarcoidosis (B) primary Sjögren’s syndrome (C) human immunodeficiency virus (HIV) infection (D) lymphoma (E) amyloidosis
  • 141. • (B) The sicca syndrome is a recognized feature of Sjögren’s syndrome. It can be primary or secondary to other autoimmune disorders such as RA, SLE, scleroderma, or vasculitis. Primary Sjögren’s syndrome is most common in middle-aged women; sicca symptoms can also occur as a complication of HIV infection or in sarcoidosis. Sjögren’s syndrome is more likely to have positive serology, while the serology in sarcoid or HIV is negative. Both HIV and Sjögren can have lymphocytic infiltration, but in HIV it is predominantly by CD8+ lymphocytes, whereas in Sjögren’s syndrome, the infiltration is by CD4+ lymphocytes. In sarcoidosis, biopsy reveals granulomas.
  • 142. RABDOMIOLISIS • 104. En que consiste la rabdomiolisis? R = Consiste en la NECROSIS de musculo esquelético, relacionado comúnmente a traumatismo. • 105. Por que se produce IRA en rabdomiolisis? R = Daño tubular por filtración de MIOGLOBINA • 106. Cual es el manejo de la rabdomiolisis? R = AUMENTO DE LIQUIDOS de 4-6 litros y vigilar función renal.
  • 143. POLIARTERITIS NODOSA • 107. Cuales son los signos y síntomas de poliarteritis nodosa?  R = DOLOR EN EXTREMIDAD POR ARTRALGIA, MIALGIA QUE AFECTA PANTORRILLAS, puede afectar riñón causando HAS, IRA O HEMORRAGIA POR MICROANEURISMAS. MONONEURITIS MÚLTIPLE (PIE CAÍDO), corazón con IAM. Livides reticular, nódulos subcutáneos, y ulceras en piel. Son necesarios 3 de 10 criterios para su diagnostico. • 108. Cuales son las pruebas de laboratorio en la poliarteritis nodosa y con que otro virus se relaciona habitualmente?  R = DESCARTAR VHB QUE SE ASOCIA EN 20-30%. ANCA (-) Y ANA (-) .Anemia, leucocitosis y trombocitosis. Se confirma diagnostico con toma de BIOPSIA DE LA LESIÓN. • 109. Que datos se obtienen al tomar la biopsia en la poliarteritis nodosa?  R = BIOPSIA DE ARTERIA de mediano o pequeño calibre con PRESENCIA DE INFILTRADO GRANULOCITICO, PMN Y CÉLULAS MONONUCLEARES EN LA PARED DEL VASO. • 110. Cual es el manejo de la poliarteritis nodosa?  R = ESTEROIDES, pero SI SE ASOCIA CON VHB SE AGREGAN RETROVIRALES.
  • 144.
  • 145. CASO CLINICO • A 39-year-old man has had several weeks of fever, abdominal pain, weight loss, and lack of energy. Three days prior to assessment, he developed a left foot drop. Physical examination confirms left peroneal nerve damage and a bilateral sensory peripheral neuropathy in both legs. Laboratory evaluation reveals ESR of 105 mm/h, WBC of 14,000/mL, and a negative serologic test for ANCA. The eosinophil count is normal. Which of the following is a reasonable method of establishing a diagnosis? (A) testicular biopsy (B) skin biopsy (C) spiral computerized tomography (CT) of chest (D) further serologic testing (E) abdominal angiography
  • 146. (E) This patient likely has PAN. ANCA and other serology are usually negative in PAN; positive serology suggests another diagnosis. The optimal diagnostic strategy is the biopsy of an affected organ. However angiography to look for aneurysms of small- and medium sized arteries generally has higher yield than blind biopsy of unaffected organs. The lungs are not a characteristic site of involvement.
  • 147. CASO CLINICO • A 45-year-old man has had several weeks of fever, abdominal pain, weight loss, and lack of energy. Three days prior to assessment, he developed a left foot drop. His blood pressure is 160/90 mm Hg, pulse 80/min, and physical examination confirms left peroneal nerve damage and a bilateral sensory peripheral neuropathy in both legs. There are no skin rashes. Laboratory evaluation reveals ESR of 105 mm/h, WBC of 14,000/mL, and negative serologic tests for ANCA and ANA. Eosinophil count is normal. Which of the following chronic viral infections is sometimes associated with this? (A) high cytomegalovirus (CMV) titers (B) herpesvirus material in circulating immune complexes (C) hepatitis B material in circulating immune complexes (D) epidemiologic relationship to Coxsackie B virus (E) triggering of symptoms following viral gastroenteritis secondary to rotavirus infection
  • 148. (C) About 20–30% of patients with PAN have hepatitis B antigenemia. Circulating immune complexes containing hepatitis B antigen and immunoglobulin have been detected, and immunofluorescence of blood vessel walls have also demonstrated hepatitis B antigen. Antiviral therapy has been used in these cases.
  • 149. CASO CLINICO • A 42-year-old woman has had several weeks of fever, abdominal pain, weight loss, and lack of energy. Three days prior to assessment, she developed a left foot drop and rash on her legs. Her blood pressure is 160/90 mm Hg, pulse 80/min, and physical examination confirms left peroneal nerve damage and a bilateral sensory peripheral neuropathy in both legs. The rash looks like livedo reticularis. Laboratory evaluation reveals ESR of 105 mm/h, WBC of 14,000/mL, and negative serologic tests for ANCA and ANA. Eosinophil count is normal, and urinalysis is negative for casts, protein, and red cells. Biopsy of the skin rash shows inflammation of the small blood vessels. Which of the following is the most appropriate next step in management? (A) plasmapheresis (B) steroid therapy alone (C) combination therapy with steroids and cyclophosphamide (D) cyclophosphamide therapy alone (E) combination therapy with steroids and methotrexate
  • 150. (C) Current treatment for polyarteritis nodosa (PAN) mimics that of Wegener’s granulomatosis in the initial treatment with combination steroid and cyclophosphamide therapy. This will result in up to a 90% long-term remission rate even after discontinuation of therapy. In cases associated with hepatitis B infection, plasmapheresis is sometimes used as initial therapy.
  • 151. CASO CLINICO • A 55-year-old woman has had several weeks of fever, abdominal pain, weight loss, and lack of energy. Three days prior to the assessment, she developed a left foot drop. Her blood pressure is 160/90 mm Hg, pulse 80/min, and physical examination confirms left peroneal nerve damage and a bilateral sensory peripheral neuropathy in both legs. There are no skin rashes. Laboratory evaluation reveals ESR of 105 mm/h, WBC of 14,000/mL, and negative serologic tests for antineutrophil cytoplasmic antibody (ANCA) and ANA. The eosinophil count is normal, and urinalysis is negative for casts, protein, and red cells. Which of the following is the most likely mechanism for renal injury in this condition? (A) nephrotic syndrome (B) diffuse glomerulonephritis (C) granuloma formation (D) necrotizing vasculitis of vessels (E) exclusively small vessel involvement
  • 152. (D) This patient has polyarteritis nodosa (PAN) and in classic PAN, unlike microscopic polyangiitis, both small and medium vessels are involved. The renal lesions are ischemic secondary to fibrinoid necrosis of the vessels. In microscopic polyangiitis, a diffuse glomerulonephritis is frequently present. The most common organ systems involved are the kidneys, musculoskeletal system, and peripheral nervous system.
  • 153. POLIMIALGIA REUMATICA/ ARTERITIS DE CELULAS GIGANTES • 111. Cual es la diferencia entre polimialgia reumática y arteritis de células gigantes?  R = LA POLIMIALGIA REUMÁTICA SOLA NO PRODUCE CEGUERA y responde a la terapéutico con prednisona y la ARTERITIS DE CÉLULAS GIGANTES OCASIONA CEGUERA y necesita mayores dosis de prednisona. • 112. Cuales son los datos clínicos de polimialgia reumática?  R = DOLOR Y RIGIDEZ DE HOMBRO y región de la cintura acompañado de fiebre y disminución de peso. • 113. Cuales son los datos clínicos de arteritis de células gigantes?  R = Cefalea, SENSIBILIDAD DE CUERO CABELLUDO, síntomas visuales, CLAUDICACIÓN MANDIBULAR. • 114. Cuales son los datos de la biopsia y USG en ACG de la arteria temporal?  R = INFILTRADO en túnica ½ y adventicia con linfocitos y células plasmáticas. USG CON SIGNO DEL HALO con sensibilidad 62% y especificidad 82%
  • 154. • 115. Que dato se encuentra en la biopsia de la arteria temporal en ACG?  R = Fragmentación de la lamina elástica e infiltrado por linfocitos y macrófagos, células gigantes en la lamina elástica interna. • 116. Cual es el manejo de la polimialgia reumática?  R = PREDNISONA POR 1ª • 117. Cual es el manejo de la ACG?  R = PREDNISONA Y PREVENCIÓN DE CEGUERA PERMANENTE
  • 155.
  • 156. CASO CLINICO • A 74-year-old man presents with a history of increasing frequency of headaches, fatigue, and weight loss for 3 months. He has had migraine headaches in the past, but these are differentfrom them. He is also experiencing back, shoulder, and hip discomfort, which is worse in the morning. His head and neck examination is normal. Range of motion in the shoulders and hips is reduced because of discomfort but there is no active inflammation. Which of the following signs or symptoms is most helpful in the diagnosis? (A) throat pain on swallowing (B) pain in the jaw when chewing (C) malaise (D) fatigue (E) sweating
  • 157. (B) Although malaise, fatigue, and sweating are common in temporal arteritis, they are too nonspecific to help in making the diagnosis. Claudication of the jaw and tongue, while not very sensitive for temporal arteritis, are more specific than the constitutional symptoms. Odynophagia is not a characteristic of this disease.
  • 158. CASO CLINICO • A 69-year-old man comes to the office complaining of fatigue, and weight loss for 3 months. He also reports having frequent headaches, which are new for him. There are no other constitutional symptoms of fever, chills or night sweats. He does have chronic lower back pain but lately he has noticed pain in his shoulder, and hip as well. The muscle and joint symptoms are worse in the morning, and the stiffness lasts for 1 hour. His jaw also hurts when he is chewing food. Head and neck examination is normal, there is no lymphadenopathy and fundoscopy is normal. Range of motion in theshoulders and hips is reduced because of discomfort but there is no active inflammation. Which of the following is the most feared complication in patients with this condition? (A) blindness (B) cortical stroke (C) limb claudication (D) renal infarction (E) aortic aneurysm
  • 159. (A) Although all these complications have been reported in giant cell arteritis, the only one with a significant likelihood is blindness secondary to ischemic optic neuropathy. Thus, if the disease is suspected, urgent diagnosis and treatment is required.
  • 160. CASO CLINICO • A 67-year-old man complains of frequent headaches that are new for him. They are usually not very severe, and relieved with acetaminophen. He also has some back, shoulder, and hip discomfort, which is worse in themorning as well he feels quite fatigued and does not have his usual energy level. On examination, his neck is supple to flexion, fundi and thyroid examination are normal. Range of motion in the shoulders and hips is reduced because of discomfort but there is no active inflammation. There are no focal deficits on screening neurologic examination. Which of the following is the most appropriate initial diagnostic test? (A) immunoelectrophoresis (B) c-ANCA levels (C) ESR (D) creatine phosphokinase (CPK) (E) hemoglobin and red cell indices
  • 161. (C) Almost all patients with temporal arteritis will have an elevated ESR. Although a high ESR cannot make the diagnosis, a normal ESR helps in excluding the diagnosis. C-ANCA is a diagnostic tool for Wegener’s granulomatosis. Elevated CPK is not seen in temporal arteritis, even with associated PMR. Normochromic, or slightly hypochromic, anemia often seen in temporal arteritis is too nonspecific to be of much diagnostic help.
  • 162. CASO CLINICO • A 57-year-old woman is complaining of frequent headaches and scalp tenderness. She also has arthralgias, fatigue, and discomfort in her jaw when she chews. On examination, her head and neck is normal, but the right temporal artery is tender on palpation. Her erythrocyte sedimentation rate (ESR) is 50 mm/h and hemoglobin 10.5 g/dL. Atemporal artery biopsy is obtained for diagnostic confirmation. Which of the following is the most appropriate next step in management? (A) intravenous high-dose steroids (B) acetylsalicylic acid (C) indomethacin (D) low-dose (prednisone 40 mg/day) steroids by mouth (E) topical steroid creams
  • 163. • (D) The response of pain, stiffness, and headaches to 40–60 mg of prednisone is dramatic in giant cell arteritis. The duration of treatment is not known but most patients require treatment for more than 2 years. ESR is used to monitor response to therapy. Patients need treatment and evaluation for the complications of long-term steroid use such as osteoporosis and diabetes.
  • 164. CASO CLINICO • A 75-year-old woman has abrupt onset of soreness, and severe stiffness of the shoulders and upper thighs with low grade fever. Physical examination is entirely normal, but ESR is over 100 mm/h. Which of the following is the most likely diagnosis? (A) dermatomyositis (DM) (B) osteoarthritis (C) polymyalgia rheumatica (PMR) (D) midline granuloma (E) sarcoidosis
  • 165. • (C) This patient has PMR. Proximal arm and hip muscle/joint discomfort is the hallmark of this disorder. Difficulty in getting out of bed or rising from a chair may suggest polymyositis, but the muscles are normal when muscle strength is assessed. In general, PMR causes painful muscles, not weak muscles. However, pain may lead to profound disuse atrophy and apparent muscle weakness. In these cases, normal CK and nonspecific muscle biopsy still allow accurate differentiation from polymyositis.
  • 166. CASO CLINICO • A71-year-old woman comes to the office with a history of headaches, fatigue, and weight loss for 3 months. The headaches are new for her, and usually not very severe. Her jaw also hurts when she is chewing food. Two days prior, she had briefly lost partial vision in her left eye. There were no other neurologic symptoms at the time. On examination, her neck is supple to flexion, fundi and neurologic examinations are normal. She is started on prednisone 60 mg/day and a biopsy is performed to confirm the diagnosis. Which of the following is the most likely change seen on the biopsy to confirm the diagnosis? (A) immune complex deposition (B) arteritis with giant cells (C) lymphocytic infiltration (D) type II muscle fiber atrophy (E) polyphasic potentials on electromyography (EMG)
  • 167. (B) Temporal artery biopsy is required for definitive diagnosis of giant cell arteritis, because of the relatively nonspecific nature of the presenting symptoms, signs, and routine laboratory tests. The arteritis can be segmental, however, and great care must be taken in the pathologic assessment.
  • 168. GRANULOMATOSIS DE WEGENER • 118. Cuales son las vasculitis asociadas a ANCA?  R = Granulomatosis de WEGENER, SX DE CHURG-STRAUSS y POLIANGEITIS MICROSCÓPICA. • 119. En que consiste la granulomatosis de Wegener?  R = VASCULITIS de arterias pequeñas, LESIONES GRANULOMATOSAS NECROSANTES de VÍAS AÉREAS SUPERIORES E INFERIORES Y GLOMERULONEFRITIS. Se sospecha en personas con AFECCIONES RESPIRATORIAS RECURRENTES. • 120. Cuales son los datos clínicos de la granulomatosis de Wegener?  R = RINITIS crónica o SINUSITIS CRÓNICA. PULMONAR: 96% con ESPUTO SANGUINOLENTO, dolor torácico y disnea. SÍNTOMAS DE VÍAS AÉREAS SUPERIORES E INFERIORES CON REBELDÍA AL TRATAMIENTO 90%. Afección RENAL: con GLOMERULOESCLEROSIS FOCAL Y SEGMENTARIA que puede evolucionar a IR rápidamente progresiva. • 121. Cuales son los datos de gabinete de la granulomatosis de Wegener?  R = ANCAc. ANCAp .Biopsia renal con GLOMERULONEFRITIS FOCAL Y SEGMENTARIA NECROSANTE. TAC de tórax con infiltrados, nódulos, granulomas y calcificaciones. Histologicamente vasculitis, inflamación granulomatosa, necrosis. • 122. Cual es el manejo de granulomatosis de Wegener?  R = CICLOFOSFAMIDA, prednisona, metotrexate.
  • 169. CASO CLINICO A 28-year-old woman presents with a recent episode of coughing up some blood, frequent nosebleeds, and now decreased urine output. A nasal mucosa ulcer was seen on inspection. Her urinalysis is positive for protein and red cells consistent with a GN. The CXR shows two cavitary lesions and her serology is positive for antineutrophil cytoplasmic antibodies (ANCA). Which of the following is the most likely diagnosis? (A) Wegener’s granulomatosis (B) bacterial endocarditis (C) Goodpasture’s syndrome (D) lupus erythematosus (E) poststreptococcal disease
  • 170. (A) Numerous diseases are associated with renal and pulmonary manifestations, including lupus, Goodpasture’s syndrome, and Wegener’s granulomatosis. Wegener’s is typically associated with antineutrophil cytoplasmic antibodies. (Kasper, p. 2005)
  • 171. SX DE CHURG STRAUSS • 123. Como se define el síndrome de Churg-Strauss?  R = Como una VASCULITIS necrosante sistémica y tiene una gran ASOCIACIÓN CON ASMA 98-100% • 124. Cual es el cc del síndrome de Churg-Strauss?  FASE I: ASMA o RINITIS alérgica con meses o años de duración.  FASE II: INFILTRACIÓN TISULAR EOSINOFILICA con cuenta arriba de 1000 por ml.  FASE III: Vasculitis sistémica que afecta corazón, pulmones, piel, nervios periféricos • 125. Cuales son los datos de gabinete y biopsia en el síndrome de Churg-Strauss?  R = ANCAp Y EOSINOFILIA. Biopsia con GRANULOMA EOSINOFILO extravascular con zona de necrosis central rodeada de infiltrado celular mixto. • 126. Cual es el manejo del síndrome de Churg-Strauss?  R = El tratamiento para el síndrome de Churg-Strauss incluye GLUCOCORTICOIDES como la prednisolona y otras drogas inmunosupresoras como la azatioprina y ciclofosfamida. La enfermedad es crónica y de por vida.
  • 172. POLIANGEITIS MICROSCOPICA • 127. Cuales son los datos clínicos de la poliangeitis microscópica?  R = RIÑÓN con GLOMERULONEFRITIS de rápida progresión y PULMÓN con HEMORRAGIA difusa. • 128. Que se observa en la biopsia renal en un paciente con poliangeitis microscópica?  R = Demuestra GLOMERULONEFRITIS NECROSANTE FOCAL Y SEGMENTARIA pauciinmune, con PROLIFERACIÓN CAPILAR EN FORMA DE MEDIAS LUNAS. • 129. Que datos radiológicos y sintomáticos tiene un paciente a nivel pulmonar en un paciente con poliangeitis microscópica?  R = La HEMORRAGIA PULMONAR se documenta al observar un infiltrado o sombras alveolares en un paciente con HEMOPTISIS, disnea (hipoxemia) y anemia en ausencia de edema pulmonar o infección. • 130. Cual es el diagnostico diferencial en un paciente con poliangeitis microscópica?  R = SX DE GOODPASTURE que también presenta un síndrome pulmón-riñón pero NO PRESENTA VASCULITIS NI AFECCIÓN MULTISISTEMICA. • 131 Cual es el manejo de la poliangeitis microscópica?  R = GLUCOCORTICOIDES Y CICLOFOSFAMIDA. Sin tratamiento la mortalidad a 5 años es del 85%.
  • 173.
  • 174. PURPURA DE HENOCH SCHONLEIN • 132. En que consiste la purpura de Henoch Schonlein?  R = Vasculitis sistémica de pequeño calibre mas común en niños • 133. Cuales son los rasgos típicos de la purpura de Henoch Schonlein? 1) ANTECEDENTE DE ENFERMEDAD VIRAL. 2) TRIADA CLÁSICA DE PURPURA, DOLOR ABDOMINAL TIPO CÓLICO Y ARTRITIS. 3) Purpura palpable NO TROMBOPENICA en región glútea y miembros pelvicos, dolor abdominal, artritis, hematuria. 4) Lesiones en la piel de forma clásica en extremidades inferiores pero pueden manifestarse en cualquier zona. 5) GI: Dolor tipo cólico, nausea y vomito. 6) RENAL: Glomerulonefritis o síndrome nefrótico. • 134. Cuales son lo hallazgos patológicos en la biopsia y gabinete de la purpura de Henoch Schonlein?  R = Depósitos de IgA EN BIOPSIA y análisis sanguíneo SIN TROMBOPENIA ni alteración de la coagulación.
  • 175.
  • 176. SÍNDROME DE BECHET • 135. Que caracteriza al síndrome de Bechet?  1) ULCERA ORAL, ENCIA Y LABIOS: Ulcera dolorosa redonda (aftosa), borde eritematoso y cubierta por pseudomembrana amarillento.  2) ULCERAS GENITALES: Pene, escroto o vulva similares a las orales.  3) OFTALMICAS: Hipopion, epiescleritis, vasculitis retineana e UVEÍTIS con dolor, ojo rojo y visión borrosa. • 136. Que pruebas diagnostica existe para el diagnostico de síndrome de Bechet?  R = PATERGIA + que consiste en puncionar con una aguja estéril el antebrazo del paciente y si aparece un nódulo eritematoso aséptico o una pústula mayor a 2 mm es positivo. • 137. Cual es el manejo del síndrome de Bechet? 1) GLUCOCORTICOIDES. 2) TALIDOMIDApara ULCERAS ORAL Y VAGINAL
  • 177.
  • 178. ARTERITIS DE TAKAYASU • 138. Que tipo de alteración es la arteritis de Takayasu?  R = Es una vasculitis que afecta a la aorta y sus ramas así como a las arterias pulmonares produciendo síntomas isquémicos principalmente. • 139. Cuales son los datos clínicos de la arteritis de Takayasu? 1) SISTEMICOS: FIEBRE, pérdida de peso, astenia, artralgias. 2) Manifestaciones isquémicas como frialdad de una o mas extremidades, cefalea, amaurosis, diplopía y la PRESIÓN EN AMBOS BRAZOS DIFIERE 10 MM/HG. • 140. Cuales son los datos de laboratorio y gabinete principales en la arteritis de Takayasu? 1) LABORATORIO: VSG AUMENTADA, trombocitosis, ANEMIA NORMO-NORMO. 2) ANGIOGRAFIA: estrechamientos con adelgazamientos progresivos, oclusiones y aneurismas. • 141. Cual es el manejo de la arteritis de Takayasu? 1) PREDNISONA 2) ANGIOPLASTIA
  • 179.
  • 180. CASO CLINICO • A 24-year-old woman notices pain in her left arm, made worse with use. She also has fatigue, fever, night sweats, and arthralgias. On examination, there are no palpable lymph nodes, and the joints and muscle strength are normal. The left radial pulse is absent, and there is a bruit over the left subclavian and common carotid arteries. Preliminary laboratory investigations reveal an elevated ESR and mild anemia. Which of the following vascular findings is most likely to be found in her? (A) high pressure in the legs and low pressure in the arms (B) low pressure in the legs and high pressure in the arms (C) high-pitched diastolic murmur (D) a relentless course to death (E) hypotension
  • 181. (A) High pressure in the legs and low pressure in the arms characterize Takayasu’s syndrome. Clinical manifestations include easy fatigability of the arms and atrophy of the soft tissues of the face. The course is variable, and spontaneous remissions can occur. The disease predominantly affects young women.
  • 182. ENFERMEDAD DE KAWASAKY • 142. Cual es el cc de la enfermedad de Kawasaky? R = Enfermedad FEBRIL generalmente EN <5ª y se caracteriza por eritema en mucosa oral, EROSIONES EN LOS LABIOS, LENGUA EN FRESA, vasculitis coronaria. • 143. Cual es el manejo de la enfermedad de Kawasaky? R = GAMAGLOBULINA IV + ASA por 14 días.
  • 183.
  • 184. OSTEOMIELITIS TB O MAL DE POTT • 145. En un paciente con ANEMIA DE CÉLULAS FALCIFORMES, que agente causal es mas común de provocar osteomielitis?  R = Salmonella • 146. Cual es el tratamiento de elección para todas las lesiones óseas causadas por coccidioides inmitis?  R = ITRACONAZOL • 147. Cual es el cc de la osteomielitis tuberculosa?  R = DOLOR DORSAL Y LUMBAR con dificultad para sentarse, ponerse de pie o flexionar el cuerpo, con dolor a la palpación y contracción muscular protectora, se aprecia malestar general crónico y manifestaciones pulmonares de tb. • 148. Cuales son los hallazgos radiológicos de la osteomielitis tuberculosa o mal de Pott?  R = LESIONES OSTEOLITICAS EN LA COLUMNA, ASÍ COMO PERDIDA DE LA ALTURA DEL ESPACIO INTERVERTEBRAL.
  • 185. • 149. Cual es la complicación mas grave de la osteomielitis tb en la columna vertebral (espondilitis tuberculosa o enfermedad de Pott)?  R = Paraplejia • 150. Cual es el régimen de tx en la osteomielitis tuberculosa?  R = I, R Y P durante 6 MESES • 151. Cual es el cc de la artritis tuberculosa? • R = La artritis periférica se manifiesta como MONOARTRITIS EN MAS DEL 90% de los casos, siendo las ARTICULACIONES GRANDES COMO LA RODILLA O LA CADERA LAS MAS AFECTADAS. El dolor o limitación funcional son síntomas tardíos. • 152. Cual es la triada clásica de Phemister radiológico en la osteomelitis tuberculosa o mal de Pott? • R = OSTEOPENIA PERIARTICULAR, EROSIONES ARTICULARES PERIFÉRICAS Y DISMINUCIÓN GRADUAL DEL ESPACIO ARTICULAR. El diagnostico definitivo se realiza mediante la biopsia de tejido sinovial, con cultivos y tinciones demostrando el bacilo.
  • 186.
  • 187. OSTEOSARCOMA • 153. Cuales son los tumores benignos de los huesos y cual es su tratamiento de elección?  R = OSTEOMAS OSTEOIDES y el tratamiento de elección es la FOTOCOAGULACIÓN CON LASER • 154. Que es una atropatia neurogena o articulación de Charcot?  R = Consiste en la DESTRUCCIÓN ARTICULAR resultante de la PERDIDA o disminución en la PROPIOSEPCION DEL DOLOR Y TEMPERATURA. LA INYECCIÓN INTRARTICULAR DE HIDROCORTISONA PUEDE OCASIONARLO • 155. Que lugar ocupa en frecuencia el osteosarcoma?  R = 2do lugar después del mieloma y es maligno, ataca principalmente a niños, adolescentes y adultos jóvenes CRECIENDO GENERALMENTE EN ZONAS DE DESARROLLO EPIFISIARIO. • 156. Cual es el cc del osteosarcoma?  R = Es una neoplasia que crece con rapidez y altamente destructiva metastatiza al pulmón rápidamente. El síntoma principal es el DOLOR con afección articular, AUMENTO DE LA TEMPERATURA LOCAL. • 157. Cuales son los hallazgos de gabinete en el osteosarcoma?  R = La FA esta aumentada. La RM es de elección. En la radiografía simple presenta una IMAGEN EN ¨SOL NACIENTE¨. • 158. Cual es el manejo del osteosarcoma?  R = QUIMIOTERAPIA neoadyuvante mas RESECCIÓN con supervivencia de un 90%
  • 188. TUMOR DE EWING • 159. Donde se localiza frecuentemente el tumor de Ewing? 1) El sarcoma de Ewing es una neoplasia maligna de crecimiento rápido, que NACE DE LAS CÉLULAS PRIMITIVAS DE LA MEDULA ÓSEA, normalmente en la cavidad medular de HUESOS LARGOS. 2) Inicia creciendo en el interior de la cavidad medular, perforando el hueso cortical de la diáfisis y elevando el periostio que da el signo RADIOGRÁFICO EN CAPAS DE CEBOLLA. • 160. Cual es el cc del tumor de Ewing?  R = DOLOR, SENSIBILIDAD LOCAL, manifestaciones sistémicas. DEBIDO AL CENTRO NECRÓTICO HAY FEBRÍCULA, LEUCOCITOSIS y elevación de la VSG. • 161. Cual es el pronóstico del tumor de Ewing? 1) MALO CON UNA MORTALIDAD DEL 95% pese al tratamiento debido a METÁSTASIS PULMONARES. 2) El tratamiento temprano reduce la mortalidad a 50% y se hace con quimio y radioterapia.
  • 189. OSTEOMIELITIS AGUDA • 162. Cual es el microorganismo mas frecuente de la osteomielitis aguda? 1) STAPHYLOCOCCUS AUREUS, la puerta de entrada suele ser la piel a través de erosiones, abrasiones, pústulas o quemaduras. 2) En LACTANTES los agentes mas frecuentes son los ESTREPTOCOCOS (NEUMOCOCO) O HAEMOPHILUS INLUENZAE. • 163. Cual es el cc característico de la osteomielitis aguda?  R = Existe el ANTECEDENTE DE TRAUMA O INFECCIÓN recurrente de VÍAS RESPIRATORIAS O PIEL. El primer síntoma es el dolor, acompañado de anorexia y FIEBRE. La TUMEFACCIÓN de partes blandas es un signo tardío. • 164. Que estudios de gabinete son útiles para el diagnostico de osteomielitis aguda?  R = La radiografía demuestran cambios solo en etapas tardías, pero el CAMBIO MAS PRECOZ DE ESTA ES UNA REACCIÓN PERIOSTICA. LA RESONANCIA Y LA GAMAGRAFIA SON MAS ESPECIFICAS DE MANERA TEMPRANA. • 165. Cual es el estándar de oro para el dx de osteomielitis aguda?  R = LA BIOPSIA DE HUESO, a la que se le realiza CULTIVO CON ANTIBIOGRAMA. • 166. Cual es el manejo de la osteomielitis aguda?  R = Manejo empírico con PENICILINA G Y CLINDAMICINA DE 4-6 SEMANAS y ESCARIFICACIÓN para aumentar el flujo sanguíneo y promover la formación de vasos nuevos en el hueso.
  • 190. OSTEOMIELITIS CRONICA • 167. Cual es el cc de la osteomielitis crónica?  R = EL paciente PERSISTE CON UNA LESIÓN RESIDUAL DOLOROSA después de la resolución de la fase aguda, acompañada de tumefacción, dolor a la palpación e impotencia. • 168. Cuales son los hallazgos de gabinete en la osteomielitis crónica?  R = Se encuentra un SECUESTRO CLARO EN EL HUESO QUE SE OBSERVAN COMO CAVIDADES RADIOLUCIDAS CON TEJIDO DE GRANULACIÓN EN SU INTERIOR, con aumento de volumen de tejidos blandos y posteriormente OSTEOPENIA y LISIS ÓSEA POR DESTRUCCIÓN TRABECULAR. • 169. Cual es el manejo de la ostemielitis crónica?  R = RETIRAR TODO EL HUESO NECRÓTICO, y antibióticos partiendo de los resultados de la biopsia de hueso.
  • 191. ARTRITIS SEPTICA PIOGENA • 171. Cuales son los factores de riesgo para la artritis séptica piógena (gonocócica o no)?  R = Abuso de drogas IV, jóvenes sexualmente activos, enfermedades concomitante como DM, AR o lupus, antecedentes de cirugía o prótesis. • 172. Cual es el cc de la artritis séptica piógena gonocócica o no?  R = Es una entidad de inicio agudo y progresivo caracterizado por dolor e inflamación local (salvo en el primer año de vida donde es similar a la sepsis). El dolor es moderado a intenso con limitación funcional, aumento de la temperatura local y rubor es monoarticular, cuando es poliarticular es en paciente inmunocomprometido siendo la rodilla la mas afectada. • 173. Cuales son los hallazgos radiológicos en la artritis séptica piógena?  R = Edema de tejidos blandos y derrame articular. EN AFECCIÓN POR ANAEROBIOS SE VE GAS EN TEJIDOS BLANDOS. El gamagrama óseo con tecnecio (valora flujo) o con galio (valora actividad inflamatoria) pueden utilizarse de manera temprana. • 174. Cual es el manejo de la artritis séptica piógena? 1) Consiste en la administración de antibióticos y drenaje de la articulación afectada. En caso de ARTRITIS PIÓGENA debe administrarse de 3 A 4 SEMANAS y en caso de ARTRITIS GONOCÓCICA DE 7 A 14 DÍAS. 2) En menores de 2M: DICLOXACILINA mas CEFOTAXIMA 3) En LACTANTES: DICLOXACILINA mas CEFOTAXIMA 4) En niños MAYORES DE 3ª: DICLOXACILINA 5) Artritis séptica GONOCÓCICA: 1g por via IM o IV cada 24 hrs. 6) Artritis séptica en DROGADICTOS por via IV: DICLOXACILINA mas GENTAMICINA 7) Artritis séptica en INMUNOCOMPROMETIDOS: CEFTAZIDIMA mas AMIKACINA 8) Artritis séptica POSTOPERATORIA: VANCOMICINA mas AMIKACINA 9) Artitis séptica en FRACTURA ABIERTA: AMOXICILINA con ACIDO CLAVULANICA
  • 192.
  • 193. RESUMEN DE LAS ENFERMEDADES METABOLICAS OSEAS
  • 194.
  • 195. ENFERMEDAD DE PAGET • Figure shows an x-ray of a 40-year-old Caucasian man with symptoms of sinusitis and an incidental finding in the skull. Which of the following is the most likely diagnosis? (A) normal variant (B) osteomyelitis (C) Paget’s disease (D) hemangioma (E) metastatic disease
  • 196. • (C) This is a rarefied area involving the frontal and parietal bones, and it is an early stage of Paget’s disease in which calvarial thickening and foci or radiopacity are not present within the radiolucent area. At this stage of the disease, a cross-section through the margin of the lesion reveals a compact inner and outer table in the normal portion, whereas the diploe widens and extends to the outer and inner surfaces of the calvarium without a change in the calvarial thickness in the lesion. • El tratamiento incluye AINES, bifosfonatos y calcitonina.
  • 197. CPK • Las siguientes patologías cursan con aumento de la CPK 1) HIPOTIROIDISMO 2) RAZA NEGRA 3) POLIMIOSITIS Y 4) DERMATOMIOSITIS
  • 198. Consejos 1. Leer diariamente y en bloques 2. Adiós Partys un tiempo 3. Has ejercicio y come bien durante el estudio 4. Toma algún curso bueno si tienes la posibilidad 5. Ten Fe.
  • 199. BIBLIOGRAFIA • EXARMED • PAPADAKIS • CTO • HARRISON • AMIR • USMLE STEPS

Notas del editor

  1. ACJ (artritis cronica juvenil)
  2. El quiste de Baker es una prominencia que aparece en la cara posterior de la rodilla, NO es especifica de la enfermedad y puede darse en otros procesos que aumenten el liquido sinovial como la osteoartrosis. En ocasiones se rompe provocando dolor brusco que hace pensar en tromboflebitis.
  3. OXCA (oxalato calcico), UMS (urato monosodico), PPCD (pirofosfato calcico dihidratado) HA (hidroxiapatita)
  4. El manejo esta en hemato